Вы находитесь на странице: 1из 82

Да помогут нам высшие силы Аумин!!!!!

1.Девочка 4-месячного возраста доставлена на прием к врачу в связи с отрыжкой и рвотой,


возникающими через 10-15 минут после кормления в течение последних 3-х недель. Она
на смешанном вскармливании. Родилась на 38-й неделе, вес при рождении 2966 г. При
осмотре – выглядит здоровой, вес на приеме – 5878 г. Живот мягкий, безболезненный.
Какой диагноз наиболее вероятен?
А) +Доброкачественный гастроэзофагеальный рефлюкс (ГЭР)
В) Гастроэзофагеальнаярефлюксная болезнь (ГЭРБ)
С) Синдром циклической рвоты
D) Гипертрофический пилорический стеноз
Е) Синдром мальабсорбции

2. Мальчик 3 мес, родители отказались от второй вакцинации против гепатита В в срок 4-8
нед. С родителями была проведена беседа, они согласились вакцинировать ребенка.
Когда необходимо провести вторую вакцинацию против гепатита В ребенку, не
получившему вакцину в возрасте 4-8 недель?
А) в 3-х месячном возрасте
В) +:в 4-х месячном возрасте
С) в 5-х месячном возрасте
D) в 6-х месячном возрасте
Е) в 7-х месячном возрасте

3. Женщина 42 лет, обратилась с жалобами на боль в животе, снижение веса,


пальпируемое образование в правом подреберье и необъяснимое ухудшение состояния.
Выслушивается шум трения брюшины над печенью. Состоит на диспансерном учете.
Какой из перечисленных показателей возможно использовать в качестве скринингового
теста гепатоцеллюлярной карциномы?
А) +Альфа-фетопротеин (АФП)
В) Билирубин
С) D-димер
D) Креатинин
Е) Гликированный гемоглобин

4. 42-летняя женщина обратилась по поводу 2-кратной потери сознания на прошлой


неделе, на которые она не обратила внимания. В течение последних 5 месяцев появились
сердцебиение и одышка, из-за которых не может выполнять повседневную домашнюю
работу. Ничего не принимает. Т тела 37.3ᴼС, PS 115 уд/мин, нерегулярный, АД 108/70 мм
рт ст. При осмотре - видимая пульсация ПЖ, пальпируемая у левого края грудины,
набухание яремных вен. Аускультативно - усиленный ("хлопающий") I тон,
выслушивается акцент II тона над лёгочной артерией.
Какой диагноз наиболее вероятен?
A) Недостаточность митрального клапана
B) Недостаточность клапана аорты
C) Стеноз клапана аорты
D) +Стеноз митрального клапана
Е) Пролапс митрального клапана
5.61-летний мужчина обратился по поводу утомляемости, диареи и схваткообразных
болей в животе в течение 3-х недель. Боль усиливается после еды. В течение прошлой
недели было до 4 водянистых испражнений ежедневно. Также беспокоят боли в деснах и
во рту в течение 6 дней. Принимает левотироксин, метопролол, варфарин. Курит по 1
пачке сигарет в день в течение 40 лет. Т тела 37, 9ᴼС, PS 81/мин, АД 120/75 мм рт ст.
Отмечается легкая болезненность при пальпации в правом нижнем квадранте. ОАК: Hb
115 г/л, лейкоцитов – 11.8х109\л, тромбоцитов – 360. На колоноскопии – неказеозные
гранулемы и нейтрофильное воспаление крипт.
Какой диагноз наиболее вероятен?
A) Дивертикулит
B) Ишемический колит
C) Болезнь Бехчета
D) Болезнь Уипла
E) +Болезнь Крона

6. 15-летний подросток обратился с жалобами на высокую температуру, отеки лица,


голеней, боль в поясничной области, уменьшение количества мочи. 3 недели перенес
скарлатину. Два дня назад появились вышеуказанные жалобы. Объективно: состояние
средней тяжести. Отечность лица, голеней. Температура тела 38,5°C. Симптом
поколачивания положительный с обеих сторон. Общий анализ мочи: реакция щелочная,
цвет «мясных помоев», относительная плотность – 1028, белок – 0,6 г/л, эритроциты – все
поле зрения, лейкоциты – 3-5 в п/з, цилиндры гиалиновые 5-7 в п/з, зернистые – 2-4 в п/з.

Какой диагноз наиболее вероятен?


A) Острый пиелонефрит
B) Острый тубулоинтерстициальный нефрит
C) Хронический пиелонефрит, обострение
D) Хронический гломерулонефрит
E) +:Острый гломерулонефрит

7. 75-летний мужчина жалуется на сильную, распирающую боль над симфизом, слабость,


отсутствие мочеиспускания в течение суток. В анамнезе: состоит на диспансерном учете с
артериальной гипертензией 1 стадии, сахарным диабетом 2 типа, доброкачественной
гиперплазией простаты. Лечится регулярно. Объективно: кожные покровы бледные,
влажные. Тоны сердца глухие, ритмичные, пульс 100ударов в мин, артериальное давление
160/90 мм рт.ст. Перкуторно мочевой пузырь определяется на 10 см над лобком.
Какое мероприятие является первоочередным в данном случае?
A) +Выведение мочи катетером
B) Инъекция дротаверина
C) Назначение альфа-адреноблокаторов
D) Инъекция эналаприлата
E) Экстренная госпитализация

8. Мужчина 57 лет, в анамнезе ИБС. Стабильная стенокардия. Сахарный диабет 2 типа.


Какой целевой уровень артериального давления наиболее приемлем для данного пациента
(протокол МЗСР РК «ИБС. Стенокардия» от 30.11.
2015 г.)?
A) Ниже 130/80 мм рт ст.
В) Ниже 130/85 мм рт ст.
С) Ниже 135/85 мм рт ст.
D) +Ниже 140/85 мм рт ст.
Е) Ниже 140/90 мм рт ст.

9. Женщина 47 лет отмечает боли в области сердца жгучего характера, купируемые


приемом валидола или седативных микстур. Часто возникает чувство жара,
сопровождающееся гиперемией лица, потливостью. У женщины аменорея в течение 6
месяцев. При осмотре: кожные покровы влажные, гиперемированные. ЧСС-102 в мин,
АД-145/85 мм рт. ст. Анаприлиновая проба положительна. Влагалищный мазок
атрофический. Какой из перечисленных диагнозов НАИБОЛЕЕ вероятен?
a) ИБС. Стенокардия Принцметала
b) +Патологический климакс
c)НЦД по гипертоническому типу
d)Артериальная гипертензия
e) ИБС. Стенокардия напряжения ФК III

10. Женщина 32 лет. Беременность 26-27 недель. Беременность первая. При очередном
обследовании выявлена гликемия 6.2 ммоль/л. Обследование 2 года назад гликемия - 4,8
ммоль/л. Какой прогноз для ребенка наиболее вероятен в данном случае?
A. вероятность задержки внутриутробного развития
B. высокий риск развития ВПС
C. вероятность веса меньше 2500 г при рождении
D. +высокий риск крупного плода
E. высокий риск преждевременных родов

11. Женщина 27 лет, во время беременности при возникновении угрозы прерывания


беременности был назначен дюфастон (дидрогестерон). Через 2 недели после приёма
дюфастона отмечались кожный зуд, повышение показателей трансаминаз (АЛТ – 885 ед/л,
АСТ – 447 ед/л), которые снизились после отмены препарата. Родила здорового ребенка.
Какие рекомендации по контрацепции целесообразны в послеродовом периоде?
A. контрацепция мини-пили
B. контрацепция дезагистрел
C. контрацепция левоноргистрел
D. контрацепция депо-проверой
E. +контрацепция барьерными средствами

12. Мама ребенка 4-х лет, обратилась с жалобами на общее недомогание, субфебрильную
температуру, частые ОРВИ. Из анамнеза: установлен однократный контакт с больным
активным туберкулезом дядей. Объективно: состояние средней тяжести, умеренные
симптомы интоксикации, температура - 37,5 0. Бледные кожные покровы, пониженного
питания. Физикальные данные – без патологии. На рентгенограмме: негомогенное
затемнение в верхней доле правого легкого, связанное дорожкой с увеличенным
трахеобронхиальным лимфоузлом. Какое из перечисленных исследований необходимо
провести в первую очередь для уточнения диагноза?
a) +Проба Манту и Диаскинтест
b) Микроскопия и бакпосев мокроты на МБТ
c) Компьютерная томография легких
d) Общий и биохимический анализ крови и мочи
e) ИФА и ПЦР диагностика
13. Женщина 27 лет на сроке беременности 6-7 недель обратилась с целью искусственного
прерывания беременности. Две недели назад перенесла ОРВИ в тяжелой форме, по поводу
которого лечилась амбулаторно. Женщина является гражданкой РК и имеет постоянную
работу. Какой документ следует ей выдать?
a) +Лист о временной нетрудоспособности
b) Невостребованный бланк листа о временной нетрудоспособности
c) Справка о временной инвалидности
d) Заключение врачебно-консультативной комиссии
e) Справка о временной нетрудоспособности

14. Студент одного из вузов обратился к участковому врачу. Ему выставлен диагноз
"Внебольничная пневмония, легкое течение". Лечение было проведено амбулаторно.
Какой документ о временной нетрудоспособности ему следует выдать?
a) Лист о временной нетрудоспособности
b) Невостребованный бланк листа о временной нетрудоспособности
c) Справка о временной инвалидности
d) Заключение врачебно-консультативной комиссии
e) +Справка о временной нетрудоспособности

15. Мужчина, будучи в состоянии алкогольного опьянения получил перелом лучевой


кости. Первую помощь была ему оказана в травмпункте. Какой документ о временной
нетрудоспособности ему следует выдать?
a) Лист о временной нетрудоспособности
b) Невостребованный бланк листа о временной нетрудоспособности
c) Справка о временной инвалидности
d) Заключение врачебно-консультативной комиссии
e) +Справка о временной нетрудоспособности

16. Мужчина 35 лет, гражданин Российской федерации. Обратился по поводу ОРВИ. При
осмотре состояние средней тяжести, лихорадка 38.0°С, катаральные явления. Работает в
одной из компаний в Астане. Какой документ о временной нетрудоспособности ему
следует выдать?
a) +Лист о временной нетрудоспособности
b) Невостребованный бланк листа о временной нетрудоспособности
c) Справка о временной инвалидности
d) Заключение врачебно-консультативной комиссии
e) Справка о временной нетрудоспособности

17. Мужчина 45 лет, обратился к участковому врачу с жалобами на преходящие боли за


грудиной, которые появились 2 недели назад, длительностью 1-2 мин, проходят
спонтанно, возникают при физической нагрузке. Курит 1 пачку в день. Объективно: ИМТ
- 32, АД 135/65 мм рт.ст. ЧСС 75 в мин, тоны сердца ясные ритмичные. Какое
обследование показано на первом этапе диагностического поиска?
a) Р-графия органов грудной клетки
b) Эхокардиография
c) +Электрокардиография
d) Допплер-сонография сонной артерии
e) Проба с эргометрином
18. В школе был проведен профилактический осмотр детей младших классов. Было
проведено обследование 25 детей. В какую форму медицинской документации должны
быть внесены результаты скриннинга?
a) +ф. 026/у
b) ф.030/у
c) ф. 076/у
d) ф. 080/у
e) ф.112/у

19. Пациент 22 лет, обратился к терапевту с жалобами на повышение температуры до


37,7о по вечерам, потливость, слабость, недомогание, кашель с мокротой около 2-х
недель, боль в грудной клетке справа, потерю веса. Какое мероприятие из перечисленных
является наиболее целесообразным?
a) +Исследование мокроты на МБТ, обзорная рентгенограмма
b) Бронхоскопия, обзорная рентгенограмма
c) Спирография, обзорная рентгенограмма
d) Спирография. Бронхоскопия
e) УЗИ плевральной полости, рентгенография

20 Мужчина 52 лет, житель Кызылординской области, чабан, находился на стационарном


лечении с 1 июня по 21 июня. Жалобы при поступлении на повышение температуры тела
до 39,9°С, сильную головную боль, озноб, ломоту в теле, носовое кровотечение.
Объективно: состояние тяжелое, температура - 39,8°С, на туловище необильная
петехиальная сыпь, в местах инъекций гематомы. ЧСС – 100 в 1 мин., АД-90/60 мм рт.ст.
Печень увеличена на 1,0 см. Больной выписан через 21 день с улучшением состояния.
Какова наиболее оптимальная длительность срока диспансерного наблюдения за
переболевшим ?
a) +:12 месяцев
b) 6 месяцев
c) 3 месяца
d) 2 месяца
e) 1 месяц

21. Женщина 53 лет жалуется на одышку при физической нагрузке, кашель с


трудноотделяемой мокротой, сердцебиение. Более 20 лет страдает бронхолегочной
патологией, обострения по 2-3 раза в год в холодное время. Об-но: пальцы в виде
«барабанных палочек», бочкообразная грудная клетка. При аускультации по всем полям
сухие жужжащие хрипы, выдох удлинен. ЧД 26 в мин. Данные спирографии: ОФВ1 68 %
от должного; ОФВ1/ФЖЕЛ 95%. Какая тактика ведения пациентки целесообразна для
предупреждения осложнений основного заболевания в осенне-зимний период?
a) профилактическое назначение антибиотиков группы макролидов
b) увеличивать дозировку ипратропия бромида в осенне-зимний период
c) проводить курсы инозина пранобекса с целью иммуностимуляции
d) проводить курсы УФО в осенне-зимний период
e) +проводить плановую вакцинацию от гриппа в сентябре-октябре

22. Мужчину 42 лет в течение 2 лет беспокоят боли за грудиной сжимающего характера,
иррадиирующие в левое плечо и лопатку, возникающие при быстрой ходьбе и
купирующиеся в покое. Ухудшение самочувствия связывает с ночными сменами и
переутомлением. Об-но: АД 140/80 мм рт.ст. Тоны сердца ясные, ритм правильный, ЧСС-
72 уд/мин. Велоэргометрическая проба показала высокую толерантность к физической
нагрузке. Какой из перечисленных диагнозов НАИБОЛЕЕ вероятен?
a) +:Стабильная стенокардия напряжения ФК I
b) Стабильная стенокардия напряжения ФК II
c) Стабильная стенокардия напряжения ФК III
d) Впервые возникшая стенокардия
e) Прогрессирующая стенокардия

23. Мужчина 44 лет, АД поднимается до 150-160/90-95 мм рт.ст. в течение 5 лет.


Сахарный диабет 2 типа, принимает диабетон. Объективно: левая граница по левой
средне-ключичной линий. В легких везикулярное дыхание. Тоны сердца ясные, ритм
правильный. ЧСС- 80 в мин. АД- 160/94 мм.рт.ст. Холестерин сыворотки -6,0 ммоль /л,
(до5) креатинин сыворотки - 75 мкм/л (44-106) . Сахар крови-5,4 ммоль/л. МАУ-100 мг в
сутки. (микроальбуминурия) Назначение какого гипотензивного препарата НАИБОЛЕЕ
целесообразно?
a) каптоприл
b) бисопролол
c) гидрохлортиазид
d) +валсартан
e) нифедипин

24. Мужчина 68 лет. В последние 2 года отмечает появление перебоев в работе сердца,
учащенное сердцебиение. Артериальная гипертензия 2 степени и ИБС в течение 15 лет.
При аускультации сердца: ритм неправильный, периодически «пушечный тон», число
сердечных сокращении 100 в мин, пульс-86 в мин., нерегулярный. Что Вы, ожидаете
увидеть на ЭКГ больного?
a) +зубцы P отсутствуют, расстояния R-R разные, волны f во II, III. аvF
b) наличие внеочередных QRS и компенсаторных пауз
c) периодическое выпадение QRS после зубца Р
d) расстояния R-R одинаковые, зубец Р перед узким QRS
e) расстояния R-R одинаковые, зубец Р перед QRS, Т отрицательный

25. В родильном доме родился мальчик с массой тела 2100,0 гр. На 2-й день жизни
появилась желтушность кожных покровов, которая стала нарастать. Объективно:
гепатомегалия, петехиальная сыпь на коже туловища. В анализах: в мазке из слизистой
щек обнаружены гигантские многоядерные клетки с внутриклеточными включениями в
ядрах и цитоплазме. НАИБОЛЕЕ вероятный предварительный диагноз
a) Токсоплазмоз
b) Листериоз
c) +:ЦМВ инфекция
d) Сифилис
e) Иерсиниоз

26. Мальчик 12 лет, 2 года назад после ангины появились гиперкинезы, непроизвольные
движения конечности и тремор кистей, артрит левого коленнного сустава, коротким
курсом принимал преднизолон 20 мг/сутки и антибиотики с положительным эффектом. В
настоящий момент проходит диспансерный осмотр, жалоб не предъявляет. При
физикальном исследовании выявляется систолическим шум на 2 и 5 точке выслушивания
и там же 2 тон акцентирован, ослабления Iтона на верхушке. Печень не увеличена, отеков
нет. Анализ крови: Hb - 120 г/л, лейкоциты - 5,0х109/л, СОЭ - 10 мм/час, С-реактивный
белок – 3,4 мг/мл, АСЛ-О - 1:100МЕ/мл. Какая продолжительность вторичной
профилактики бензатинбензилпенициллином наиболее целесообразно у пациента?
a) До достижения 25-летнего возраста
b) До 3 лет
c) До 10 лет после атаки
d) +:До достижения 21-летнего возраста
e) До достижения 40-летнего возраста

27. Мальчик 8 лет, в сопровождении матери обратился к врачу с жалобами на лихорадку


на 37,8 º, припухлость левого голеностопного сустава с выраженным ограничением
движения, выраженную слабость. Из анамнеза: частые ангины и ОРВИ 1-2 раза в месяц,
неделю назад лечился тонзиллитом. Вчера возобновилась лихорадка и припухлость в
суставе Объективно: артрит левого голеностопного сустава. Анализ крови: Hb - 120 г/л,
лейкоциты - 15х109 /л, СОЭ - 32 мм/час, СРБ– 42 мг/мл, АСЛ-О - 1:450 МЕ/л, РФ – 10
МЕ/мл. Рентгенография голеностопных суставов – патология не выявлена. Какой метод
обследования наиболее достоверно позволит подтвердить диагноз?
a) +:мазок из зева на предмет В-гемолитический стрептококк группы А
b) РПГА на Y.entеrocolitica
c) мазок из носа на предмет золотистого стафилококка
d) бактериальный посев крови
e) бактериальный посев кала

28. У девочки 12-ти лет во время пребывания на юге появились отеки лица, стоп, боли в
мелких суставах кистей. Ранее ничем не болела. Вчера появилась головные боли и в
пояснице на фоне лихорадки. В крови: Hb – 100 г/л, лейкоциты – 3,0х109/л, СОЭ – 31
мм/час; АНФ - 1:80. В анализе мочи: белок – 5,3 г/л, эритроциты – 30-40 в поле зрения,
лейкоциты – 8-10 в поле зрения. Какое из нижеперечисленных исследований наиболее
целесообразно провести пациенту для верификации диагноза:
a) +Биопсия почки
b) Рентген кистей
c) ЭхоКГ
d) УЗДГ сосудов
e) Кровь на маркеры гепатитов В и С

29. Мальчик 10 лет жалуется на боли и припухлость левого коленного сустава с


выраженным ограничением движения, появления синяков на коже конечностей. Мама
заметила повышенную кровоточивость десен, носовые кровотечение за последний месяц.
Из анамнеза: отец и тетя мамы страдали повышенной кровоточивостью. При осмотре:
кровоизлияния в подкожную область по типу «синяков», левый коленный сустав увеличен
в объеме, кожа над ним гиперемирована и горячая на ощупь. ОАК: Hb – 100г/л,
лейкоциты – 4,6х109/л, тромбоциты – 177х109 /л, СОЭ – 25 мм/час; ОАМ – гематурия;
Коагулограмма: АЧТВ – 60 сек, ТВ – 15 сек, фибриноген – 3 г/л, ристоцетин-кофакторная
активность – в норме; РФ – 7 МЕ/мл, СРБ – 22 мг/мл. Рентген коленных суставов (фото)
Какой метод обследование целесообразно провести для верификации диагноза?
a) маркеры гепатитов В и С
b) +определения факторов свертывания крови VII и IX
c) кровь на ВИЧ
d) определение фактора Виллебранда
e) определения HLA-B27

30 На приеме у врача ребенок 9 мес. На грудном вскармливании. Объективно: снижен


мышечный тонус, не ползает, живот увеличен, при сидении выражен кифоз грудного
отдела позвоночника. Со стороны костной системы макроцефалия, большой родничок 2х2
см, уплощение и облысение затылочной кости, лобные и теменные бугры, рахитические
четки, гаррисонова борозда Масса тела 9200 г. Задержки физического и психомоторного
развития нет. Врач назначила лечение. Укажите дозу холекальцеферола (вит Д3) для
лечения гиповитаминоза у данного ребенка:
a) 500 МЕ/сутки в течение 1 года
b) 1000-1500 МЕ/сутки в течение 1 мес
c) 1500-2000 МЕ/сутки в течение 1 мес
d) +2000-2500 МЕ/сутки в течение 1 мес
e) 3000-3500 МЕ/сутки в течение 1 мес

31 Мать ребенка 5 лет обратилась к врачу по поводу частого сухого кашля, беспокоящего
ребенка ночью и рано утром в течение трех месяцев. В последнее время после
физической нагрузки (бег, прыжки) периодически возникает свистящее дыхание, которое
самостоятельно проходит после отдыха. Что может явиться причиной данного состояния:
a) ВПС с гиперволемией в малом круге кровообращения
b) острый неревматический кардит, НК 2 А.
c) хронический бронхит
d) +бронхиальная астма
e) рецидивирующий обструктивный бронхит

32 Женщина 37 лет, обратилась к врачу с жалобами на боль и припухлость


лучезапястного сустава, сильную болезненность и слабость в руке, невозможность сжать
кисть и удержать в ней предмет; субфебрилитет по вечерам, ноющие боли в коленных
суставах. Хирург ввел ей дипроспан внутрисуставно, что дало почти моментальное
облегчение. На следующий день у женщины сустав сильно опух, покраснел, появилась
дергающая боль, поднялась температура. Какой диагноз наиболее вероятен?
a) псевдосептический артрит при ревматоидном артрите
b) острый подагрический артрит
c) острый постстрептококковый артрит
d) +:острый гнойный артрит, вызванный стафилококком
e) бруцеллезный артрит, спровоцированный введением дипроспана

33 Мальчик 11 лет, катаясь на роликовых коньках во дворе у дома, упал и сильно


поцарапал кожу в области коленных суставов, ладоней обеих кистей. Загрязненные
землей раны, родители промыли проточной водой и отвезли ребенка в травмпункт.
Медотвод от прививок с 2 лет в связи с бронхиальной астмой. какое решение должен
принять врач:
a) обработать рану перекисью водорода, наложить бактерицидный лейкопластырь
b) обработать рану, наложить повязку и освободить от занятий
c) после обработки раны рекомендовать рентгенографию коленного сустава
d) Прививки против столбняка воздержаться, наблюдать
e) +ввести противостолбнячный и противодифтерийный анатоксины

34 Женщина, 27 лет, обратилась к врачу общей практики с жалобами на выраженную боль


в правом предплечье, отёк, гиперемию в этой области, общую слабость, недомогание,
повышение температуры тела до 38. За четверо суток до этого получила ссадину на
тыльной поверхности правого предплечья. При осмотре тыльной поверхности правого
предплечья определяется плотный воспалительный инфильтрат с отёком, гиперемией и
гипертермией. Пальпации инфильтрата сопряжена с выраженной болезненностью.
Симптом флюктуации положителен. Ваш предварительный диагноз?
a) Закрытый перелом костей правого предплечья.
b) Открытый перелом костей правого предплечья.
c) Вывих лучезапястного сустава.
d) Вывих локтевого сустава.
e) +Флегмона правого предплечья.

35 Два года назад больной перенес холецистэктомию по поводу калькулезного


холецистита. Через 6 месяцев появились боли в правом подреберье, темная моча. В
биохимическом анализе крови – общий билирубин 120 мкмоль/л, прямой 87 мкмоль/л.
Какой из перечисленных методов диагностики является в данном случае НАИБОЛЕЕ
информативным?
a) УЗИ
b) сцинтиграфия печени
c) внутривенная холангиография
d) +эндоскопическая ретроградная панкреатохолангиография
e) лапароскопия.

36 На приеме у врача общей практики женщина 53 лет. После трагической смерти мужа,
больная обвиняет себя в том, что она не смогла предотвратить гибель мужа, недостаточно
была внимательна к нему, со слезами рассказывала об этом врачу, высказывала
суицидальные мысли. О каком синдроме идет речь?
a) +Депрессивный синдром
b) Кататонический синдром
c) Обсессивный синдром
d) Паранойяльный синдром
e) Дементный синдром

37 На приеме у врача общей практики мальчик 11 лет. Бабушка привела его на


обследование в связи с тем, что он отстает от школьной программы. Мальчик легко идет
на контакт, речь развита, читает, кругозор не соответствует возрасту, однако память
хорошая, способность к абстрактному мышлению не ограничена. Из анамнеза:
беременность и роды без особенностей, раннее развитие без отставания в психомоторном
развитии. Последние годы родители выпивают, мальчик часто пропускает школу. О каком
состоянии можно думать?
a) Шизофрения
b) +Педагогическая запущенность
c) Расстройство личности
d) Психический инфантилизм
e) Деменция

38 Вызов врача общей практики на дом. У 2-х летнего ребенка клиника вирусной
инфекции с подъемом температуры до 39°. Со слов родителей на высоте температуры
ребенок внезапно «вытянулся», мышцы верхних конечностей были напряжены, затем «как
плеть повис», изо рта выделилась слюна, затем заснул. Ваш предварительный диагноз?
a) Эпилептический припадок.
b) Нейролептический синдром.
c) +:Фебрильные судороги.
d) Внутричерепная гипертензия.
e) Неврозоподобный синдром.

39 У 26-летнего мужчина возникла сильная загрудинная боль. В анамнезе отрицает прием


наркотиков и алкоголя, не курит. На ЭКГ – подъем сегмента ST выше изолинии в 5
грудных отведениях. Употребление какого вещества, наиболее вероятно, будет
причиной?
a) Фенциклидин
b) Седативные средства
c) Галлюциногены
d) +Стимуляторы
e) Алкоголь

40 Девочка 15 лет обратилась к врачу терапевту. При этом она сообщила, что имеет
проблему - ест слишком много и быстрее, чем обычно, и не может контролировать эти
эпизоды, обычно ест в одиночестве. После таких «приступов обжорства» выпивает
большое количество слабительных средств. Какой из нижеперечисленных симптомов
наиболее вероятен?
a) +Булимия
b) Нарушения контролирования импульса
c) Анорексия
d) Навязчивые мысли
e) Сверхценные идеи

41 Мужчина, 60 лет, при осмотре на дому, жалуется на затруднение речи, слабость, в


правой руке и ноге. Из анамнеза: заболел остро, утром после сна обнаружил
вышеуказанные жалобы. Два дня назад, отмечались эти же симптомы, прошли
самостоятельно через час. АД=110/70 мм.рт.ст., ЧСС=110 уд/мин. При осмотре: сознание
ясное, ориентирован, зрачки OD=OS, краевое недоведение глазных яблок с двух сторон (2
мм.). Язык девиирует вправо, сухожильные рефлексы D> S, рефлекс Бабинского справа,
мышечная сила в правых конечностях-3балла, менингиальных знаков нет. Какая тактика
лечения показана больному?
a) оставить больного на дому, под наблюдением семейного врача.
b) госпитализация в нейрососудистое отделение на следующий день, с целью
нейропротекции
c) +:срочная госпитализация в нейрососудистое отделение, для проведения
тромболизиса
d) госпитализация в нейрососудистое отделение через неделю, с целью
нейропротекции
e) срочная госпитализация в нейрохирургию, для проведения операции

42 У юноши 19 лет после гриппа появились боли в нижней челюсти справа


приступообразного характера, продолжительностью 2-3 сек. Приступы возникают при
разговоре, жевании, умывании. В неврологическом статусе: болезненность в точке
выхода III ветви V нерва справа, курковая зона в области угла рта справа. Другой
неврологической симптоматики нет. Глазное дно без патологии. МРТ головного мозга без
патологии. Какой клинический диагноз наиболее вероятный?
a) +:невралгия III ветви тройничного нерва справа
b) острая невропатия лицевого нерва справа
c) невралгия I ветви тройничного нерва справа
d) невралгия II ветви тройничного нерва справа
e) острая невропатия подъязычного нерва справа

43 Мужчина 72 лет был обнаружен лежащим на скамье в сквере. Сознание не терял. При
осмотре на месте врачом скорой помощи выявлено нарушение движений в правой руке и
затруднение речи – произносил отдельные слова, из которых можно понять, что у него
внезапно возникло головокружение. Рвоты не отмечалось. Доставлен в приемное
отделение. При осмотре: сознание сохранено, но вял, апатичен. В речевой контакт не
вступает. На обследование реагирует гримасой неудовольствия. Пульс аритмичный, 104
удара в минуту, тоны сердца глухие, АД 150/100 мм рт.ст. Правый угол рта опущен.
Правая рука неподвижна. Правая стопа повернута кнаружи. Сухожильные рефлексы
справа выше, чем слева. Справа рефлекс Бабинского. Какое обследование должно быть
проведено на следующем диагностическом этапе?
a) Электроэнцефалография
b) +:магнитно-резонансная томография
c) общий анализ крови
d) биохимический анализ крови
e) электрокардиография

44 Юноша 16 лет, жалобы на высокую температуру, головную боль, головокружение с


тошнотой, эмоциональную лабильность, иногда отмечаются галлюцинации. Данное
состояние беспокоит в течении трех дней. Объективно: гиперкинезы в конечностях,
сходящееся косоглазие, диплопия, гипергидроз, легкая пирамидная недостаточность со
снижением мышечного тонуса. В анамнезе частые ангины. АСЛ-О – 220 МЕ/мл. Какой
предварительный диагноз наиболее вероятный?
a) Нейросифилис
b) Нейробруцеллез
c) хорея Гентингтона
d) +:нейроревматизм
e) рассеянный склероз

45. Девушка-подросток, 18 лет имеет сексуально активную жизнь и часто меняет


сексуальных партнеров. Какой метод контрацепции наиболее приемлем ей?
A. +Презерватив
B. Биологические методы
C. Комбинированные оральные контрацептивы
D. Внутриматочная спираль
E. Диафрагма

46 Мужчина 55 лет, жалобы на кашель в течение дня с мокротой, одышку, возникающую


при быстрой ходьбе, небольшом подъеме. В анамнезе - курит в течение 25 лет по 1 пачке
(20 сигарет) в день. Отмечает усиление одышки при простудных заболеваниях в течение
последних 5 лет.
Какова степень выраженности одышки по шкале MedicalResearchCouncilDyspneaScale
(MRS)?
A) степень 0
B) +:степень 1
C) степень 2
D) степень 3
E) степень 4

47 К врачу общей практики на скрининговый осмотр пришел молодой мужчина М., 37


лет. Жалоб не предъявляет. Курит. В анамнезе: отец болен ишемической болезнью сердца
и АГ. По результатам физикального осмотра ИМТ - 34, периодическое повышение
артериального давления до 135-140/85 мм РТ ст. К какой группе динамического
наблюдения относится этот пациент?
a) +:Д-IБ
b) Д-IА
c) Д-II
d) Д-III
e) Д-IV

48. 70-летний мужчина обратился в связи с болями и судорогами, покалываниями в


нижних конечностях в течение последних 6 месяцев. Симптомы усиливаются при ходьбе
более 500 м, проходят в покое. В анамнезе сахарный диабет 2 типа. Курит в течение 50 лет
по 1 пачке сигарет в день. Не пьет алкоголь. Принимает метформин и аспирин. Снижение
температуры тела в области голеней.
Что является наиболее целесообразным в ведении этого пациента?
A) Ношение компрессионыхчулков
B) +Назначение антиагрегантов
C) Эндарэктомия
D) Операция шунтирования
E) Эндовенозная термическая абляция

49. Мужчина 55 лет вызвал врача на дом с жалобами на повышение температуры,


головную боль, сухой кашель. Объективно: Температура 39,20С, сосуды склер
инъецированы, в зеве слизистая ярко гиперемирована, в легких жесткое дыхание, хрипов
нет. Тоны сердца приглушены, ЧСС- 124 уд/мин, АД-110/70 мм.рт.ст., ЧДД – 18. В OAK:
эр.-3,4х1012 /л, гем. -135 г/л, Л. -4,2x109 /л, э. – 5%, п\я – 7%, с\я – 63%, м – 5%, л – 20%,
СОЭ - 18мм/ч. В городе объявлена эпидемия гриппа. Какая тактика наиболее
целесообразна?
a) Анаферон
b) Ремантадин
c) +:Осельтамивир
d) интерферон рекомбинантный альфа-2b
e) имидазолилэтанамидпептан-дионовой кислоты

50. Юноша 15 лет через 2 недели после перенесенной лакунарной ангины предъявляет
жалобы на боли в крупных суставах, боль, припухлость и ограничение подвижности
коленного сустава, боль очень сильная, в\м введение кетонала почти не снимает боль.
Температура – 37,5°. ОАК: Л- 9х10 /л, Нб – 123 г/л, СОЭ 25 мм/ч. ЭКГ – синусовая
тахикардия. Р-графия ОГК – без особенностей. Какое обследование должно быть
проведено в обязательном порядке для верификации диагноза?
a) ревматоидный фактор
b) антинуклеарные антитела
c) маркеры гепатитов В и С
d) антитела к парвовирусу В19
e) +антистрептококковые антитела

51. В поликлинике в течение дня был проведен скрининг 28 взрослых от 40 до 55 лет. В


какую форму медицинской документации должны быть внесены результаты скриннинга?
a) +ф. 025-08/у
b) ф.030/у
c) ф. 076/у
d) ф. 080/у
e) ф. 112/у

52 Мужчина 45 лет обратился с жалобами на боль и припухлость в левом коленном


суставе, боли мигрирующего характера в других крупных суставах, мышечные боли,
головные боли; повышение температуры тела до 37,5° по вечерам, периодически
температура повышается до 38,5° по несколько дней; снижение массы тела на 5 кг. Все
симптомы появились после переохлаждения 6 месяца назад. В анамнезе – частое
употребление молочных продуктов кустарного производства. Объективно: Гипергидроз
ладоней и стоп, несимметричная припухлость коленного сустава, локальная температура,
положительный симптом баллотирования наколенника. ОАК: Эр – 3,2, Нб – 110 г/л, Л- 4,9
х109/л, Нф – 68%, Лф – 38%, СОЭ 29 мм/ч. РФ – 22 МЕ/мл, СРБ - +++. РХ – отрицательно
(реакция агглютинации пластинчатая Хеддльсона); РА – отрицательно (реакция
агглютинации Райта. Какое обследование с наибольшей достоверностью подтвердит
диагноз?
a) Антицитруллиновые антитела
b) Рентгенография коленных суставов
c) Исследование мочевой кислоты в крови
d) +ИФА на антитела к бруцеллезным антигенам
e) Проба Манту и Р-снимок органов грудной клетки

53. Мужчина 48 лет, обратился к врачу с жалобами на кашель с выделением гнойно-


слизистой мокроты, чаще по утрам. Кашель беспокоит в течение 15 лет. Вредные
привычки - курит с 20 лет по 1 пачке в день. Отмечает обострения заболевания около 3 раз
в год. Постбронходилатационный тест: ОФВ1<30% от прогнозируемого. Оценка CAT=18
баллов. По шкале mMRC 4. К какой категории групп GOLD (2019 Global Initiative for
Chronic Obstructive Lung Disease) относится пациент?
A) GOLD 1, категория А
B) GOLD 2, категория В
C) GOLD 3, категория В
D) GOLD 4, категория В
E) +:GOLD 4, категория D
54. 57-летний мужчина обратился к врачу из-за усиления в течение 2 месяцев одышки при
ходьбе. В анамнезе гиперхолестеринемия, принимает симвастатин. В течение 35 лет
работал в компании по сносу домов. Курит 33 года по 1 пачке сигарет в день.
Выслушиваются мелкие хрипы в обоих легких. На рентгене – диффузные инфильтраты в
нижних долях и кальцинированные плевральные бляшки. Какое из состояний разовьется у
пациента?
A) Карцинома щитовидной железы
B) Туберкулез
C) Саркоидоз
D) Мезотелиома
E) +Бронхогенная карцинома

55. 59-летняя женщина обратилась по поводу эпизодического кашля и одышки в течение 1


мес. Кашель непродуктивен, усиливается при подъеме по лестнице и ночью. 8 недель
назад у нее была температура, боль в горле и заложенность носа. В течение 10 лет АГ.
Выкуривает по полпачке сигарет в день ежедневно в течение 16 лет. PS 78/мин, ЧДД
18/мин, АД 145/95 мм рт ст. Пульсоксиметрия 96%. Аускультативно – диффузные хрипы
в конце выдоха. Спирометрия: FVC 65% и FEV 60%.
Какой диагноз наиболее вероятен?
A) +Бронхиальная астма
B) Внеболничная пневмония
C) Хроническая сердечная недостаточность
D) Дефицит α1-антитрипсина
E) ГЭРБ

56. 52-летний мужчина обратился по поводу потери веса 3.6 кг в течение 3-х месяцев,
утомляемости. 6 лет назад был выставлен диагноз гепатит С. Его отец умер от рака
толстой кишки. Курил 1 пачку сигарет в день в течение 35 лет и выпивал пиво. В прошлом
– употреблял героин. Рост 175 см, вес 71 кг, ИМТ=22.9 кг/м2. Склеры желтые,
двустороннее покраснение ладоней, несколько телеангиоэктазий на груди и спине. Печень
плотная, узелковой консистенции. ОАК: Hb 116 г/л, лейкоцитов – 9,6х109\л,
тромбоцитов– 223. Какой результат ожидаем при дальнейшей оценке пациента?
A) Положительные культуры крови
B) +:Повышенный α-фетопротеин
C) Экзофитная опухоль при колоноскопии
D) Повышенный карциноэмбриональный антиген
E) Поражение легких на рентгенограмме

57. Мужчина 63 лет. Жалобы на боли в левой половине грудной клетки, которые
появляются при физической нагрузке, повышение АД. Боли беспокоят в течение
нескольких месяцев. При осмотре (фото). АД – 160/90 мм рт.ст. При аускультации акцент
2 тона на аорте. Какой результат липидограммы следует ожидать у больного?
a) снижение триглицеридов
b) повышение альфа-липопротеидов
c) повышение липопротеидов высокой плотности
d) +:снижение липопротеидов высокой плотности
e) снижение липопротеидов низкой плотности

58) Ребенок 3 месяцев в тяжелом состоянии. Болен 4 день. Одышка, пероральный цианоз
в покое. Частота дыхания 84 в мин, дыхание шумное, слышно на расстоянии. Умеренные
проявления токсикоза с эксикозом. Над легочными полями перкуторно - тимпанит,
аускультативно: удлиненный затрудненный выдох, по всем полям влажные
мелкопузырчатые хрипы. SO2 89 %. ОАК RBC 4,3х1012/л, HB 112 г/л, PLT 210х109/л, 1
WBC 12,1х10х109/л с/ядерные 52%, эозинофилы 2%, моноциты 7%, лимфоциты 38% СОЭ
20 мм/час Какой из перечисленных ниже диагнозов наиболее вероятный:
a) острый бронхит
b) +:острый бронхиолит
c) острый обструктивный бронхит
d) двусторонняя очаговая пневмония
e) интерстициальная пневмония

59) Мальчик 12 лет жалуется на боли в крупных суставах, припухлость в левом локтевом
суставе, мышечную слабость, лихорадку до 39,8º с потрясающими ознобами. Температура
повышается в утренние часы и после обеда, держится около 2-х часов, далее снижается с
профузным потоотделением до нормы. Лечили антибактериальными препаратами без
эффекта. С момента начала заболевания сильно похудел. При осмотре выявлены
полиморфная сыпь на туловище и конечностях, увеличение подчелюстных и
подмышечных лимфоузлов, и гепатоспленломегалия. ОАК: Hb – 90 г/л, лейкоциты –
20,6х109/л, тромбоциты – 230х109 /л, СОЭ – 55 мм/час. СРБ – 50 мг/мл, АНА –
отрицательно, РФ – 5 Ме/мл, АСЛ-О – 130 МЕ/л, ферритин – 1000 мкг/л, прокальцитонин
– 0,3 нг/мл. С какой патологией необходимо дифференцировать в первую очередь?
a) +Лейкозы
b) системный васкулит
c) системная красная волчанка
d) острая ревматическая лихорадка
e) сепсис

60) Мужчина 64 лет обратился с жалобами на наличие образования на боковой


поверхности туловища справа. Из анамнеза образование заметил 4 месяца назад. В
последнее время оно увеличилось. При осмотре врач определил образование мягко-
эластичной консистенции, подвижное, размерами 2,5х3,0 см. с четкими контурами в
пределах подкожной клетчатки. Какова Ваша дальнейшая лечебная тактика в данном
случае?
a) Рекомендовать наблюдение в динамике
b) Выполнить пункцию образования
c) +Удалить и направить на гистологию
d) Назначить рассасывающую терапию
e) Рекомендовать согревающие компрессы

61) На приеме пациент 72 лет в сопровождении родственников. С их слов у больного


наблюдается ухудшение памяти, особенно в усвоении новой информации. Нарушение
памяти представляет собой серьезное препятствие для повседневной жизни пациента.
Удерживается только очень хорошо усвоенный или очень знакомый материал. Новая
информация удерживается лишь случайно или на очень короткое время. Больной не в
состоянии вспомнить основные сведения о том, где он живет, что он недавно делал или
как зовут его знакомых. Объективно состояние удовлетворительное. АД 150\100 мм рт ст.
пульс 80 в 1 мин. Тоны сердца приглушены, ритм правильный. Пастозность стоп. Ваш
предварительный диагноз?
a) +:Болезнь Альцгеймера.
b) Болезнь Паркинсона.
c) Гипертоническая энцефалопатия.
d) Возрастная деменция.
e) Опухоль мозга.

62) Женщина 65 лет, На фоне повышения АД до 195/110 мм рт.ст. появилось нарушение


речи - внезапно перестала говорить. В неврологическом статусе: сознание ясное, зрачки
D=S, сглажена правая носогубная складка, моторная афазия, правосторонний гемипарез с
высоким мышечным тонусом и высокими сухожильными рефлексами, с симптомом
Бабинского. Какой из нижеперечисленных диагнозов является наиболее вероятным?
a) острая гипертоническая энцефалопатия
b) +:ишемический инсульт
c) транзиторная ишемическая атака
d) серозный менингит
e) геморрагический инсульт

63) Женщина, 27 лет, после профилактического флюороосмотра вызвана на


дообследование. Жалоб не предъявляет. В легких выслушивается везикулярное дыхание.
Рентгенограмма: слева в верхней доле очаговые тени слабой интенсивности на фоне
усиленного легочного рисунка. В анализе промывных вод бронхов МБТ не обнаружены. В
анализе крови лейкоциты - 4,8 х 109, СОЭ – 4 мм/час. Какой из перечисленных ниже
предварительных диагнозов наиболее вероятен?
a) инфильтративный туберкулез верхней доли левого легкого
b) +очаговый туберкулез верхней доли левого легкого
c) карциноматоз верхней доли левого легкого
d) туберкулема верхней доли левого легкого
e) милиарный туберкулез легких
64) Женщина 35 лет, предъявляет жалобы на боли в груди, слабость, кашель. В мокроте
прожилки крови. Субфебрильная температура более 5 дней. Дыхание везикулярное,
хрипов нет. В мокроте МБТ не обнаружено. Было начато лечение антибиотиками. Через
две недели: субфебрилитет сохраняется, в анализе крови: лейкоциты – 9,5 х 10, СОЭ – 20
мм/час. Рентгенограмма: - во 2-м сегменте справа негомогенное, малой интенсивности
затемнение до 4 см в диаметре с нечеткими контурами, с просветлением в центре. Какой
из перечисленных ниже предварительных диагнозов наиболее вероятен?
a) +инфильтративный туберкулез легких
b) эозинофильный инфильтрат
c) абсцедирующая пневмония
d) внебольничная пневмония
e) раковая пневмония

65) Женщина 24 лет с вирусным гепатитом В, беременность 32 недели. Биохимия: АЛТ –


0,64 ммоль/л. АСТ – 0,47 ммоль/л. Билирубин – 22,25 мкмоль/л. HBsAg – положительный,
anti HBs – отр., antiHBc IgM – положит., antiHBc IgG – положит., HBeAg – положит., anti
HBe – отр.;. ПЦР HBV ДНК – 106 копий/мл. Какая тактика наиболее целесообразна для
снижения риска перинатальной передачи вируса ребенку?
a) +:Ламивудин 100 мг\сут до родов
b) Интерферон короткого действия
c) Интерферон в виде свечей до родов
d) Адеметионин+урсодезоксихолевая кислота
e) Эссенциальные фосфолипиды до 38-39 недели

66) Женщина 28 лет. Беременность 14-15 недель. Около недели назад заболела -
поднялась температуратела до 37.5°С, появилась ринорея, сухой кашель. Два назад
состояние ухудшилось - появилась одышка, кашель усилился, температура - 38,5°С.
Объективно - в нижних отделах левого легкого выслушиваются мелкопузырчатые хрипы
на фоне жесткого дыхания. Общее состояние страдает мало. От госпитализации
отказывается. Какая лечебная тактика наиболее целесообразна?
A. гентамицин в/м
B. левофлоксацин
C. азитромицин
D. +:ампициллин
E. метронидазол

67) Больной с диагнозом неспецифический язвенный колит. Болен в течение 10 лет.


Получает поддерживающую терапию месалазином. Придерживается диетических
рекомендаций, принимает пробиотики, не курит. Относительная ремиссия последние 2
года. Какое плановое обследование необходимо провести пациенту с целью раннего
выявления возможного осложнения данного заболевания у этого пациента?
a) +: колоноскопия с прицельной биопсией
b) МРТ органов брюшной полости
c) Альфа-фетопротеин
d) Ирригосокпия
e) ФЭГДС

68) Женщина 80 лет. Отмечает снижение массы тела за последние шесть месяцев на 7
кг, появления отвращения к мясу. В анамнезе - хронический гастрит. Объективно:
бледность кожных покровов, пальпация живота безболезненная. В крови выявлена анемия
2 степени, СОЭ 48 мм рт.ст. Какое обследование необходимо провести для уточнения
диагноза?
a. @+:ФЭГДС
b. @Р-графия ОГК
c. @Уреазный дыхательный тест
d. @Трепанобиопсия костного мозга
e. @ СЕА и СА 19-9

69) Мужчина 45 лет. Жалобы на изжогу, боли и дискомфорт за грудиной в течение


ряда лет. При ФГДС были выявлены участки метаплазии эпителия (фото). С какой
периодичностью больному необходимо проводить контрольное обследование для раннего
выявления возможного осложнения?

a. @+:1 раз в три месяца+


b. @1 раз в месяц
c. @ 1 раз в год
d. @1 раз в два года
e. @ 1 раз в три года

70) У беременной 27 лет на сроке 36 недель отмечается повышение АД до 170/100 мм


рт.ст., увеличение массы тела на 4 кг за неделю, отеки на нижних конечностях,
снижение суточного диуреза до 400 мл. В крови: тромбоциты – 90 тыс, альбумины
– 16 г/л. В моче: белок – 2,9 г/л. На УЗИ: задержка развития плода на 4 недели.
Какая тактика ведения беременной НАИБОЛЕЕ целесообразна?
a. @ Досрочное родоразрешение+++
b. @прерывание беременности с антигипертензивной терапией
c. @пролонгирование беременности до самопроизвольного родоразрешения
d. @пролонгирование беременности с антигипертензивной терапией
e. @ антигипертензивная терапия

71) Женщина 29 лет, беременность вторая, 9-10 недель. Обратилась с жалобами на


тошноту и рвоту по утрам, частотой до 10 раз в сутки в течение последних двух
недель. Для постановки на учет обратилась на сроке 6-7 недель, масса тела на тот
момент - 74 кг. Сейчас вес - 68 кг. Объективно: состояние средней тяжести, сухость
кожи, за время осмотра дважды были позывы на рвоту. Какая тактика наиболее
целесообразна?
@+:Немедленная госпитализация
@лечить амбулаторно, перорально солевые растворы
@лечит в условиях дневного стационара, инфузионно кристалоидные растворы
@решение вопроса о немедленном родоразрешении
@ консультация инфекциониста

72) Женщина 32 лет, наблюдается в женской консультации по поводу беременности.


На 28 неделе обратилась с жалобами на кожный зуд из-за которого плохо спит,
слабость, подташнивание. В биохимическом анализе крови: АЛТ-56 Ед/л, АСТ-42
Ед/л, общий билирубин-12,4 ммоль/л, ЩФ-480 Ед/л. Маркерная диагностика:
HBsAg – отр., a-HBs - положительный, a-HВcore IgG-отр, HBeAg - отр, a-HBe- отр,
anti-HCV IgM - отрицат, anti-HCV IgG – отрицательный. Какой диагноз?
a. @+:холестаз беременных
b. @хронический вирусный гепатит
c. @гепатоз беременных
d. @это нормальное состояние для данного срока беременности
e. @ обострение хронического холецистита

73) Женщина 25 лет, беременность 12-13 недель. Жалобы: на частое болезненное


мочеиспускание, боль над лобком, частые императивные позывы. Данные жалобы
в течение 2-3 дней после переохлаждения. При обследовании ОАК без
особенностей, в ОАМ – лейкоциы до 20-30 в п/зр, бактерии+++. Какой из
перечисленных предварительных диагнозов является НАИБОЛЕЕ вероятным?
a. @+:инфекция мочевых путей+++
b. @острый пиелонефрит
c. @мочекаменная болезнь
d. @бессимптомная бактериурия
e. @ тубулоинтерстициальный нефрит

74) Женщина 28 лет. Месяц назад родила в срок здорового ребенка весом 3300 г, рост
51 см. Особых жалоб не предьявляет, лактация хорошая, ребенок полностью на
грудном вскармливании. При осмотре обращает внимание бледность слизистых,
при расспросе - появилась слоистость ногтей, волосы потускнели и стали больше
выпадать. Питается удовлетворительно, никаких дополнительных препаратов
(поливитаминов и прочее) не принимает. Принимала "Прегнавит" во время
беременности, но посчитала что после родов необходимость в витаминах и
микроэлементах отпала. В крови - гемоглобин 106 г/л, ЦП - 0,7. В течение какого
периода рекомендуется назначение железа в дозе не менее 60 мг в сутки
родильницам?
А. в течение первого месяца после родов
B. в течение трех месяцев после родов
C. в течение шести месяцев после родов
D. до введения первого прикорма
E. не менее 12 месяцев после родов

75) Пациентка 26 лет, беременная в сроке 22-23 недели обратилась к врачу общей
практики (ВОП) с жалобами на головную боль, тошноту, рвоту, боли в правом
подреберье и снижение мочеотделения. При объективном осмотре пастозность
нижних конечностей, АД 150/90 мм рт ст на обеих руках, пульс 96 ударов в одну
минуту. Выберите гипотензивный препарат для данной пациентки?
a. @метилдопа
b. @каптоприл
c. @гипохлортиазид
d. @карведилол
e. @ бисопролол
76) Мужчина 25 лет, с впервые выявленным инфильтративным туберкулезом S1
правого легкого, МБТ+ находился в стационаре 2 месяца, где проведена
интенсивная фаза четырьмя ПТП. Далее ему должна была быть проведена 4-
месячная поддерживающая фаза. Больной не придерживался рекомендованной
схемы лечения. Бактериовыделение прекратилось. На рентгенограмме – в S1
правого легкого определятся четко округлая гомогенная тень, d – 3 см. Какой из
перечисленных ниже диагнозов наиболее вероятен?
a. @+:Туберкулема
b. @склерозировавшийся очаг
c. @периферический рак
d. @заполненная киста
e. @ эхинококк

77) Мужчина 29 лет, обратился к терапевту с жалобами на повышение температуры по


вечерам 38о, слабость, утомляемость, снижение аппетита, кашель с мокротой
слизистого характера с прожилками крови, боль в грудной клетке справа, снижение
веса за последние 3 месяца на 2 кг. Туб. контакт отрицает. В легких справа
выслушиваются единичные мелкопузырчатые влажные хрипы. В анализе крови: эр
– 3,28х 1012, Нb – 138, э – 2, с – 69, м – 11, л – 18, лейк. – 10,2 х 109, СОЭ – 38
мм/час. По алгоритму диагностики проведено лечение антибиотиками широкого
спектра действия. От проведенной терапии эффекта не наступило,
интоксикационный синдром не снят. На контрольной рентгенограмме
отрицательная динамика легочного инфильтрата с обсеменением. Какая тактика
ведения пациента наиболее целесообразна?
a. @+:Анализ мокроты на МБТ и консультация фтизиатра
b. @анализ мокроты на атипические клетки и консультация онколога
c. @анализ мокроты на вторичную флору с определением лекарственной
чувствительности
d. @анализ мокроты на атипические микобактерии
e. @ анализ мокроты на кандиты

78) Девочка 4 месячного возраста осмотрена участковым врачом. Со слов матери


жалобы на повышение температуры тела до 39С, вялость, отказ от груди, рвоту.
Сегодня, на 4-й день болезни присоединилась желтуха, ребенок стал резко
возбужденным, отмечались судороги, рвота типа "кофейной гущи". В анамнезе: 3
месяца назад ребенку проводилось переливание крови (парентеральный путь). В
биохимическом анализе крови: Общий билирубин-165 мкмоль/л (норма до 21),
прямая фракция-144 мкмоль/л (норма до 5), АЛТ-5,8 мкмоль/л, АСТ-3,7 мкмоль/л,
тимоловая проба -12 Ед (норма до 4, для поражений печени при гепатитах
характерна повышенная тимоловая проба).
Какое исследование наиболее правильно уточнит этиологию заболевания?
А)маркерная диагностика на гепатит В и С+++++
В)кал на я\глист
С)люмбальная пункция
Д)кровь на стерильность
Е)кровь на толстую каплю

79) Женщина со сроком гестации 27-28 нед. Жалуется на повышение температуры,


сухой кашель, боль в грудной клетке, одышку. В городе отмечается эпидемический
подъем заболеваемости гриппа и ОРВИ. Температура 39,40С, в зеве слизистая ярко
гиперемирована, задняя стенка глотки зернистая. Инъекция сосудов склер. В
легких жесткое дыхание, хрипов нет. Тоны сердца приглушены, ЧСС 130 уд/мин,
АД 90/60 мм.рт.ст., ЧДД – 32. В OAK: эр.-3,2х1012 /л, гем. -115 г/л, Л. -3,2x109 /л,
э. – 3%, п\я – 15%, с\я – 60%, м – 7%, л – 15%, СОЭ - 15 мм/ч. Какая тактика
наиболее целесообазна?
a. @+:Экстренная госпитализация, основной препарат осельтамавир
b. @лечение амбулаторно, основной препарат азитрамицин
c. @лечение в условиях дневного стационара, детоксикационная терапия
d. @плановая госпитализация в терапевтическое отделение, ацикловир
e. @ экстренная госпитализация, основной препарат цефтриаксон

80) Мальчик 11 лет, катаясь на роликовых коньках во дворе у дома, упал и сильно
поцарапал кожу в области коленных суставов, ладоней обеих кистей. Загрязненные
землей раны, родители промыли проточной водой и отвезли ребенка в травмпункт.
Медотвод от прививок с 2 лет в связи с бронхиальной астмой. Какое решение
должен принять врач:
a. +ввести противостолбнячный анатоксин
b. обработать рану перекисью водорода, наложить бактерицидный
лейкопластырь
c. обработать рану, наложить повязку и освободить от занятий
d. после обработки раны рекомендовать рентгенографию коленного сустава
e. прививки против столбняка воздержаться, наблюдать

81) Мальчик 6 месяцев, жалобы на сонливость, потерю аппетита, отказ от питья,


повышение температуры до 39,8. Из анамнеза: болен в течении 3-х дней.
Объективно: дыхание затрудненное, частота дыхания за 1 минуту 68. Отмечается
втяжение грудной клетки. Правильная тактика врача по программе ИВБДВ
a. @+:антибиотик в/м, срочная госпитализация
b. @антибиотик в\м, сальбутамол
c. @ антибиотик в\м, лечение на дому
d. @ антибиотик в\м, плановая госпитализация
e. @ сальбутамол, антибиотик через рот

82) Ребенку 2 года. Жалобы матери на плохой аппетит, вялость, плаксивость,


негативизм при общении с детьми. Объективно: оценить насколько бледен ребенок
трудно, так как ребенок смуглый. Обращает внимание бледность ладоней. Кожа
суховатая, в углах рта мокнущие трещины, тургор тканей и тонус мышц несколько
снижены. Дыхание жестковатое, ЧД 30 в мин. Тоны сердца приглушены,
систолический шум на верхушке и в V точке. Страдает энурезом. При опросе
выяснилось пристрастие к поеданию угля, сырого фарша, газетной бумаги. ОАК –
эритроциты 4,1х1012/л, гемоглобин 112 г/л, ЦП 0,7. Какое обследование следует
назначить?
a. @уровень сывороточного железа
b. @АЛТ и АСТ
c. @Уровень билирубина
d. @ЭКГ
e. @Щелочная фосфатаза

83) Мужчина 56 лет, предъявляет жалобы на частые головные боли при перемене
погоды, к концу рабочего дня. 7 лет назад обнаружена артериальная гипертензия,
лечился не регулярно. 2 года назад перенес ишемический инсульт, курит 20 сигарет
в день. При осмотре тоны сердца ясные, акцент 2 тона на аорте. АД 165/100
мм.рт.ст. Левая граница сердца смещена влево на 1 см. На ЭхоКГ: индекс массы
миокарда левого желудочка 140 г/м2, УЗДГ сонной артерии: комплекс интима-
медиа-1,1 мм, в области бифуркации сонной артерии -1,5 мм. Лечебная тактика?
1. монотерапиябисопрололом
2. монотерапия лизиноприлом
3. +:фозиноприл и карведилол
4. лизиноприл и валсартан
5. амлодипин и каптоприл

84) Мужчина 62 лет, отмечает повышения АД 175-190/95-110 мм рт.ст. Лечится не


регулярно. Вес-100 кг, рост- 165см., объем талии-105см. Объективно: левая
граница сердца на 1 см влево от левой средне ключичной линии, по 5 межреберью.
На ЭКГ: ритм синусовый, индекс Соколова-Лайона – 40мм. МАУ – 300 мг/сутки.
Комплекс интима-медиа сонной артерии - 1мм. Какая вероятность развития
кардио-васкулярных катастроф в ближайшие 10 лет?
a. @выше 30%
b. @до 10%
c. @10-15%
d. @16-20%
e. @21-30%

85) Женщина 63 лет поступила с жалобами на одышку, лихорадку до 38.0º, слабость,


потливость. Состояние ухудшилось 2 дня назад на фоне ОРВИ. Объективно: кожа
влажная, бледная, барабанные палочки. АД 130/90 мм рт. ст., ЧСС – 100 в мин,
ЧДД – 28 в мин. Выслушиваются влажные мелкопузырчатые хрипы слева в
нижних отделах на фоне ослабленного дыхания. Пульсоксиметрия – 88%. На
рентгенограмме: горизонтальное расположение ребер, расширенные межреберные
промежутки, расширенные корни легких, усиление легочного рисунка за счет
фиброзного и сосудистого компонентов, инфильтративные сливные тени в нижней
доле левого легкого. Какая тактика наиболее целесообразна?
a. @ экстренная госпитализация в ОРИТ
b. @ лечение амбулаторно ципрофлоксацином
c. @ лечение в условиях стационара на дому левофлоксацином
d. @ лечение в условиях дневного стационара цефтриаксоном
e. @ планово госпитализировать в терапевтическое отделение, ровамицин

86) В школе был проведен профилактический осмотр детей младших классов. Среди
обследованных был выявлен ребенок 7 лет, который в течение учебного года 7 раз
переболел ОРВИ и ОРЗ. При осмотре нервно-психическое и физическое развитие
соотвественно возрасту. Осмотр ЛОР органов патологии не выявил, немного
увеличены, безболезненные средне-шейные лимфоузлы. К какой группе здоровья
следует отнести данного ребенка?
a. @2 группа
b. @1 группа
c. @3 группа
d. @4 группа
e. @5 группа

87) Ребенку в возрасте 1 года 4 месяцев поставлена проба Манту – папула 8 мм.
Контакт с больным туберкулезом не установлен. Жалоб нет. Симптомы
интоксикации отсутствуют. Диаскинтест – отрицательный. Какой наиболее
вероятный предварительный диагноз в данном случае?
a. @поствакцинальная аллергия
b. @инфекционная аллергия
c. @аллергическая реакция
d. @специфическая реакция
e. @ токсико-аллергическая реакция
88) Мальчик 5 дней. Из анамнеза: мама страдает атопическим дерматитом. Роды
срочные, естественные. Масса тела 2800 грамм при рождении, длина - 49 см.
Ребенок на естественном вскармливании. Масса тела при осмотре 2 600 грамм.
Объективно: легкая желтушность, состояние удовлетворительное. Сосет активно,
физиологические рефлексы живые. Пупочная ранка сухая. Большой родничок
2,0x2,0 см, не выбухает. Какова тактика врача по проведению профилактических
прививок?
a. @+: в обычные сроки с предварительной подготовкой
b. @относительный мед отвод
c. @абсолютный мед отвод
d. @обычными методами по общепринятому календарю
e. @щадящими методами по индивидуальному календарю

89) Мужчина 42 лет, при прохождении скрининга на выявление глаукомы были


выявлены следующие показатели внутриглазного давления бесконтактным
методом: правый глаз - 25 мм рт.ст, левый глаз - 18 мм рт.ст. Какая дальнейшая
тактика?
@направить на дообследование к офтальмологу
@рекомендовать повторное обследование через 3 месяца
@направить к терапевту
@направить в глаукомный кабинет
@направить на дообследование в условиях стационара

90) При скрининге у мужчины 30 лет были выявлены жалобы на жажду, сухость во
рту. При осмотре: Рост 176 см. Вес 84 кг. ИМТ 27,1. В крови: глюкоза натощак 6,7
ммоль/л. Какое обследование должно быть проведено на следующем этапе?
@+:определение гликемического профиля
@определение лейкоформулы в крови
@определение глюкозы в моче
@определение С пептида натощак
@липидный профиль

91) Врач общей практики обслуживает вызов на дому. Будучи инвалидом II группы по
основному заболеванию суставов, пациент не может быть госпитализирован в
стационар и в то же время нуждается в уходе. На какой срок выдается лист
временной нетрудоспособности его родственнику, приехавшему из другой
местности, для ухода за заболевшим?
a. @До 3-х дней
b. @До 7 дней
c. @До 14 дней ВКК
d. @+:На весь период заболевания
e. @Выдается справка о временной нетрудоспособности

92) Проводится профосмотр в детском саду перед школой. Девочка 6 лет. От 1


доношенной беременности и родов, протекавших без патологии. Растет и
развивается соответственно возрасту. Перенесенные заболевания – перинатальная
энцефалопатия до 1 года, атопический дерматит-после 3-х лет. На момент осмотра
состояние удовлетворительное. Физическое и психомоторное развитие
соответствует возрасту. По внутренним органам без особенностей. Какую
медицинскую документацию необходимо заполнить ребенку?
a. @ +: форма 026/у, 063/у, паспорт здоровья.+++++++жб
b. @ форма 026/у, 0112/у, паспорт здоровья
c. @форма 026/у, паспорт здоровья
d. @форма 063/у, паспорт здоровья
e. @форма 112/у, 063/у, паспорт здоровья

93) Женщина 72 лет, в анамнезе инфаркт миокарда, сахарный диабет. В анализах:


холестерин 5,97 мммоль/л, ЛПНП 4,53 ммоль/л, ЛПВП 0,98 ммоль/л, триглицериды 2,24
ммоль/л., коэффициент атерогенности 5,09. Рассчитанный 10 летний показатель
SCORE≥10%. Врач назначил аторвастатин. До какого целевого уровня необходимо
снижение уровня ЛПНП (УД-1В) согласно протоколу МЗ РК «Артериальная гипертензия»
от 0,.10.2019 г.?
A) < 1,9 ммоль/л
B) < 1,8 ммоль/л+++
C) < 2,0 ммоль/л
D) < 2,5 ммоль/л
E) < 2,8 ммоль/л

94) Мама ребенка 9 лет обратилась к врачу за разъяснениями по поводу медицинского


страхования - надо ли ей платить за него взнос? До какого возраста дети освобождены от
взносов в систему ОСМС?
A) До 10 лет
B) До 12 лет
C) До 14 лет
D) До 16 лет
E) До 18 лет+++

95) Женщина 55 лет обратилась с жалобами на головные боли , которые появились


несколько недель назад , возникают к концу рабочего дня. При измерении АД в кабинете
доврачебного осмотра медсестрой – 145/95 мм.рт.ст. Женщина с ИМТ 30, вредных
привычек нет, наследственность не отягощена. Какое обследование показано на
следующем этапе?
@Электрокардиография
@Р-графия органов грудной клетки
@Эхокардиография
@ Доплер-сонография сонной артерии
@+: Повторное измерение АД не менее чем через 6 часов

96) Ребенок 1 год, привит в родильном доме БЦЖ, рубчика на левом плече нет.
Поставлена проба Манту- папула 13 мм. Определите характер туберкулиновой
чувствительности?
@+:Первичное инфицирование МБТ
@Аллергическая реакция на туберкулин
@Поствакцинальная аллергия
@Заболевание туберкулезом
@Латентный период

97) Мужчина 60 лет, обратился к врачу с жалобами на кашель с выделением слизистой


мокроты, чаще по утрам. Кашель беспокоит в течение 25 лет. Вредные привычки - курит с
25 лет по 1 пачке в день. Ежегодно вакцинируется противогриппозной вакциной,
обострений не отмечает. Постбронходилатационный тест: ОФВ1<30% от
прогнозируемого. Оценка CAT=18 баллов. Пошкале mMRC 4. Ккакойкатегориигрупп
GOLD (2019 Global Initiative for Chronic Obstructive Lung Disease) относитсяпациент?
A) GOLD 1, категория А
B) GOLD 2, категория В
++C) GOLD 3, категория В
D) GOLD 4, категория В
E) GOLD 4, категория D

98) Девушка 25 лет, жалобы на выраженную слабость, одышку при малейшей физической
нагрузке, пастозность лодыжек, сердцебиение. 2 недели назад перенесла ОРВИ.
Объективно: бледность, пастозность лодыжек. При аускультации: в легких – крепитация в
нижних отделах, тоны сердца приглушены, систолический шум на верхушке, ЧСС 110
мин., АД 100/70 мм.рт.ст. ЭКГ(фото). В анализе крови- СОЭ- 25 мм/ч. Какой диагноз?

+@+:острый миокардит , НК2А


@ОРЛ, ревмокардит, НК 1
@дилятационная кардиомиопатия, НК2А
@рестриктивная кардиомиопат
@хронический миокардит, обострение

99) Женщина 32 лет, обратилась с жалобами на лихорадку до 38,5, кашель со скудной


слизисто-гнойной мокротой, небольшую одышку. Больна около 10 дней, заболевание
началось остро с лихорадки, боли в горле и сухого кашля, 3 дня назад состояние
ухудшилось, появилась одышка. Объективно: АД 110/70 мм.рт.ст., ЧСС 95 в мин. ЧДД –
21 в мин, в нижних отделах правого легкого выслушивается крепитация на фоне немного
ослабленного дыхания, в средних и верхних отделах – бронхиальное дыхание.
Пульсоксиметрия-95%. В ОАК-Л 11*109/л, Эр-3,2*09/л, Нв-123 г/л, СОЭ-24 мм рт.ст. Р-
графия легких – инфильтративные тени средней интенсивности в нижней доле правого
легкого. Какая тактика наиболее целесообразна?
@лечение в условиях дневного стационара цефтриаксоном
@лечение амбулаторно пефлоксацином
+@+:амбулаторно амоксициллином
@госпитализировать в терапевтическое отделение, ровамицин
@госпитализировать в ОРИТ, ципрофлоксацин

100) Женщина 69 лет. После посещения поликлиники через несколько часов появилось
недомогание, лихорадка, заложенность носа, слезотечение, кашель, боль во всем теле.
Больная состоит на учете по поводу ХОБЛ. Какой препарат необходимо ей назначить
чтобы предупредить развитие осложнений начинающегося заболевания?
+: @осельтамавир
@моксифлоксацин
@беродуал
@метронидазол
@азитромицин

101) Мальчик 5 лет, жалуется на высыпание на лице и по всему телу, лихорадку до 39,5,
боли и припухлость в локтевом суставе справа. Из анамнеза: со слов мамы вчера
появилось высыпание на лице и к вечеру распространилось по всему телу. Ребенок
посещает детский садик больше 1 месяца, вакцинации после 6 месяца жизни не получал.
Объективно: розово-красная пятнисто-папулезная сыпь по всему телу, артрит правого
локтевого сустава. Заднешейные и затылочные лимфоузлы пальпируются. Анализ крови:
Нв – 121 г/л, лейкоциты – 10*109/л, СОЭ-20 мм/ч, СРБ-8 мг/мл, АСЛ – 1:100 МЕ/л, РФ – 5
МЕ/мл. Какой наиболее вероятный этиологический фактор возникновения заболевания?

+@вирус краснухи
@возбудитель скарлатины
@вирус кори
@вирус Коксаки
@вирус ветряной оспы

102) Ребенку 2 года. Жалобы матери на плохой аппетит, вялость, плаксивость, негативизм
при общении с детьми. Объективно: оценить насколько бледен ребенок трудно, так как
ребенок смуглый. Обращает внимание бледность ладоней. Кожа суховатая, в углах рта
мокнущие трещины, тургор тканей и тонус мышц несколько снижены.слизистые розовые.
Дыхание жестковатое, ЧДД 30 в мин. Тоны сердца приглушены, систолический шум на
верхушке в 5 точке. При опросе выяснилось пристрастие к поеданию угля, сырого фарша,
газетной бумаги. ОАК – эритроциты 4,1х1012/л, НВ 112 г/л. Какая доза элементарного
железа необходима:
@2 мг/кг
@1 мг/кг
@4 мг/кг
+@+:3 мг/кг
@5 мг/кг

103) Женщина 65 лет, обратилась с жалобами на сухость во рту, постоянную жажду и


общую слабость. Им анамнеза 2 года она оперирована по поводу острого
панкреатита. При осмотре живот мягкий, безболезненный. На УЗИ брюшной полости
размеры поджелудочной железы в пределах нормы, без патологических образований.
Лабораторное исследование, позволяющее уточнить развития отдаленного осложнения
острого панкреатита?
+@+:анализ крови на глюкозу
@анализ мочи на диастазу
@анализ крови на диастазу
@анализ кала на скрытую кровь
@анализ крови на свертываемость
104) 9-летний мальчик с жалобами на боли при глотании и жжение во рту в течение
последних 1- дней. За последние 3 недели отмечает утомляемость. У отца –
желчнокаменная болезнь, холецистэктомия. Мальчик на вид слабый и вялый, бледный,
склеры желтоваты, опухший красный язык. Т тела 37,7ᴼС, PS 105/мин, АД 110/68 мм рт
ст., ЧДД 28/мин. Селезенка прощупывается на 2-3 см ниже левого реберного края. ОАК:
Hb 98 г/л, ретикулоцитов – 0,4%.
Какая тактика наиболее целесообразна?
A) +Назначение фолиевой кислоты
B) Назначение безглютеновой диеты
C) Переливание крови
D) Вакцинация против пневмонии
E) Назначение В12 витамина

105) У женщины 23 лет, со сроком гестации 12 недель при скрининговом обследовании


обнаружен HBsAg. Жалоб не предъявляет, самочувствие хорошее. При осмотре: кожные
покровы обычной окраски, живот мягкий, б/б, печень +1,0 см. ниже края реберной дуги,
плотноватой консистенции, край закруглен, б/б. Селезенка не пальпируется. В ОАК: Нв-
100г/л, эритроциты- 3,9 х1012/л, лейкоциты- 9,2 х109/л, тромбоциты-180 х 109/л, СОЭ-
25мм/час. В б/х крови: АЛТ- 30Ед/л, АСТ - 40Ед/л, общий билирубин-18 ммоль/л, общий
белок - 65г/л, глюкоза - 3,2ммоль/л, ЩФ - 240 ед/л. Какое обследование является наиболее
определяющим для назначения противовирусной терапии и с какой целью?
a) ИФА HBeAg - для определения репликации вируса
b) +:PCR DNA HBV – для определения вирусной нагрузки
c) PCR DNA HBV – для подтверждения диагноза гепатита В
d) ИФА anti HBcor IgM – для определения активности гепатита В
e) ИФА anti HDV – для определения коинфекции с гепатитом дельта

106) Женщина 56 лет. Обратилась с жалобами на одышку при небольшой физической


нагрузке, сухой кашель. Одышка носит смешанный характер. Состоит на Д учете с
диагнозом ХОБЛ в течение 5 лет. Курит более 30 лет по 1-1,5 пачки в день. Больная
отмечала появление прожилок крови при приступах надсадного малопродуктивного
кашля. Отмечает что и раньше была одышка, но теперь она сильнее и стало более трудно
вдохнуть, нежели выдохнуть. Р-грамма ОГК не выявила существеннных различий с
ренген-снимками прошлого года. В анализе крови - СОЭ 54 мм/ч. Какой обследование
необходимо провести в первую очередь?
A. КТ органов грудной клетки
B. Спирографию
C. +:Бронхоскопию с биопсией
D. определение онкомаркера SYFRA
E. GenExpert

107) Женщина 22 лет, недавно начала регулярную половую жизнь. Обратилась с


жалобами на светлые скудные выделения и неприятные ощущения во время полового
контакта. Обследование на ЗППП выявило антитела к Chlamydia trachomatis IgM. ПЦР
материала из цервикального канала подтвердил наличие Chlamydia trachomatis. У ее
полового партнера - ИФА на антитела к Chlamydia trachomatis дали отрицательный
результат. Какая тактика наиболее целесообразна?
A. курс азитромицина, лечить только женщину
B. +курс азитромицина, лечить обоих половых партнеров
C. левофлоксацин, лечить обоих половых партнеров
D. курс ровамицина, пролечить женщину, ее партнеру повторить ИФА через 3 месяца
E. курс клиндамицина, ее партнера проверить методом ПЦР

108) Больной 74 лет получал антибактериальную по поводу пневмонии. В динамике


состояние без улучшения, нарастает слабость, похудение, боли в грудной клетке. При
контрольной компьютерной томографии – очаг в размерах несколько увеличился, высокой
плотности с неровными контурами. Какой результат исследования мокроты НАИБОЛЕЕ
вероятен?

+@+:Атипичные клетки
@Грам-отрицательные кислотоустойчивые бациллы
@Нейтрофильный цитоз
@Грибковый мицеллий
@Гантелевидные тельца

109) Супружеская пара усыновила ребенка из дома малютки. Ребенок новорожденный,


родился недоношенным. Супруга работает на ответственной должности, супруг-работник
творческой профессии. Они хотят возложить основную заботу о ребенке на приемного
отца. Необходимо выдать документ о временной нетрудоспособности. Кому и какой
документ может быть выдан?
@только приемной отцу, лист о временной нетрудоспособности
@только приемной матери, лист о временной нетрудоспособности
+@любому из приемных родителей, лист о временной нетрудоспособности
@любому из приемных родителей, справку о временной нетрудоспособности
@можно приемному отцу, справку о временной нетрудоспособности

110) Женщина 65 лет, обратилась с жалобами на сухость во рту, постоянную жажду и


общую слабость. Из анамнеза: 2 года назад она оперирована по поводу острого
панкреатита. При осмотре живот мягкий, безболезненный. На УЗИ брюшной полости
размеры поджелудочной железы в пределах нормы, без патологических образований.
Лабораторное исследование, позволяющее уточнить развития отдаленного осложнения
острого панкреатита?
a) +анализ крови на глюкозу
b) анализ мочи на диастазу
c) анализ крови на диастазу
d) анализ кала на скрытую кровь
e) анализ крови на свертываемость

111) 24-летняя женщина обратилась по поводу усталости. В течение последних 6 месяцев


она обращалась к врачам несколько раз. Во время приема она заигрывает с врачом.
Одевается очень моно и ярко. Когда в кабинет заходит медсестра, женщина начинает
нервничать и прекращает разговаривать. На замечание врача, что она ведет себя
неуместно, пациентка начинает плакать и говорить, что «ее никто не понимает, и ей надо,
чтобы ее только выслушали». Потом она говорит, что будет дальше искать врача, который
ей поможет, но не надеется на успех. Какой диагноз наиболее вероятен?
A) Шизоидное расстройство личности
B) Нарциссическое расстройство личности
C) Зависимое расстройство личности
+D) Истерическое расстройство личности
E) Пограничное расстройство личности

112) 35-летняя женщина пришла к врачу из-за влагалищного кровотечения после полового
акта, которое стало беспокоить в течение последних 2 месяцев. Менархе с 13 лет,
менструации регулярные, 28-дневный цикл. Гинекологическое исследование –
нерегулярное поражение шейки матки. Гистологическая оценка биопсии шейки матки –
подтверждает рак шейки матки in-situ. Из каких клеток вероятнее всего происходит этот
рак?
A) Простой столбчатый эпителий с трубчатыми железами
B) Мерцательный простой столбчатый эпителий
C) Простой кубовидный эпителий
D) Кератинизированный многослойный плоский эпителий
+ E) Некератинизированный многослойный плоский эпителий

113) 52-летний мужчина доставлен в приемный покой с внезапным началом


генерализованной головной болью в течение 2 часов. Появилась тошнота, была рвота.
Симптомы начались внезапно, когда он был дома. 6 дней назад у него была сильная
головная боль, которая прошла самостоятельно. Курит по 2 пачки сигарет ежедневно в
течение 30 лет. Принимает лизиноприл, симвастатин. PS 82/мин, АД 160/100 мм.рт.ст.
зрачки ровные, круглые, реагируют на свет. Черепные нервы II-XII интактны, КТ головы
не показывает никаких отклонений. Какой шаг в управлении заболеванием является
наиболее подходящим?

A) Повторить КТ через 24 часа

+B) Назначить люмбальную пункцию

C) Администрирование кислорода

D) Назначить нитропруссид натрия

E) Наблюдение в динамике.

114) 80-летний мужчина из дома престарелых с жалобами на усиливающийся кашель,


лихорадки и одышки в течение 2 дней. В анамнезе – СД 2 типа, АГ, деменция. Принимает
инсулин, эналаприл, донепезил. Т тела 38, 1ᴼС, PS 112/мин, ЧДД 35/мин., АД 78/60 мм рт
ст. Пульсоксиметрия 77%. На рентгене – инфильтраты верхней и средней доли правого
легкого. Что из перечисленного вероятнее всего будет обнаружено при исследовании
мокроты пациента?

1 Грамположительные диплококки+++

2 Грамположительные кокки

3 Грамотрицательные кокобациллы

4 Грамотрицательные палочки

5 Грамположительные ветвящиеся бактерии

115) 72-летняя женщина обратилась по поводу лихорадки, миалгии, кашля в течение 3


дней. Живет во времянке, у соседей были похожие симптомы. В анамнезе – АГ,
принимает лизиноприл. Температура тела 38,9ᴼС, пульс 105/мин, ЧДД 22/мин, АД 110/60
мм рт ст., SаO2 89%. В крови – лейкоцитов 10 500/мм3, креатинин – 0,9 мг/дл,
прокальцитонин 0,05 мкг/л (норма<0,06). На R легких – двустороннее потемнение в
нижних долях. Начато лечение цефтриаксоном и азитромицином. Через два дня -
температура тела 37,6ᴼС, прокальцитонин 0,04 мкг/л, SаO2 96%. Что является
подходящим шагом в управлении заболеванием?
1 Продолжать цефтриаксон и азитромицин до 7 дней
2 Прекратить цефтриаксон и азитромицин
3 Повторная рентгенография легких
4 Прекратить цефтриаксон и продолжить азитромицин
5 Посев мокроты

116) 33-летний мужчина обратился по поводу постепенно нарастающей боли в ногах в


течение 2-х месяцев. Боли возникали после длительной ходьбы, проходили в покое. На
прошлой неделе боль была постоянная и сопровождалась ощущением жжения. Также
были преходящие болезненные узелки по ходу вен ног в течение 4 месяцев, которые
проходили самопроизвольно. Курит 2 пачки сигарет ежедневно в течение 15 лет. При
осмотре – язвы на дистальной части левого большого, второго и пятого пальцев. Какое
вмешательство наиболее вероятно уменьшит риск ампутации конечности у пациента?
1 Компрессионные чулки
2 Симвастатин терапия
3 Назначение эноксопарина
4 Преднизолоновая терапия
5 Отказ от курения++

117) Мама 6-дневного новорожденного обратилась по поводу пожелтения за один день


его кожи и глаз. Мать сообщает, что кормит сына около 7 раз в день. Отмечает, что вчера
у сына было 2 мокрых подгузника и два испражнения. Родился на 38 нед беременности и
весил 3500 г; сейчас весит 3000г. При осомтре – Т тела 37ᴼС, пульс 180/мин, АД 75/45 мм
рт ст., желтушность склер, распространенная желтуха, сухие слизистые оболочки. В крови
– общий билирубин 9 мг/дл, непрямой 0,7 мг/дл, АСТ 15 Ед/л. Что из нижеследующего
является наиболее подходящим для следующего шага в ведении этого пациента?

1 Введение внутривенно иммуноглобулина


2 Назначение фенобарбитала

3 +:Увеличение частоты грудного вскармливания

4 УЗИ брюшной полости

5 Светолечение

118) У 4-х летней девочки нарастает желтуха, которая началась 8 дней назад. У нее были
подобные эпизоды в прошлом. Ее отец перенес спленэктомию в подростковом периоде.
При осмотре легкая спленомегалия. В крови: гемоглобин 101 г/л, MCHC 41% Hb/клетке,
MCV 81 мкм3, тромбоциты – 250 000/мм3, RDW 16%, ретикулоциты 11%, СОЭ 10 мм/час.
В мазке крови – сфероциты. Какой тест наиболее специфичен для подтверждения
состояния пациентки?

1 Тест Кумбса

2 +:Тест на связывание эозин-5-малеимида

3 Электрофорез гемоглобина

4 Уровень ферритина в сыворотке

5 Тест осмотической хрупкости

119) 38-летний мужчина обратился к врачу из-за болезненной сыпи на левой ноге.
Последние 2 года у него периодически появлялись эпизоды изменения цвета пальцев на
холоде – от белого до синего и красного, которые потом проходили. Курит 2 пачки
сигарет ежедневно в течение 20 лет. АД 115/78 мм рт ст. При осмотре – множественные
темно-фиолетовые узелки на боковой поверхности левой стопы с окружающей эритемой.
Сухие язвы на кончике правого указательного пальца. Какой диагноз наиболее вероятен?

1 Облитерирующий тромбофлебит+++

2 Дислипидемия

3 Артериит Такаясу

4 Полиневропатия

5 Тромбоз глубокиз вен нижних конечностей

120) Мужчина, 23года, болеет три недели: повысилась температура, появилась сильная
головная боль, многократная рвота, вялость, двоение в глазах. Из анамнеза выяснено, что
год назад лечился по поводу туберкулеза легких. Объективно: t тела 37,8°С. Пациент
вялый, сонлив, иногда дезориентирован во времени и пространстве, лежит на боку с
приведенными к животу ногами, положительные симптомы Кернига, верхний
Брудзинский. Анизокория, асимметрия лица, сглаженность правой носогубной складки,
расходящееся косоглазие, двоение при взгляде вниз. Какой НАИБОЛЕЕ вероятный
диагноз?

1 туберкулезный спондилит
2 туберкулезный менингит

3 эпидемический энцефалит

4 туберкулезный менингомиелит

5 туберкулезный менингоэнцефалит

121) 32-летняя женщина обратилась к врачу из-за усталости, болезненности груди,


увеличение частоты мочеиспускания и периодическую тошноту в течение 2-х недель.
Последний менструальный цикл был 7 недель назад. В анамнезе эпилепсия, которую
лечили карбамазепином. При осмотре – патологии не выявлено. Тест на беременность
положительный. Наибольший риск какого осложнения наиболее вероятен у будущего
ребенка?

1 почечная дисплазия

2+: дефект нервной трубки (менингоцеле)

3 изменение цвета зубов

4 нейросенсорная тугоухость

5 порок развития конечностей

122) Через три недели после родов 28-летняя женщина обратилась к врачу для осмотра. В
анамнезе - через 2 дня после родов ей был выставлен диагноз «послеродовой эндометрит»,
по поводу чего она получала лечение – в/в антибиотики в течение 2-х дней. У нее начался
болезненный отек груди при кормлении грудью, симптомы уменьшились при учащении
кормлений и применении теплых компрессов. При физикальном осмотре – отклонений
нет. Пациентка интересуется надежным методом контрацепции. Что следует
рекомендовать пациентке для контрацепции?

1 внутриматочная спираль

2 спермицид

3 эстрогенный пластырь

4 метод базальной температуры тела

5 противозачаточные таблетки, содержащие только прогестин+++

123) 42-летняя женщина на приеме. 2 месяца назад ей поставили диагноз «астма».


Принимает сальбутамол и беклометазон, но кашель беспокоит по ночам, когда она лежит в
постели. Последние 2 недели ее также беспокоят боли в груди. Рост 158 см, вес 75 кг, ИМТ
30 кг/м2. У нее осиплость голоса и она часто прочищает горло во время обследования.
Легкие чистые. ОВФ1 78% от должных. Что является следующим шагом в управлении
заболеванием?
1 Назначить эхокардиограмму
2 Добавить ингалятор сальметерола
3 Добавить преднизолон
4 Проверить IgE
5 +:Добавить ингибитор протонной помпы

124) Мужчина 62 лет, в анамнезе инсульт, сахарный диабет. 10-летний риск по шкале
SCORE ≥10%. Принимает антигипертензивную терапию. Участвует в программе
управления заболеваниями. Какая периодичность осмотров врачом ПМСП рекомендована
этому пациенту согласно «Правил оказания первичной медико-санитарной помощи и
Правил прикрепления к организациям первичной медико-санитарной помощи» (приказ №
281 от 28 апреля 2015 г)?
1 1 раз в 3 месяца+++
2 1 раз в 1 месяца
3 1 раз в 2 месяца
4 1 раз в 6 месяцев
5 1 раз в 12 месяцев

125) 9-летнюю девочку в течение 4-х месяцев беспокоят ранние утренние головные боли и
усталость. Головные боли часто сопровождаются тошнотой, иногда бывает рвота, которая
приносит облегчение от головной боли. Перестала ходить на занятия балетом 2 недели
назад, в связи с тем, что она часто падала. Температура тела 36,8ᴼС, пульс 98/мин, АД
105/65 мм рт ст. У нее широкая походка. Стоя обеими ногами вместе и поднимая руки,
закрытие глаз не влияет на равновесие. Не может выполнять быстрые, чередующиеся
движения руками. На МРТ - большие (3,5 см), хорошо определены, контрастные
кистозные массы в боковой задней ямке без участия желудочков. Какой диагноз наиболее
вероятен?
1 Медуллобластома
2 Пилоцетарная астроцитома++
3 Диффузная глиома ствола мозга
4 Эпендиома
5 Олигодендроглиома

126) 42-летняя женщина обратилась в приемное отделение в связи с болью в правой


верхней части живота и тошноты, беспокоящих ее в течение 2 дней. Рост 163 см, вес 91 кг,
ИМТ 34кг/м2. Т 38,5°С. При осмотре – живот вздут, болезненность в правом верхнем
квадранте с нормальными звуками кишечника. В анализах – лейкоцитов 14 000/мм3,
общий билирубин 1,1 мг/дл, АСТ 32 Ед/л, щелочная фосфотаза 68 Ед/л. Что является
наиболее вероятной причиной данного состояния?

1+:Обструкция пузырного протока

2Гепатотропный вирус

3Фиброз общего желчного протока

4Гепатома желчного пузыря

5Фистула между желчным протоком и тонкой кишкой

127) 59-летняя женщина обратилась по поводу приступообразного кашля и одышки в


течение 1 месяца. Кашель непродуктивен, усиливается при подъеме по лестнице и ночью.
8 недель назад у нее была температура, боль в горле и заложенность носа. В течение 10
лет АГ. Выкуривает по полпачке сигарет в день ежедневно в течение 16 лет. PS 78/мин,
ЧДД 18/мин, АД 145/95 мм рт ст. Пульсоксиметрия 96%. Аускультативно – диффузные
хрипы в конце выдоха. Спирометрия: FVC 65% и FEV 60%. Какой диагноз наиболее
вероятен?

1Бронхиальная астма++

2Внеболничная пневмония

3Хроническая сердечная недостаточность

4Дефицит α1-антитрипсина

5ГЭРБ

128) 55-летняя женщина с жалобами на сильные влагалищные кровотечения в течение


трех дней, использует до пяти прокладок в день. Менопауза наступила 1 год назад.
Менархе в 10 лет. В анамнезе – сахарный диабет и гипотиреоз, принимает метформин и
левотироксин. Курила по 1 пачке сигарет в течение 20 лет, бросила курить 5 лет назад.
Рост 165 см, вес 86 кг, ИМТ 32. При осмотре – нормальная шейка матки и легкая атрофия
влагалища. Матка и придатки не пальпируются. Трансвагинальное УЗИ – толщина
эндометрия 6 см. Биопсия эндометрия – атипичных клеток нет. Какая тактика наиболее
целесообразна?
1 тотальная гистерэктомия
2 эстроген вагинальный крем
3 наблюдение
4 анастрозол терапия
5 прогестиновая терапия+

129) 51-летний мужчина на приеме, 9 месяцев назад перенес острый вирусный гепатит В.
При осмотре – патологии не выявлено. Исследование сыворотки крови – повышенная
печеночная трансаминазная активность, вирусная нагрузка гепатита В 4286 МЕ/мл. Какие
результаты ожидаемы у пациента?
1 HBeAg -, антигемоглобин +, Anti-HBc IgG +, Anti-HBc IgМ-
2 HBeAg -, антигемоглобин -, Anti-HBc IgG +, Anti-HBc IgМ-
3 HBeAg -, антигемоглобин +, Anti-HBc IgG -, Anti-HBc IgМ-
4 HBeAg -, антигемоглобин -, Anti-HBc IgG -, Anti-HBc IgМ+
5 HBeAg +, антигемоглобин -, Anti-HBc IgG -, Anti-HBc IgМ+ ++++

130) Мальчик 11 лет, жалобы на приступы затрудненного дыхания менее 2 раз в неделю,
после контакта с кошкой, длятся по 15-20 минут. Какой из перечисленных препаратов
наиболее целесообразно назначить в качестве симптоматической терапии «по
потребности» для купирования приступов удушья?
A) Ipratropiibromidum
B) Budesonidum
С) Fluticasonum
D) Betamethasonum
E) +:Salbutamolum

131)Женщина 32 лет., беременность 35-36 недель. Стоит на Д учете с диагнозом


"Хронический гломерулонефрит". В последнюю неделю появились отеки на лице,
лодыжках, АД 160/100 мм рт ст. на правой руке, 150/95 мм рт.ст на левой руке.
Лабораторно: общий белок 55 г/л, альбумин - 28 г/л креатинин 150 мкмоль/л, мочевина 10
ммоль/л, холестерин 12 ммоль/л. СКФ=53 мл/мин. В ОАМ белок 4 г/л, Лейкоциты 5-6 в
п/зр, Эритроциты - 25-30 в п/зр. Какое из ниже перечисленных состояний является
НАИБОЛЕЕ вероятным?
a) Преэклампсия
b) Прогрессирование нефрита+++
c) Гипертоническая нефропатия
d) Характерное течение нефрита в 3- триместре
e) Острая почечная недостаточность

131) Подросток 15 лет, обратился по поводу фурункулеза. Из анамнеза: предрасположен к


простудным инфекциям, старший брат болеет сахарным диабетом. Объективно:
физическое развитие на 7 лет. Вес 40 кг при росте 150 см. Кожные покровы бледные,
сухие.Множественные фурункулы на коже туловища и шее. АД 105/65 мм.рт.ст. Пульс 88
в мин. Какое обследование в первую очередь является НАИБОЛЕЕ информативным и
достоверно позволит подтвердить основной диагноз?
a) определение лейкоформулы в крови
b) определение глюкозы в моче
c) +:определение гликемического профиля
d) определение С пептида натощак
e) бактериологическое исследование

132) 43-летний мужчина, ВИЧ-инфицированный обратился в связи с лихорадкой и


ночными потами в течение 15 дней, головными болями, слабостью. Получает
противовирусную терапию. Состоит на учете в наркодиспансере, курит, пьет алкоголь.
Узелки на подушечках пальцах, кровизлияния под ногтями, кровоизлияния в конъюктиву
нижнего века. Пансистолический шум вдоль левого края грудины, усиливается на вдохе.
АД 130/80 мм рт ст. ОАК: лейкоциты 12.8х109/л, СОЭ 52 мм/ч.
Что является наиболее вероятным осложнением состояния?
A) Кровоизлияния под ногтями
B) Узелки на пальцах
C) Ретинальные кровоизлияния
D) Гематурия
E) +Легочная эмболия

133) В лабораторию отправлен бакпосев крови перед началом антибактериальной


терапии. На ЭХОКГ – на трехстворчатом клапане обширные вегетации, умеренная
трикуспидальная регургитация. Фракция выброса левого желудочка 76%.
Какой микроорганизм вероятнее всего является причиной заболевания?
A) Streptococcus sanguinis+++
B) Enterococcus faecalis
C) Staphylococcus aureus
D) Neisseria gonorrhoea
E) Staphylococcusepidermidis

134) 45-летняя женщина с жалобами на генерализованное покраснение кожи и зуд в


течение последних 2-х недель. Симптомы появляются каждый вечер перед сном и длятся
около 30 минут. ИМТ=29 кг/м2. В анамнезе АГ и гиперхолестеринемия. Принимает
лизиноприл, никотиновую кислоту, димедрол, оральные контрацептивы. ХН-247 мг/дл,
ЛПВП=39 мг/дл, ЛПНП=172 мг/дл, триглицериды=152 мг/дл.
Что является наиболее целесообразным в ведении пациентки?
A) +администрирование никотиновой кислоты++++
B) администрирование лизиноприла
C) администрирование димедрола
D) снижение ИМТ
E) назначение преднизолона

135) 68-летний мужчина с жалобами на тошноту, дискомфорт в верхней части живота


последние 4 месяца, возникающие после приема обильной пищи. Последние 3 недели эти
симптомы стали появляться при подъеме на 3-й этаж. В анамнезе – АГ, СД 2 типа,
облитерирующий эндартереиит. ИМТ=45 кг/м2, PS 78 уд/мин, АД 148/86 мм рт ст. ЭКГ –
без патологии.
Какая диагностическая тактика наиболее целесообразна?
A) +Велэргометрия++++
B) КТ брюшной полости
C) ФГДС
D) УЗИ брюшной полости
E) Холангипанкреатография

136) Мужчина 42 лет, обратился с жалобами на приступы затруднения дыхания в течение


последней недели 3 раза в день, ночных приступов было 2, потребность в сальбутамоле -
3-4 раза за сутки, кашель со скудной труднотделяемой мокротой. При аускультации
выслушиваются сухие свистящие хрипы в средних и нижних отделах легких.
Пикфлоуметрия - 350-360-380-400-375-320 л/мин. Получает пульмикорт 500 мкг 2 раза в
день. Какая тактика наиболее целесообразна?
a) добавить к лечению преднизолон перорально
b) +сменить пульмикорт на серетид?
c) госпитализировать в аллергологическое отделение
d) увеличить дозировку пульмикорта
e) преднизолон 60 мг и эуфиллин в\в в течение 5-6 дней

Условие задачи одно и тоже, но варианты ответов разные(((


@ направить на консультацию к аллергологу
@ направить на аллергопробу
@ направить на консультацию к пульмонологу (точно!)
@ направить на рентгенологическое исследование
@ госпитализировать в пульмонологическое отделение

137) На прием к врачу обратился мужчина 48 лет,.После стресса, 4 часа назад резко
усилилась головная боль, появился шум в ушах. При осмотре: лицо гиперемировано,
очаговой неврологической симптоматики нет. В легких везикулярное дыхание. АД
160/100 мм.рт. ст., тоны сердца ритмичные. ЧСС-70 в мин. Акцент 2 тона на аорте.
Назначение какого лекарственного препарата НАИБОЛЕЕ целесообразно для
неотложной помощи?
a) Валсартал
b) Небетолол
c) +Каптоприл
d) Бисопролол
e) Нитроглицерин

138) Мужчина 46 лет. Жалобы на жжение за грудиной, дискомфорт в эпигастрии и за


грудиной при глотании. При отходе к сну иногда появляется приступообразный кашель,
затруднение дыхания, чувство переполнения в эпигастрии. Объективно: не выявлено
никаких отклонений от нормы. Какое обследование целесообразно провести?
a) Спирометрию
b) Пикфлоуметрию с бронхолитиком
c) Бронхоскопию
d) +Эзофагогастроскопию
e) Электрокардиографию (+каз)

139) Сколько времени составляет обработка вызова с момента его получения диспетчером
ССМП до передачи для обслуживания бригаде СМП составляет, в течение которого
проводится сортировка по категории срочности вызова (Согласно Приказу №450 МЗ РК
от 03.07.2017г. «Об утверждении Правил оказания скорой медицинской помощи в
Республике Казахстан»)
A) 1 мин
В) 2 мин
C) 3 мин
D) 4 мин
E) 5 мин+++

140) Сколько времени составляет прибытие фельдшерских и специализированных


(врачебных) бригад до места нахождения пациента с момента получения вызова от
диспетчера ССМП при 1 категории срочности (Согласно Приказу №450 МЗ РК от
03.07.2017г. «Об утверждении Правил оказания скорой медицинской помощи в
Республике Казахстан»)?
A) до 10 минут+++
В) до 15 минут
C) до 30 минут
D) до 45 минут
E) до 60 минут

141) Сколько времени составляет прибытие фельдшерских и специализированных


(врачебных) бригад до места нахождения пациента с момента получения вызова от
диспетчера ССМП при 2 категории срочности (Согласно Приказу №450 МЗ РК от
03.07.2017г. «Об утверждении Правил оказания скорой медицинской помощи в
Республике Казахстан»)?
A) до 10 минут
В) до 15 минут+++
C) до 30 минут
D) до 45 минут
E) до 60 минут

142) Сколько времени составляет прибытие фельдшерских и специализированных


(врачебных) бригад до места нахождения пациента с момента получения вызова от
диспетчера ССМП при 3 категории срочности (Согласно Приказу №450 МЗ РК от
03.07.2017г. «Об утверждении Правил оказания скорой медицинской помощи в
Республике Казахстан»)?
A) до 10 минут
В) до 15 минут
C) до 30 минут+++
D) до 45 минут
E) до 60 минут
143) Сколько времени составляет прибытие фельдшерских и специализированных
(врачебных) бригад до места нахождения пациента с момента получения вызова от
диспетчера ССМП при 4 категории срочности (Согласно Приказу №450 МЗ РК от
03.07.2017г. «Об утверждении Правил оказания скорой медицинской помощи в
Республике Казахстан»)?
A) до 10 минут
В) до 15 минут
C) до 30 минут
D) до 45 минут
E) до 60 минут+++

144) При плановом диспансерном осмотре 38 летней женщины врач обратил внимание на
учащенный (120 в мин) и аритмичный пульс, повышение АД до 160/80 мм.рт.ст. Сама
пациентка чувствует себя хорошо, жалоб не предьявляет. При активном опросе выяснили,
что больная похудела на 6 кг за последние 4 мес.
Какое исследование для уточнения диагноза необходимо провести?
А) УЗИ щитовидной железы
В) определение экскреции метанефинов с суточной мочой
С) ЭКГ
D) +определение тиреотропного гормона (ТТГ) в крови+++
Е) КТГ надпочечников

145) Какая группа препаратов (класс I, уровень А) из нижеперечисленных рекомендована


для медикаментозной терапии всем пациентам со стабильной стенокардией для
профилактики «событий»?
А) +Статины
В) β-блокаторы
С) Блокаторы кальциевых каналов
D) Прологированные нитраты
Е) Короткодействующие нитраты

146) Какая группа препаратов (класс IIb, уровень В) из нижеперечисленных


рекомендована в качестве препаратов второй линии для медикаментозной терапии всем
пациентам со стабильной стенокардией для облегчения симптомов стенокардии/ишемии?
А) +Триметазидин
В) β-блокаторы
С) Блокаторы кальциевых каналов
D) Пролонгированные нитраты
Е) Короткодействующие нитраты

147) Мужчина, 48 лет, внезапно проснулся от резких болей за грудиной, иррадиирущих в


левую лопатку, 2 раза принял изокет, боли не прошли. Стала нарастать выраженная
слабость. Жена вызвала скорую помощь.
В течение какого времени ожидается прибытие бригады скорой помощи до пациента с
момента получения вызова от диспетчера ССМП (Согласно Приказу №450 МЗ РК от
03.07.2017г. "Об утверждении Правил оказания скорой медицинской помощи в
Республике Казахстан")?
A) +до 10 минут
В) до 15 минут
C) до 30 минут
D) до 45 минут
E) до 60 минут
148)45-летний мужчина, в анамнезе гиперхолестеринемия, которая хорошо
контролируется аторвастатином. Бегает по утрам по 2 км. Его отец умер от инфаркта в 43
года. Пациент не курит, алкоголь не употребляет. Лабораторные исследования в пределах
нормы.???На ЭКГ. Какой диагноз наиболее вероятен?
A) +AV-блок Мобитц тип I????????
B) AV-блок Мобитц тип II
C) Инфаркт миокарда
D) Мерцательная аритмия
E) Трепетание предсердий

149) Женщина 30 лет. Беременность 19-20 недель. Состоит на Д учете по поводу


Бронхиальной астмы, гормонозависимая. В течение последней недели появилась
лихорадка до 38,4°С, одышка, сухие свистящие хрипы, потребность в сальбутамоле
выросла до 5-6 раз в день. Была госпитализирована, был взят материал из носоглотки на
ПЦР - на 6 день был получен результат - вирус H3N1. На 6 день госпитализации
состояние ухудшилось, при обследовании зафиксирована замершая беременность.
Развился сепсис с картиной септического шока. По жизненным показаниям была сделана
гистерэктомия, проведена массивная антибактериальная терапия. Какое мероприятие,
могло предотвратить развитие данного патологического состояния?
a) +: вакцинация от гриппа субъединичной вакциной в первом триместре
b) вакцинация от гриппа сплит -вакциной перед началом эпидемического периода
c) вакцинация от гриппа живой вакциной в первом триместр
d) профилактическое назначение осельтамавира
e) профилактическое назначение занамивира

150) Женщина 26 лет, беременность 27-28 недели. При осмотре умеренно выраженные
отеки, пациентке пришлось снять кольца, поскольку пальцы стали толще. АД 150/80 мм
ртст.на левой руке, 135/ 75 мм рт.ст. на правой руке. В ОАМ белок 0,33 г/л, лейкоциты до
10 в п/зр, эритроциты 2-3 в п/зр. Какое из перечисленных состояний НАИБОЛЕЕ вероятно
развилось у данной женщины?
a) +:Преэклампсия
b) Острый гломерулонефрит
c) Хронический гломерулонефрит, обострение
d) Пиелонефрит беременных
e) Вегетососудистая дистония

151) У беременной 27 лет на сроке 36 недель отмечается повышение АД до 170/100 мм


рт.ст., увеличение массы тела на 4 кг за неделю, отеки на нижних конечностях, снижение
суточного диуреза до 400 мл. В крови: тромбоциты – 90 тыс, альбумины – 16 г/л. В моче:
белок – 2,9 г/л. На УЗИ: задержка развития плода на 4 недели. Какая тактика ведения
беременной НАИБОЛЕЕ целесообразна?
A. Пролонгирование беременности до самопроизвольного родоразрешения
B. Пролонгирование беременности с антигипертензивной терапией
C. Прерывание беременности с антигипертензивной терапией
D. Антигипертензивная терапия
E. +:Досрочное родоразрешение

152) 25-летний мужчина с жалобами на жидкий стул, вздутие живота, тошноту, рвоту в
течение последних 3-х дней. Стул по описанию пациента мягкий, пенистый, жирный. На
прошлой неделе пациент ходил в поход и пил воду из родника. Три месяца назад провел
отпуск в Бразилии, где также был жидкий стул, ел много традиционных блюд. Пациент
выглядит обезвоженным, «мраморная бледность». Т тела 37ᴼС, PS 80 в мин, АД 100/60 мм
ртст. Сухость слизистых, болезненность в пилородуоденальной зоне.
Что из нижеследующего является целесообразным в управлении над заболеванием?
A) Назначение ципрофлоксацина
B) +Назначение метронидазола
C) Назначение ванкомицина
D) Назначение сульфасалазина
E) Назначение флуконазола

153) 55-летнюю женщину привела на прием дочь. Со слов дочери у матери -


прогрессирующая потеря памяти, слабость в течение последних 6 месяцев, появился
жидкий стул Мать не может заниматься повседневными делами, несколько раз падала.
Курит 10 сигарет в день в течение 25 лет, бросила курить 8 лет назад. Выпивает около 300
граммов водки ежедневно. При обследовании – глоссит, гиперпигментированная сыпь на
лице и руках, синяки на руках. Кратковременная память нарушена, может вспомнить 0 из
5 объектов через 10 минут.
Недостаток какого витамина вызвал данное состояние?
А) +Недостаток витамина В3
В) Недостаток витамина В5
С) Недостаток витамина В7
D) Недостаток витамина В1
Е) Недостаток витамина В6

154) 49-летняя женщина обратилась по поводу трудностей в ходьбе и головокружения в


течение последних 2 недель. Также отмечает усталость, изжогу, диарею, потерю 2.2 кг
веса в течение 4-х месяцев. Стул дурно пахнет и плохо смывается. Принимает
безрецептурный антацид. Со стороны матери – болезнь Крона, аутоимунное заболевание
щитовидной железы. Рост 150 см, вес 43 кг, ИМТ=19, 1 кг/м2. Живот мягкий, при
пальпации – болезненность в эпигастральной области. Hb 110 г/л, уровень витамина Е и D
снижен. ФГДС – язвы в антральном отделе желудка и 12-ПК.
Что послужило причиной данного состояния?
А) Аутоантитела против слизистой оболочки кишечника
В) Воспалительная реакция на глютен
С) Нарушение транспорта аминокислот
D) Бактериальное разрастание тонкой кишки
Е) +Инактивация панкреатических ферментов

155) 17-летняя девочка обратилась по поводу болей в животе и зудящих высыпаний на


коже плеч в течение последних 6 месяцев. Чувство раздутости живота после еды.
Последние 3 месяца ощущает в течение дня движения кишечника. Выглядит худой. Т тела
37ᴼС, PS 90/мин, ЧДД 16/мин, АД 120/78 мм рт ст. Бледность конъюнктивы, воспаление в
уголках рта. Живот мягкий, диффузная болезненность.
Какой результат ожидаем при дальнейшей оценке пациента?
A) увеличение сывороточной липазы
B) PAS-позитивные макрофаги
C) снижение уровня альбумина
D) +антитела к тканевой трансглутаминазе (AGA)
E) снижение гемоглобина

156) 16-летний мальчик обратился по поводу жидкого, иногда кровянистого стула 2-3 раза
в течение дня, в течение 3-х недель, утомляемость, снижение веса на 2 кг. В это же время
появляются спастические боли в животе. Т тела 37, 1ᴼС, PS 82/мин, АД 106/68 мм рт ст.
Живот мягкий, болезненность в нижней эпигастральной области при пальпации. ОАК - Hb
110 г/л, лейкоциты – 12.4х109/л, СОЭ 14 мм/час. Колоноскопия – воспаленная рыхлая
слизистая, кровоточит при контакте с эндоскопом.
Какая терапия наиболее целесообразна для пациента?
A) Беглютеновая диета
B) Проктоколэктомия
C) +Назначение 5-АСА (5-аминосалициловая кислота)
D) Назначение преднизолона
E) Назначение циклостатиков

157) Студент одного из вузов, находясь на производственной практике обратился с


жалобами на тошноту, рвоту, недомогание. Был выставлен диагноз "Острая пищевая
токсикоинфекция". Какой документ о временной нетрудоспособности ему следует
выдать?
@лист о временной нетрудоспособности
@невостребованный бланк листа о временной нетрудоспособности
@заключение МСЭК
@заключение врачебно-консультативной комиссии
@+:справка о временной нетрудоспособности

158) В поликлинику обратился ребенок 6 лет из социально обеспеченной семьи для


оформления в школу. Ранее ребенок дошкольные учреждения не посещал. В 3 летнем
возрасте перенес острую пневмонию без осложнений, в 5 лет – острую кишечную
инфекцию. При осмотре нервно-психическое и физическое развитие соответствует
возрасту. Привит согласно календарному плану. Какая первичная медицинская
документация сдается в школу?
@026/у
@030/у
@076/у
@080/у
@+:063/у

159) Мужчина 45 лет, обратился к участковому врачу с жалобами на повышение АД до


145/95 мм рт.ст. Повышенное АД было выявлено при профосмотре однократно. Не курит,
вредных привычек нет, наследственность не отягощена. Объективно: ИМТ - 32, АД
135/65. Глюкоза - 4.8 ммоль/л, холестерин - 4,8 ммоль/л. Какая рекомендация должна быть
дана пациенту?
@+:пациент должен быть направлен в школу здоровья по АГ
@дать рекомендации по ЗОЖ и рекомендовать осмотр через 2 года
@дать рекомендации соблюдать ЗОЖ и повторно обследоваться через 6 месяцев
@направить на консультацию к эндокринологу
@ направить на консультацию к кардиологу

160) Женщина 45 лет. При прохождении скрининга было выявлено: ИМТ 26, АД 120/80
мм рт.ст., ЧСС - 75 в мин, уровень глюкозы - 5.4 ммоль/л, холестерин - 4.7 ммоль/л. ЭКГ -
нормальное положение ЭОС, ритм синусовый, неполная блокада правой ножки пучка
Гиса. Гинекологический осмотр патологии не выявил. РАР-тест - "цитограмма без
особенностей". Маммография - без особенностей. К какой группе диспансерного
наблюдения ее следует отнести?
@группа 1 Б
@+:группа 1
@группа 1 А
@группа 2
@группа 3
161) Женщину 47 лет в течение 2 недель беспокоит кашель со скудной слизисто-гнойной
мокротой и приступообразный сухой кашель по ночам, повышение температуры тела до
38°С. При осмотре: ЧДД- 22 в мин, в легких дыхание жесткое, рассеянные гудящие
хрипы. Рентгенологически определяется усиление легочного рисунка. Какая тактика
наиболее целесообразна?
@+: лечение амбулаторно рокситромицином
@ лечение амбулаторно пефлоксацином
@ лечение в условиях дневного стационара цефтриаксоном
@ госпитализировать в терапевтическое отделение, ровамицин
@ госпитализировать в терапевтического отделение, левофлоксацин

162) Девушка 25 лет, жалобы на выраженную слабость, одышку при малейшей


физической нагрузке, пастозность лодыжек, сердцебиение. 2 недели назад перенесла
ОРВИ. Объективно: бледность, пастозность лодыжек. При аускультации: в легких –
крепитация в нижних отделах, тоны сердца приглушены, систолический шум на
верхушке, ЧСС 110 мин, АД 100/70 мм рт.ст. ЭКГ (фото). В анализе крови – СОЭ – 25
мм/ч. Какой диагноз?

@ острый микардит, НК2А


@орл, ревмокардит, нк 1
@дилятационная кардиомиопатия, нк2а
@рестриктивная кардиомиопат
@хронический миокардит, обострение

163) К вам за советом обратилась женщина 25 лет, которая родила ребенка 3 месяца назад,
по поводу вакцинации от гриппа. Живет с мужем в благоустроенной квартире, семья
обеспечена. Какие рекомендации по вакцинации вы можете ей дать?
@достаточно вакцинироваться мужу, матери и ребенку не надо- их социальный контакт
ограничен
@вакцинировать ребенка нельзя до 6 месяцев, надо вакцинировать только мать
@ей вакцинироваться не надо, надо вакцинировать ребенка
@вакцинировать ребенка нельзя до 6 месяцев, надо вакцинироваться всем членам
семьи
@ семья не относится к группе риска – вакцинация не надо

164) Врач общей практики проводит патронаж новорожденного на дому. Врач положил
младенца на живот, при этом происходит рефлекторный поворот головы в сторону. Этот
рефлекс выражен с первых часов жизни. Как называется этот рефлекс?
@Защитный рефлекс
@рефлекс ползания
@защитный рефлекс нужно выбирать
@хоботковый рефлекс
@ хватательный рефлекс
165) Мужчина 30 лет обратился с жалобами на боль и припухлость коленных суставов,
мигрирующие боли в крупных суставах. Считает себя больным около 2 месяцев. В
анамнезе – «синдром Жильбера», «геморрагический васкулит», дисциркуляторная
энцефалопатия. На Р-графия суставов патологии не выявило. ОАК: лейкопения,
лимфоцитоз, СОЭ 12 мм/ч. Биохимия: глюкоза – 5,4 ммоль/л, АЛТ – 62 МЕ/мл, АСТ 42
МЕ/мл, билирубин 24 мкмоль/л. РФ положительный, АНА - отрицательные. Какой анализ
из нижеперечисленных следует провести пациенту в обязательном порядке?
@ИФА на маркеры гепатитов С и В
@ИФА на IgM антитела к парвовирусу В19
@антитела к двуспиральной ДНК
@антицитруллиновые антитела
@ +:антистрептолизин О

166) Новорожденному в роддоме был дан медицинский отвод на проведение вакцинации


БЦЖ на 2 месяца невропатологом по поводу родовой травмы головного мозга. Через 2
месяца после осмотра невропатолога и педиатра было решено провести вакцинацию .
Была назначена реакция Манту с 2 ТЕ. Какова цель применения пробы Манту?
@+:Для определения инфицированности организма туберкулезом
@для создания противотуберкулезного иммунитета (защита организма от туберкулеза)
@для диагностики туберкулеза
@для наблюдения за реакцией на БЦЖ
@ для диспансерного наблюдения

167) Больной инфильтративным туберкулезом легких в фазе распада, МБТ+. Получил


интенсивную и поддерживающую фазы лечения. Наступило закрытие полости распада и
прекращение бактериовыделения. Исход - вылечен. Какой наиболее вероятный срок
снятия пациента с эпидемического учета?
@+:через 1 год после прекращения бактериовыделения
@через 2 года после прекращения бактериовыделения
@через 6 месяцев после прекращения бактериовыделения
@сразу после прекращения бактериаовыделния
@ после трехкратного отсутствия МБТ

168) Мужчина 72 лет. Беспокоят упорные по нескольку дней запоры. Похудел за


последние 3 месяца на 5 кг. В крови - панцитопения и СОЭ 65 мм рт.ст. Какое
обследование необходимо провести для уточнения диагноза?
@+:колоноскопию с прицельной биопсией
@пальпаторное исследование прямой кишки
@альфа-фетопротеин
@КТ органов брюшной полости
@ ирригоскопия

169) с документа Тесты1

Как часто проводится противорецидивное лечение детям с хронической пневмонией и


деформацией бронхов?

A) 1 раз в год

B) 2 раза в год

C) 3 раза в год
D) 4 раза в год ++

E) 5 раз в год

170) 62-летний мужчина на Д учете. В анамнезе АГ, контролируемая эналаприлом. В


течение прошлого года усилились судороги в ногах при ходьбе, в покое - проходят. Со
времени последнего визита у врача занимался на беговой дорожке 4 раза в неделю. Курил
2 пачки сигарет в день в течение 35 лет, бросил курить 5 месяцев назад. Т тела 37.0ᴼС, PS
84 уд/мин, АД 132/78 мм рт ст. Кожа ног бледная, холодная на ощупь.
Что из нижеследующего является целесообразным для контроля над заболеванием?
A) Оперативная реконструкция сосудов
B) Ангиопластика и стентирование
C) Назначение ванкомицина
D) Ношение компрессионных чулков
E) +Назначение клопидогреля и симвастатина
171) 19-месячная девочка на профосмотре. Родилась в срок, здорова. При осмотре ребенок
активен. Находится на уровне 75-го процентиля по росту и 80-го процентиля по весу. PS
102 уд/мин, АД 102/54 мм рт, ЧДД 24/мин. На контрастной ЭХОКГ – мягкий
межпредсердный шунт, видный при покашливании ребенка.
Что из нижеследующего является целесообразным в управлении над заболеванием?
A) +Без вмешательства
B) Терапия варфарином
C) Терапия аспирином
D) Чрескожное закрытие
E) Регулярное ЭХОКГ

172) Новорожденная девочка, вес 3500г, рост 50 см. Мать ребенка является носителем
HBsAg.
Когда необходимо провести первую вакцинацию против гепатита В ребенку?
А) +новорожденного вакцинируют сразу после рождения
В) новорожденного вакцинируют в 6 месяцев
С) новорожденного вакцинируют в 1 месяц
D) новорожденного вакцинируют в 2 месяца
Е) новорожденного вакцинируют в 12 месяцев

173) Мальчик 13 лет, родители обратились по поводу прививки от гепатита В, так как
отказались от вакцинации ребенка против гепатита В. Сейчас они согласились
вакцинировать ребенка.
Какая тактика наиболее целесообразна?
А) Подростки, не получившие вакцинацию против вируса гепатита B, не подлежат
вакцинации
В) Подростки, не получившие вакцинацию против вируса гепатита B, подлежат
предварительному обследованию
С) Подростки, не получившие вакцинацию против вируса гепатита B, подлежат
вакцинации через 6 мес
D) Подростки, не получившие вакцинацию против вируса гепатита B, подлежат
вакцинации через 12 мес
Е) +Подростки, не получившие вакцинацию против вируса гепатита B, подлежат
немедленной вакцинации

174) 42-летняя женщина обращается по поводу болей в животе и в правом боку,


начавшихся неделю назад, сопровождаются зловонным, слегка окрашенным жидким
стулом. Боль в правом боку началась неделю назад, интенсивность боли 8 из 10,
усиливается при движении. В анамнезе – периодически возникающие артралгии
коленного сустава, пептические язвы антрального отдела желудка и 12-ПК. Принимает
омепразол. PS 89/мин, АД 110/75 мм рт ст. Живот болезнен в эпигастральной и правой
подвздошной области при пальпации. ОАМ – следы эритроцитов.
Дальнейшая оценка пациента какой вероятнее всего покажет результат?
A) Гипертонический криз
B) Кишечные ганглионевромы
C) +Гиперкальциемию
D) Легочный стеноз
E) Удлинение QT на ЭКГ

175) Ранее здоровый 37-летний мужчина обратился к врачу по поводу перемежающейся


боли в эпигастральной области, беспокоящей его в течение последних 8 недель. Также
беспокоило чувство распирания после еды. Пьет 1-3 банки пива в неделю. При пальпации
живота – легкая чувствительность в эпигастральной области.
Какое исследование необходимо назначить пациенту?
A) +Дыхательный уреазный тест
B) Фиброгастродуоденоскопию
C) Терапевтический тест с ингибиторами протонной помпы
D) УЗИ брюшной полости
E) рН-метрию

176) 65-летняя женщина обратилась по поводу затруднения глотания пищи и дискомфорт


в груди, иногда ощущает «чувство застревания пищи» и «булькающий звук» из горла во
время еды, время от времени кашляет кусочками непереваренной пищи, неприятный вкус
и запах во рту. В анамнезе – болезнь Рейно, принимает нифедипин. Отец умер от рака
желудка. ОАК: Hb 14 г/л, лейкоцитов – 9.8х109\л, тромбоцитов – 215.
Что целесообразно назначить для подтверждения диагноза?
A) +Рентгенографию с барием
B) Компьютерную томографию
C) Пищеводную манометрию
D) ЭФГДС
E) Эндосокпическое УЗИ

177) 31-летний мужчина обратился с жалобами на общую слабость, отеки, повышение


артериального давления до 180/100 мм рт ст. Со слов пациента ранее ничем не болел, не
обследовался. Ухудшение состояния связывает с переохлаждением 2 недели назад, когда
был в горах с ночевкой. У отца артериальная гипертензия. Объективно: отеки на лице и
нижних конечностях. Тоны сердца приглушены, ритмичны. Артериальное давление
190/130 мм рт.ст. Креатинин сыворотки – 300 мкмоль/л. Общий анализ мочи: белок 0,8
г/сут, эритроциты – 0-1, лейкоциты – 3-4 в поле зрения. Суточный диурез – 600 мл/сут.
Какой метод исследования наиболее показан для верификации диагноза?
A) КТ почек
B) Биохимический анализ крови
C) Иммунологические исследования
D) Рентгенография поек
E) +УЗИ почек с допплерографией сосудов

178) 16-летний юноша с жалобами на массивные отеки лица, тела, конечностей. 3 недели
назад перенес острую вирусную инфекцию. Отеки появились неделю назад. Объективно:
общее состояние тяжелое. Отеки на лице, нижних конечностях мягкие, рыхлые. Анасарка.
Артериальное давление 90/50мм.рт.ст. Общий анализ крови – гемоглобин 109 г/л,
лейкоциты 12х109/л, СОЭ 60 мм/час. Общий белок 33 г/л, альбумин 19 г/л, холестерин
общий 17,3 ммоль/л, креатинин 75 мкмоль/л. Общий анализ мочи – белок 3,6 г/л,
лейкоциты 1-2 в п/зр, Эритроциты 0-1 в п/зр. Суточная протеинурия 4,3 г/л.
Какая тактика врача общей практики является наиболее приемлемой?
A) Консультация нефролога
B) +Экстренная госпитализация
C) Плановая госпитализация
D) Амбулаторное лечение
E) Стационар на дому

179) 60-летний мужчина с жалобами на редкое мочеиспускание малыми порциями,


чувство неполного опорожнения мочевого пузыря, выраженные боли над лоном,
поясничной области, выраженную слабость, похудание, отеки на лице, руках, нижних
конечностях, повышение артериального давления, запах аммиака изо рта, потемнение
мочи. Состоит на учете у онколога по поводу заболевания легких. Ухудшение состояния
отмечает в течение месяца. Общее состояние средней тяжести. артериальное давление
170/110 мм.рт.ст, запах аммиака изо рта, отеки. Биохимический анализ крови: креатинин
крови - 256 мкмоль /л, мочевина 18 ммоль/л, калий 6,4 ммоль/л, натрий 137 ммоль/л.
Общий анализ мочи – протеинурия 1,1 г/л, лейкоциты 8-10 в поле зрения, эритроциты – 45
в поле зрения. УЗИ: двухсторонний гидронефроз, задержка мочи.
Какая тактика ведения является наиболее целесообразной?
A) Консультация онколога
B) +Экстренная госпитализация в урологическое отделение
C) Консультация уролога
D) Плановая госпитализация в онкологическое отделение
E) Стационар на дому

180) 10-летняя девочка, с жалобами на частое, болезненное мочеиспускание,


сопровождающееся резями, частые позывы, субфебрильную температуру нарушение сна,
энурез. В анамнезе: болеет третий день, связывает с переохлаждением. Общий анализ
мочи: эритроциты 5-6 в п/з, лейкоциты 10-16 в п/зр, бактерии++.
Какой диагноз наиболее вероятен?
A) Вульвит
B) Гломерулонефрит
C) +Цистит
D) Острый пиелонефрит
E) Тубулоинтерстициальный острый нефрит

181) 5-летний ребенок по поводу хронической болезни почек 4 стадии на фоне


врожденной патологии почек постоянно получает активную форму витамина D для
лечения вторичного гиперпаратиреоидизма. При прохождении диспансерного осмотра
обнаружена гиперфосфатемия 3,5ммоль/л.
Какая коррекция в лечении наиболее целесообразна?
A) Назначить гипофосфатную диету
B) Назначить активатор рецепторов витамина D
C) Назначить кальцимиметик
D) +Отменить активную форму витамина D
E) Назначить фосфат-биндер

182) Мама жалуется, что ее 1,5 годовалый ребенок употребляет жидкость в большом
количестве, часто мочится, периодически повышение температуры, рвоту. При осмотре:
ребенок обезвожен, имеет место задержка роста, гипотрофия, кожные покровы сухие,
тургор ткани снижен. Ребенок постоянно просит пить, со слов матери - диурез больше 2л
в сутки. Общий анализ крови: гемоглобин 115 г/л, гематокрит 50. Биохимический анализ
крови – натрий 157 ммоль/л, К 2,8 ммоль/л, Са 1,3 ммоль/л, Р 1,6 ммоль/л. Общий анализ
мочи – относительная плотность 1002, лейкоциты 2-3 в п/зр, Эритроциты 0-1 в п/зр. На
УЗИ почек – признаки 2-х стороннего гидронефроза. Назначен гидрохлортиазид.
Какая дополнительная терапия наиболее оптимальна?
A) +Амилорид (комбинация тиазидных с калий сберегающими)
B) Верошпирон
C) Фуросемид
D) Этакриновая кислота
E) Гипотиазид

183) 13-летний мальчик обратился после стационарного обследования и лечения по


поводу рецидивирующей гематурии в течение длительного времени. Заключение
морфологического обследования: диффузная пролиферация мезангиальных клеток и
депозиты IgA в мезангии. В контрольном общем анализе мочи: белок 0,099 г/л (<1) ,
лейкоциты 2-3 в п/зр, эритроциты 20-30 в п/зр. Биохимический анализ крови: креатинин
сыворотки 90 мкмоль/л, мочевина 8,5 ммоль/л.
Какая тактика лечения наиболее эффективна в амбулаторных условиях?
A) Диетотерапия
B) Фитотерапия
C) Малые дозы преднизолона
D) Антибактериальная терапия
E) +Ингибиторы АПФ/БРА

184) 13-летняя девочка. обратилась с жалобами на тянущие боли в поясничной области,


повышение температуры тела, озноб. Считает себя больной в течение трех дней, неделю
назад перенесла ОРЗ. При осмотре: кожные покровы обычной окраски, отеков нет,
температура тела 39°С, АД 100/ 70 мм.рт.ст. симптом поколачивания положительный с
обеих сторон. В общем анализе крови: лейкоциты 16 х 109/л, СОЭ 40мм/ч. В общем
анализе мочи: уд. вес 1016, белок 0,66г/л, лейкоциты – сплошь, эритроциты 1-2 в п/з,
цилиндры гиалиновые – единичные.
Какая тактика наиболее показана?
A) Цистография
B) Назначить посев мочи на чувствительность к антибиотикам
C) +Эмпирическая антибиотико терапия+посев мочи
D) Антибактериальная терапия
E) Консультация нефролога/уролога

185) 18-летняя девушка обратилась с жалобами на повышение температуры до 39◦С,


слабость, недомогание, боли в поясничной области. Из анамнеза: заболела остро 3 дня
назад, самостоятельно принимала парацетамол, бисептол - без эффекта. Объективно:
кожные покровы бледные, влажные. Температура тела 39,5◦С, отеков нет. Артериальное
давление 110/70 мм ртст, пульс 92 удара в мин. При пальпации – болезненность в
поясничной области. Общий анализ крови: гемоглобин 120 г/л, лейкоциты 12х109/л, СОЭ
40 мм/ч. Общий анализ мочи: мутная, белок 0,066%, лейкоциты – сплошь, бактерии +++.
Назначение какого антибиотика является наиболее целесообразным?
A) Амикацин
B) Канефрон
C) Ципрофлоксацин
D) +Цефтриаксон
E) Азитромицин
186) 67-летняя женщина обратилась по поводу оценки болей в коленях за прошедший год.
Боль усиливается при движении и облегчается при отдыхе. В анамнезе СД 2 типа.
Принимает метформин. Рост 165 см, вес 85 кг, ИМТ-31, 2 кг/м2. Осмотр – боль при
полном сгибании, разгибании суставов, крепитация при движениях в суставе, ограничен
диапазон движения обоих колен. На рентгене – сужение суставного пространства,
субхондральный склероз, субарахноидальные кисты.
Какая тактика наиболее целесообразна?
A) Внутрисуставная инъекция глюкокортикостероидов
B) +Назначение ибупрофена
C) Назначение метотрексата
D) Назначение преднизолона перорально
E) Назначение целекоксиба

187) 53-летний мужчина с жалобами на сильные, невыносимые, жгучие боли в левом


коленном суставе последние 8 часов. Не может ходить. 10 месяцев назад был приступ
острой боли и отека правого большого пальца стопы, который прошел после
индометацина. В анамнезе СД 2 типа, АГ, псориаз, гиперлипидемия. Принимает
бетаметазон, метформин, глипизид, лозартан, симвастатин, гидрохлортиазид. ИМТ-38,1
кг/м2. Множественные чешуйчатые бляшки на ладонях и подошвах. Левое колено
эритематозное, опухшее, диапазон движений ограничен.
Какая тактика наиболее целесообразна?
A) + Назначение индометацина перорально
B) Назначение колхицина перорально
C) Назначение цефтриаксона внутривенно
D) Пункция сустава
E) Назначение метотрексата перорально

188) 62-летняя женщина обратилась по поводу планового осмотра. 6 месяцев назад


вакцинировалась от гриппа. Отец умер от инфаркта миокарда в 50 лет. Мамография без
патологии. Курит 1 пачку сигарет в день в течение 19 лет, пьет 4-5 стаканов вина в день.
По поводу АГ принимает эналаприл, аспирин в низких дозах. ИМТ- 18,4 кг/м2.
Что необходимо провести?
A) +Денситометрию костей
B) Вакцинировать против гриппа
C) УЗИ брюшной полости
D) Колоноскопию
E) Эхокардиографию

189) 68-летняя женщина обратилась по поводу болей в пояснице, которая началась


внезапно 2 недели назад после того, как она встала со стула. В анамнезе – АГ, рак
молочной железы 15 лет, хроническая двусторонняя боль в коленях, нейросенсорные
нарушения слуха. Принимает гидрохлортиазид, парацетамол. Болезненность при
пальпации поясничной области, оба колена увеличены и опухшие. Рентген – перелом
позвонка L4.
Какой диагноз наиболее вероятен?
A) Остеопороз
B) Ревматоидный артрит
C) Остемаляция
D) +Болезнь Педжета
E) Холангит

190) 58-летняя женщина с жалобами на боли в левом коленом суставе в течение 3 месяцев,
утреннюю скованность при пробуждении 10-15 минут. Боль усиливается при работе стоя,
подъеме по лестнице. Принимала ибупрофен. В анамнезе – АГ, гиперхолестеринемия.
ИМТ- 31.8 кг/м2. При осмотре – болезненность при пальпации сустава, крепитация и боль
при полном сгибании и разгибании. Мочевая кислота 8,0 (у женщин норма до 6 ) мг/длЮ
СОЭ 15 мм/ч.
Что вероятнее всего выявится при визуализации левого колена?
A) Кальцификация синовиальной и хрящевой ткани на УЗИ
B) +Остеофиты с сужением суставного пространства на рентгенограмме
C) Выпот в суставе на УЗИ
D) Краевые эрозии и непрозрачные периартикулярные мягкие ткани на рентгенограмме
E) Склеротические поражения костей на рентгенограмме
191) 43-летний мужчина по поводу боли в суставах пальце ног и тошноту в течение 2
часов. Боль приходит волнами и распространяется в пах. За последний год периодически
возникали боли с двух сторон, боли в суставах пальцев ног, лодыжках. Не посещал врача
более 10 лет. У сестры ревматоидный артрит. При осмотре – болезненность в костно-
позвоночных областях с обеих сторон. На левом ухе – подкожные узелки (рисунок). На
КТ – множественные мелкие камни в почках и 7-мм камень в дистальном отделе левого
мочеточника.
Что вероятнее всего выявит биопсия подкожных узелков на левом ухе?
A) Холестерин
B) +Ураты
C) ЦистинГранулемы
D) Оксалаты кальция
E) Пирофосфаты кальция

192) 48-летняя женщина обратилась по поводу болезненной припухлости позади правого


колена, беспокоит в течение 2-х месяцев. Периодически поднималась Т тела до 37.8ᴼС,
утомляемость. За последний год периодически возникали боли в руках и запястьях с двух
сторон. В анамнезе СД 2 типа и АГ. ИМТ-25 кг/м2. При осмотре - в правой подколенной
ямке образование 3,0 см, видна при разгибании колена; легкая припухлость правого
коленного сустава.
Какой диагноз наиболее вероятен?
A) Остеоартрит
B) Псоритаический артрит
C) Аневризма подколенной артерии
D) +Ревматоидный артрит
E) Подагра

193) 45-летний мужчина пришел к врачу для оценки ограниченной подвижности правой
руки в течение 1 года, трудно вытянуть 4-й и 5-й пальцы, несмотря на занятия
гимнастикой. Работает каменщиком более 20 лет. Принимает метформин. Курит пачку
сигарет в день в течение 25 лет, пьет пиво каждый день после работы. Осмотр –
сморщивание кожи вблизи проксимальной складки сгибателей, несколько болезненных
ладонных узелков, прилегающих к дистальной ладонной складки. Активное и пассивное
разгибание в 4-м и 5-м пальцах ограниченно.
Какой основной механизм возникновения симптомов?
A) Остеортрит
B) Опухоль сухожилия
C) +Ладонный фиброматоз (Контрактура Дюпюитрена)
D) Тендовагинит
E) Ульнарное поражение нерва

194) 43-летний мужчина обратился по поводу отека в задней части левого колена,
возникшее 2 месяца назад. Отек безболезнен, но боль возникает при долгом стоянии.
Отмечает утреннюю скованность по утрам, исчезает через несколько минут. Осмотр – при
принудительном сгибании стопы – появляется боль в колене; на медиальной стороне
левой подколенной ямки – мягкое образование d=3,0 см, исчезающая при сгибании
колена.
Какой предрасполагающий фактор риска самый сильный для состояния пациента?
A) Варикозное расширение вен
B) Дефект в синтезе коллагена
C) Семейный анамнез липоматоза
D) +Подколенная киста
E) Диета, богатая пурином

195) 43-летняя женщина с жалобами на прогрессирующую слабость в течение 3 недель.


Ей стало трудно расчесывать волосы и подниматься по лестнице. У нее гипертония.
Выкуривала пачку сигарет ежедневно в течение 25 лет, не употребляет алкоголь. У матери
ИБС и СКВ. Принимает хлорталидон, витамины. Т тела 37, 8ᴼС, PS 71/мин, АД 132/84 мм
рт ст. На глазу сыпь (рисунок), диффузная эритема верхней части спины, задней части
шеи и плеч.
Какие антитела выявятся у пациентки?
A) +миозитспецифические антитела
B) антигистоновые антитела
C) антитела на основе потенциал-управляемых кальциевых каналов
D) антицентромерные антитела
E) антитела против миелопероксидазы

196) В течение последних 2 лет к врачу приходит на приме 40-летний мужчина по поводу
усталости, повышенной потливости и зуда в ногах. В анамнезе хронический бронхит,
курил 2 пачки сигарет ежедневно в течение 24 лет. Принимает ингаляторнотиотропия
бромид. Рост 175 см, вес 116 кг, ИМТ-38 кг/м2. Покраснение лица и синеватая окраска
губ. ОАК: эритроцитов – 6.9х1012/л, Hb 20 г/л, лейкоцитов – 13,0х109\л, тромбоцитов –
540, ферритин 8 нг/мл, железо сыворотки – 48 мкг/дл.
Что является наиболее подходящим в следующем этапе лечения?
A) Снижение веса
B) Прием железа
C) Трансплантация стволовых клеток
D) Назначение ингаляционного будесонида
E) +Кровопускание

197) Родители 16-летнего мальчика обратились к врачу по поводу того, что он устает, из-
за этого бросил заниматься спортом; снизилась успеваемость в школе. Дома в основном
проводит время в подвале, играя в видеоигры, ест «мюсли». Осмотр – бледный, трещины
в уголках рта, вогнутость ногтей. ОАК: Hb 115 г/л, лейкоцитов – 7,0х109\л, тромбоцитов –
290.
Что необходимо для начального этапа лечения?
A) Регулярные переливания крови
B) Назначение витамина В12
C) +Назначение железа
D) Назначение L-тироксина
E) Трансплантация стволовых клеток

198) 34-летняя женщина обратилась по поводу лихорадки и недомогания. В течение


последних 2 дней ощущала усталость и слабость, озноб. Прошлой ночью была
температура 40,2ᴼС, с трудом глотает с утра. Недавно пациентке был поставлен диагноз
«Болезнь Грейвса», принимает метимазол. Т тела 38,3ᴼС, PS 95/мин, АД 134/74 мм рт ст.
Ротоглотка эритематозная. ОАК: Гематокрит 42%, Hb 134 г/л, лейкоцитов –3,2х109\л, с/я
нейтрофилов – 9%, лимфоцитов-79%, моноцитов 11%, тромбоцитов – 230.
Какая тактика наиболее целесообразна?
A) Биопсия костного мозга
B) +Прекратить прием метимазола
C) Уменьшить дозу метимазола
D) Назначение аминопенициллина
E) Увеличение дозы метимазола
199) Родители 4-летней девочки обратились по поводу усталости в течение 3 недель, легко
образуются синяки. В течение прошлой недели у нее был жар и сильная боль в ногах, от
которой она просыпается ночью. Т тела 38,3ᴼС, PS 120/мин, ЧДД 30/мин. При осмотре -
шейная и подмышечная лимфаденопатия. Печень пальпируется на 3 см ниже правого
реберного края, селезенка – на 2см ниже левого реберного края. ОАК: Hb 101 г/л,
лейкоцитов – 63х109\л, тромбоцитов – 27. В пунктате костного мозга – преобладают
незрелые клетки.
Какой диагноз наиболее вероятен?
A) Лимфома Ходжкина
B) +Острый лимфобластный лейкоз
C) Апластическая анемия
D) Идиопатическая тромбоцитопеническая пурпура
E) Острый миелобластный лейкоз

200) 35-летний мужчина обратился по поводу утомляемости и генерализованной слабости,


которые появились в течение прошлого года. Отмечает уменьшение количества
испражнений, боли при дефекации и небольшое количество крови при вытирании. Т тела
36,5ᴼС, PS 50/мин, АД 120/90 мм рт ст., ИМТ=35 кг/м2. Сухость кожи и вздутый живот.
Отеки на нижних конечностях.
Что вероятнее всего покажет дальнейшая оценка пациента?
А) Повышенный уровень кортизола
B) Повышенный сывороточной креатинкиназы
C) снижение креатинина сыворотки
D) +Повышенный уровень ЛПНП
E) Гиперкальциемию

201) 50-летняя женщина, пианистка, обратилась по поводу боли и онемения в правой руке
в течение 6 недель, ощущение «покалывания булавками и иголками». Состояние
ухудшается ночью, проходит после сжимания руки. Отмечает эпизодические боли в левом
колене в течение дня. В анамнезе – АГ, контролируемая лизиноприл, принимает
безрецептурные лекарства от запоров. Выглядит утомленной, ИМТ=35 кг/м2, PS 57/мин,
АД 120/75 мм рт ст. Нормальный диапазон движения в запястьях, глубокие сухожильные
рефлексы 1+, положительные тесты Тинеля и Фалена, мягкий отек ног.
Что целесообразно назначить для лечения?
A) Метотрексат
B) Ибупрофен
C) Хирургическую декомпрессию
D) +L-тироксин
E) Преднизолон

202) 7-летняя девочка с жалобами на скудное безболезненное кровотечение из влагалища,


появившееся 6 часов назад. У ее старшей сестры первая меснтруация в 11 лет. На 95-м
процентиле по весу. Кожа жирная, редкие подмышечные волосы. Молочные железы и
ареолы выступают в виде конуса, без границы между ними. Наружные половые органы
нормальные. Сывороточная глюкоза 189 мг/дл. Проба с диферилином – увеличение
уровня лютеинизирующего гормона.
Что является основной причиной данного состояния?
A) Переизбыток кортизола надпочечников
B) Эктопическая выработка гормонов
С) Дефицит гормонов щитовидной железы
D) Гиперинсулинемия
E) +Высвобождение ГнРГ (гонадотропин-рилизинг-гормон)
203) 69-летняя женщина обратилась из-за усталости, вялости в течение 5 дней, также
отмечает слабость и тошноту последние 3 дня. Страдает саркоидозом, депрессией, АГ;
перенесла инсульт 5 лет назад. Принимала аспирин, нифедипин, преднизолон,
флуоксетин, розувастатин. В связи с выездом за рубеж, не принимала никаких лекарств
последние 7 дней. Т тела 36, 1ᴼС, PS 95/мин, АД 85/65 мм рт ст. Вялая. ОАК: Hb 134 г/л,
лейкоцитов – 9,6х109\л. Кортизол, АКТГ – снижены.
Какое отклонение в лабораторных анализах вероятнее всего ожидать?
A) Гиперкалиемия
B) +Гипонатриемия
C) Гипергликемия
D) Гипернатриемия
E) Гипокалиемия

204) 50-летняя женщина обратилась по поводу сердцебиения, раздражительности. За


последние 4 месяца было несколько эпизодов аритмии от 30 сек-до нескольких часов.
Потеря в весе 8.8 кг за последние 4 мес. В анамнезе астма, принимает ингаляционные
бронходилататоры. PS 102/мин, АД 148/98 мм рт ст. Кожа теплая и влажная. В передней
части шеи – пальпируется образование. Анти-ТТГ – более 1.75 МЕ/л.
Какая тактика целесообразна в долгосрочной преспективе?
A) +Терапия L-тироксином
B) Терапия йодидом калия
C) Эстрогенная заместительная терапия
D) Тиреоидэктомия
E) Терапия пропанололом

205) Мужчина 48 лет, жалобы на кашель, невыраженные симптомы интоксикации,


температура тела 37,3°С, ЧДД 18 в минуту, на рентгенограмме легочная инфильтрация в
пределах 1 сегмента, лейкоциты 9,0-10,0 х 109/л. Живет в благоустроенной квартире, с
женой. Врачом был открыт стационар на дому.
Какая форма заполняется на больного при открытии стационара на дому (Приказ МЗ
№669 от 17.08.2015г. «Об утверждении Правил оказания стационарозамещающей
помощи»)?
A) +№ 003-2/у
В) № 025-8/у
С) № 030-6/у
D) № 027/у
Е) № 066-4/у

206) Мужчина 48 лет, получил стационарозамещающую помощь по поводу


внебольничной пневмонии легкого течения.
По какой форме заполняется статистическая карта при выписке пациента, получившего
стационарозамещающую помощь (Приказ МЗ №669 от 17.08.2015г. «Об утверждении
Правил оказания стационарозамещающей помощи»)?
A) № 003-2/у
В) № 025-8/у
С) № 030-6/у
D) № 027/у
Е) +№ 066-4/у

207) Мужчина 48 лет, получил стационарозамещающую помощь по поводу


внебольничной пневмонии легкого течения.
По какой форме выдается выписка из медицинской карты больного с необходимыми
рекомендациями на руки пациенту, получившего стационарозамещающую помощь
(Приказ МЗ №669 от 17.08.2015г. «Об утверждении Правил оказания
стационарозамещающей помощи»)?
A) № 003-2/у
В) № 025-8/у
С) № 030-6/у
D) +№ 027/у
Е) № 066-4/у

208) Мама ранее здоровой 13-летней девочки обратилась по поводу изменений в


поведении ребенка в течение 6 месяцев – частые перепады настроения. Иногда девочка
бывает раздражительной в течение нескольких дней и легко выходит из себя, в перерывах
между эпизодами ведет себя «нормально», проводит время со своими друзьями,
занимается гимнастикой. Мать заметила, что девочка стала затрачивать больше времени
на подготовку к занятиям, иногда наносит чрезмерный макияж; пропускает школу.
Менструации регулярные.
Что из перечисленного является наиболее вероятным объяснением данного поведения?
A) Пограничное расстройство личности
B) +Нормальное поведение
C) Нарушение дисрегуляции настроения
D) Депрессивное расстройство
E) Предменструальный синдром

209) Мама 5-летней девочки обратилась к врачу по поводу невнимательности девочки в


детском саду, она не выполняет задания и не слушает воспитателей, отказывается
говорить с родителями и сверстниками, предпочитает играть в одиночку с 5 красными
машинами, выстраивая их неоднократно по прямой линии. Ее мать говорит: «Она
игнорирует все, что я ей говорю». На приеме избегает зрительного контакта с врачом и
матерью на протяжении всего визита.
Какой диагноз наиболее вероятен?
A) +Расстройство аутистического спектра
B) Синдром дефицита внимания и гиперактивности
C) Оппозиционно-вызывающее расстройство
D) Нарушение слуха
E) Расстройства поведения

210) Ранее здоровая 21-летняя женщина обратилась по поводу потери веса и усталости. За
последние 12 месяцев она похудела на 10, 5 кг, чувствует себя уставшей каждый день и
тяжело просыпается по утрам. В течение 2-х лет она вегетарианец, ест только пищу,
которую приготовила сама. Рост 160 см, вес 42 кг, ИМТ-16, 4кг/м2. PS 39/мин, АД 100/50
мм рт ст. Осмотр – истощена, сухая кожа, покрытая тонкими мягкими волосами по всему
телу. Биохимия крови: Na+-142 ммоль/л, CL-103 ммоль/л, K+-4.0 ммоль/л, азот мочевины
10 мг/дл, креатинин-1,0 мг/дл, глюкоза 2.0 ммоль/л.
Какая тактика наиболее целесообразна?
A) +Стационарная реабилитация
B) Амбулаторное наблюдение с ведением дневника наблюдения
C) Амбулаторная терапия с посещением психолога
D) Стационар на дому
E) Дневной стационар

211) 24-летняя безработная женщина обратилась по поводу того, что она в последнее
время «чувствует себя грустно и иногда плачет». При расспросе выяснилось, что
пациентка проводит много времени лежа в кровати, играя или читая. Персетала ходить в
спортзал, ест больше, прибавила в весе (+4,0 кг за 4 недели). Выпивает бокал вина в день,
3 недели назад стала курить марихуану. При осмотре – настроение подавлено, спокойна,
внимательна, речь организована и логична. Мысли о самоубийстве отрицает.
Какой диагноз наиболее вероятен?
A) +Расстройство адаптации
B) Депрессивное расстройство
C) нарушение использования веществ
D) Биполярное расстройство
E) Дистимическоерасстройтсво

212) 17-летняя девочку привела на прием к врачу ее мать по поводу акне. 6 месяцев назад
у девочки появились папулы на спине и плечах, девочка носит только закрытую одежду.
Она проводит много времени перед зеркалом, бросила заниматься плаванием, описывает
себя как «некрасивую». Сидела на низкоуглеводной диете, потеряла 5.2 кг. Выглядит
подавленной и раздражительной. Рост 170 см, вес 62 кг, ИМТ-21.4 кг/м2. Осмотр –
несколько маленьких папул, многочисленные царапины на шее, спине, ягодицах. Нет
суицидальных мыслей.
После создания терапевтического взаимодействия с врачом, что является наиболее
подходящим шагом в управлении заболеванием?
A) Пищевая реабилитация
B) +Когнитивно-поведенческая терапия
C) Предложить госпитализацию
D) Поведенческая терапия
E) Амбулаторное лечение

213) 53-летняя женщина обратилась по поводу сильной боли в правой ноге при ходьбе в
течение 2-х месяцев. Боль проходит после 5-минутного покоя. В анамнезе АГ,
мерцательная аритмия, СД 2 типа. Последние 32 года выкуривает пачку сигарет в день.
Принимает метформин, эналаприл, аспирин и варфарин. PS нерегулярный. Правая нижняя
конечность холоднее левой, кожа на правой ноге выглядит блестящей и сухой.
Какая тактика наиболее целесообразна?
A) МРТ обследование позвоночника
B) Исследование нервной проводимости
C) Ангиография
D) УЗИ сосудов нижних конечностей
E) +Измерение плече-лодыжечного индекса

214) Ранее здорового 14-летнего мальчика привел отец, так ребенок не выходил из своей
комнаты в течение 2-х дней. Отец говорит, что у ребенка нет аппетита, появилась
бессонница, раздражительность в течение 3 недель. Он перестал заниматься футболом,
избегает своей семьи и друзей, но признает, что одинок. Не пьет алкоголь, не употребляет
наркотики. На приеме ребенок сказал, что «не хочет лечиться, а хочет умереть и положить
конец страданиям».
Какая тактика наиболее целесообразна?
A) Начать амбулаторное лечение с препаратов лития
B) Заверить пациента, что он будет чувствовать себя лучше после лечения
C) Начать амбулаторную психотерапию
D) Согласиться с желаниями пациента
E) +Принудительная госпитализация после информирования родителей
215) 25-летний мужчина обратился по поводу сильной боли в спине. Описывает боль как
стреляющую, нанесение удара. По 10-балльной шкале оценивает боль от 9-10. Боль
появилась после поднятия тяжелого ящика на работе, работал в супермаркете кассиром,
недавно стал кладовщиком. Осмотр – диапазон движения в норме, позвоночник
безболезненный. После завершения осмотра пациент просит написать заключение о его
«неспособности работать кладовщиком».
Какой из ответов является наиболее подходящим?
A) «Прежде чем написать заключение, я хотел бы поговорить о вашей работе. Вы,
кажется, очень недовольны своей работой?»
B) «Я вам выпишу заключение. Поскольку работа может ухудшить ваше состояние, я вам
выпишу больничный лист»
C) «Я понимаю вы обеспокоены. Но я могу вас заверить, что вы сможете возобновить
свою работу в качестве кладовщика»
D) +«Я понимаю, что вам неудобно, но результаты не соответствуют серьезности ваших
симптомов. Давайте поговорим о послдених изменениях в вашей работе»
E) «Результаты физического осмотра не соответствуют вашим симптомам, что говорит о
психологической проблемах. Я вас направлю к специалисту по психическому здоровью»

216) Вы осматриваете мужчину на дому, жена пациента сидит рядом, мальчик 4 лет играет
с упаковками от лекарств. Внезапно мать обращает внимание, что блистер с теофиллином
пуст. Мать думает, что возможно ребенок случайно проглотил 10 таблеток теофиллина, но
не знает точного количества.
Какая тактика является первоочередной?
A) Дать активированный уголь ребенку, чтобы способствовать опорожнению желудка и
уменьшить поглощение теофиллина
В) +Вызвать бригаду скорой помощи и срочно отправить ребенка в больницу
C) Позвонить в токсикологический центр
D) Успокоить мать и сказать, что теофиллин безвреден для ребенка
E) Вызвать социальную службу

217) Женщина 75 лет, с жалобами на повышение АД до 160/100 мм рт ст. вызвала бригаду


скорой медицинской помощи (СМП). По результатам данных осмотра, динамики
состояния пациента на фоне проведенных лечебных мероприятий, врачом бригады СМП
было принято решение «оставить на дому (по месту проживания)».
По какой форме заполняется врачом бригады ССМП сигнальный лист в случае
заболевания пациента и необходимости его посещения на дому участковым врачом?
A) +№ 110-1/у
В) № 025/у
C) № 114/у
D) № 001-3у
E) № 003-2/у

218) Сколько времени занимает весь процесс оценки состояния одного пациента согласно
медицинской сортировке по Триаж системе?
A) не более 10 секунд
В) не более 15 секунд
C) не более 30 секунд
D) не более 45 секунд
E) +не более 60 секунд
219) Женщина 65 лет, страдает артериальной гипертензией несколько лет, вызвала
бригаду скорой медицинской помощи (СМП) в связи с повышением АД до 160/100 мм
ртст..
Какое время прибытия бригады СМП оптимальное при данной категории срочности?
A) до 10 минут
В) до 15 минут
C) до 30 минут
D) до 45 минут
E) +до 60 минут

220) Мужчина 45 лет, жалобы на кашель в течение дня, по утрам - с мокротой, одышку,
возникающую при ходьбе на 100 м, приходится останавливаться.
В анамнезе - курит в течение 25 лет по 1 пачке (20 сигарет) в день.
Какова степень выраженности одышки по шкале MedicalResearchCouncilDyspneaScale
(MRS)?
A) степень 0
B) степень 1
C) степень 2
D) +степень 3
E) степень 4

221) 8-летний мальчик с жалобами на прогрессивно ухудшающийся кашель, в течение 7


дней. Сухой приступообразный кашель эпизодами кашлевых толчков 5-10 подряд, после
каждого эпизода делает глубокий шумный вдох. Его иногда рвало после приступа кашля.
Отмечает насморк за неделю до появления кашля. Мать получила прививку против
столбняка, дифтерии и коклюша 11 лет тому назад, карта прививок отца недоступна.
ОАК: лейкоцитов – 14х109/л.
Какие рекомендации наиболее подходят для этой семьи?
A) Введение перорально азитромицина ребенку и прививка против коклюша отцу
B) Введение эритромицина перорально ребенку и прививка против коклюша отцу
C) Введение перорально азитромицина всем членам семьи и прививка против коклюша
отцу и матери
D) Назначение перорально триметоприма всем членам семьи и прививка против коклюша
отцу
E) Назначение перорально азитромицина всем членам семьи и прививка против коклюша
отцу

222) 43-летняя женщина обратилась по поводу лихорадки, тошноты и непродуктивного


кашля в течение 7 дней. В течение этого периода у нее были головные боли,
утомляемость, боли в мышцах и суставах, усиление одышки в течение 2 дней. В анамнезе
СД 2 типа и остеортрит левого коленного сустава. Принимает инсулин и ибупрофен.
Курила 2 пачки сигарет в день в течение 20 лет, прекратила 10 лет назад. Т тела 38,1ᴼС, PS
94/мин, ЧДД 18/мин, АД 132/86 мм рт ст. Поражение кожи с синим центром, бледной
промежуточной зоной и темно-красным периферическим ободком на верхнх и нижних
конечностях.
Что является вероятным возбудителем заболевания?
A) Клебсиелла
B) Пневмококк
C) Гемофильная палочка
D) Золотистый стафилококк
E) +Микоплазма
223) 50-летний мужчина обратился после падения на улице. Он поскользнулся, ударился о
поручень боковой частью груди. Курил в течение 10 лет по 1 пачке сигарет в день, бросил
18 лет назад. Пьет 1-2 банки пива в день. Т тела 37,1ᴼС, PS 78/мин, АД 126/72 мм рт ст.
Левая половина грудной клетки болезненна при пальпации. Рентген – 5 мм очаг в правом
верхнем легочном поле.
Какая тактика наиболее целесообразна?
A) Повторить рентген грудной клетки через 12 месяцев
B) +КТ грудной клетки
C) Позитронно-эмиссионная томография
D) Повторить рентген грудной клетки через 6 месяцев
E) Анализ высвобождения гамма-интерферона

224) 60-летний мужчина проходит осмотр перед холецистэктомией. У матери –


хроническое гранулематозное заболевание легких, работает на стекольном заводе. Курит в
течение 38 лет по 2 пачки сигарет в день.
На рентгене – впервые выявлен периферически локализованный одиночный легочный
узелок.
Какая тактика наиболее целесообразна?
A) Провести анализ газов артериальной крови
B) Выполнить КТ-биопсию
C) Измерение ангиотензин-превращающего фермента
D) +Запросить предыдущую рентгенографию грудной клетки
E) Выполнить КТ с высоким разрешением

225) 43-летняя женщина на обследовании, перед началом работы медсестрой. В течение


последнего года у пациентки была легкая одышка и кашель с белой мокротой, особенно
по утрам. Вакцинирована по возрасту. Курит по 2 пачки сигарет в день в течение 20 лет и
выпивает 1 бокал вина в день. Т тела 36,2ᴼС, PS 74/мин, АД 124/60 мм рт ст. Хрипы по
обоим легочным полям.
Какое исследование необходимо провести для оценки туберкулеза?
A) Анализ высвобождения гамма-интерферона
B) +Микроскопию мокроты
C) Бакпосев мокроты
D) Рентген грудной клетки
E) ПЦР мокроты

226) 24-летний мужчина пришел на прием по поддержанию здоровья. В анамнезе СД 1


типа, принимает инсулин. Последние 5 лет курил 1 пачку сигарет в день. Не получал
никаких прививок. Нормостенического телосложения. ИМТ 23 кг/м2, Т тела 36,2ᴼС, PS
72/мин, ЧДД 35/мин, АД 123/82 мм рт ст.
Какое диагностическое исследование необходимо провести?
A) Микроскопическое исследование мокроты
B) Выполнить ПЦР мокроты
C) Бакпосев мокроты
D) +Рентгенография грудной клетки
E) Пробное лечение рифампицином

227) 65-летняя женщина обратилась по поводу увеличения пальцев в течение 18 месяцев,


охриплость голоса в течение 2 месяцев. Снизился аппетит после инфекции дыхательных
путей 3 месяца назад и потеря веса на 8 кг в этот период. Никогда не курил. Был
выставлен диагноз обструктивного апноэ во сне 10 лет назад. Осмотр – увеличение
выпуклости ногтей, болезненный отек мягких тканей пальцев рук и голеностопных
суставов.
С чем связано состояние пациента?
A) Увеличение антидиуретического гормона
B) +Миоз
C) Периферический цианоз
D) Увеличение сывороточного кальция
E) Уменьшение антидиуретического гормона

228) Укажите рекомендуемую частоту визитов к врачу пациентов группы высокого риска
ИБС (пациенты, перенесшие инфаркт миокарда или инсульт, имеющие нарушения ритма,
частые приступы стенокардии, осложнения в виде сердечной недостаточности,
сопутствующий сахарный диабет) в начале наблюдения?
А) Не менее 1 раз в 1 месяц
B) Не менее 1 раз в 2 месяца
C) Не менее 1 раз в 3 месяца
D) Не менее 1 раз в 4 месяца
+E) Не менее 1 раз в 6 месяцев

229) Что из нижеперечисленного относится к так называемым психосоциальным


факторам = «триггерам», повышающим вероятность возникновения ИБС?
А) +Перегрузки и неадекватный рабочий режим
В) Повышенное содержание общего холестерина
С) Курение
D) Ожирение
Е) Возраст

230) Мужчина 58 лет, с жалобами на загрудинный дискомфорт с типичными


характеристиками.
Какое исследование необходимо в обязательном порядке провести пациенту в качестве
первичного обследования, относящемуся к группе пациентов с ПТВ (ПТВ - предтестовая
вероятность) в пределах 65%?
А) +ЭКГ-тест с физической нагрузкой
В) Стресс-ЭхоКГ
С) Коронарную ангиографию
D) МРТ сердца
Е) Однофотонную эмиссионную компьютерную томографию

231) 48-летняя женщина обратилась по поводу боли в грудной клетке. Она описывает боль
как ощущение сдавливания с отдачей в левую лопатку. Эпизод начался 15 минут назад,
когда она сидела в очереди на прием. Пациентка испытывала такую боль раньше, обычно
по вечерам, проходили в покое. PS 112 ударов/мин, АД 121/81 мм рт ст. Тропонины
сыворотки отрицательны на двух последовательно взятых анализах крови. ЭКГ без
патологии.
Что из нижеперечисленного является лучшим для долгосрочного лечения симптомов
данного пациента?
A) клопидогрель
B) дилтиазем
C) аспирин
D) эналаприл
E) метопролол+++

232) 16-летний мальчик профессионально занимается баскетболом, родители здоровы.


Внезапно упал без сознания во время игры в баскетбол. Через 20 минут был доставлен в
отделение скорой помощи, пульса нет, дыхания нет. Врач констатировал внезапную
смерть. Родители согласились на вскрытие.
Что вероятнее всего обнаружится на вскрытии?
A) Миксоидная опухоль сердца
B) +Гипертрофия межжелудочковой перегородки
C) Дефект межпредсердной перегородки
D) Эмбол в легочной артерии
E) Увеличенная нисходящая аорта

233) 55-летний мужчина обратился в связи с сухим кашлем и сильной болью в груди,
усиливающейся при дыхании. Два месяца назад был поставлен диагноз – инфаркт
миокарда, был имплантирован стент в правую коронарную артерию. В анамнезе – АГ,
гиперхолестеринемия. Принимает аспирин, клопидогрель, аторвастатин, эналаприл.
Температура тела 38,5ᴼС, PS 92 уд/мин, ЧДД 22 в мин, АД 130\80 мм рт ст.
Аускультативно – шум трения перикарда. Тропонин I 0.2 нг/мл (N<0.01 нг/мл).
Что является наиболее вероятной причиной данного состояния?
A) Тромбоэмболия легочной артерии
B) Реинфаркт
C) Расслоение аорты
D) +Синдром постмиокардиального инфаркта (Дресслера)
E) Тампонада сердца

234) Какое профилактические мероприятия в отношении снижения риска развития рака


шейки матки являются наиболее эффективными?
A. +вакцинация 4-валентной ВПЧ вакциной до начала половой жизни подростков
девочек и мальчиков
B. вакцинация 4-валентной ВПЧ вакциной с началом половой жизни у девочек-
подростков
C. регулярный скрининг с Рар-тестом всех женщин старше 35 лет
D. своевременное лечение предраковых состояний шейки матки
E. лечение ВПЧ - инфекции у всех женщин с регулярной половой жизнью

235) Девушка 14 лет, страдающая ювенильным ревматоидным артритом, получает


метотрексат. Мать девушки обеспокоена тем, что у дочери может быть повышен риск
развития онкологической патологии. В семье девушки тетя умерла от рака шейки матки.
Какое высказывание ВЕРНО в отношении развития рака шейки матки в данной ситуации?
A. Риск развития рака у данной категории больных значительно выше, чем в
популяции
B. Риск развития рака у данной категории больных значительно ниже, чем в
популяции благодаря терапии метотрексатом
C. Риск развития рака у ревматологических больных не выше, чем в популяции, но
чаще развиваются лимфомы
D. Риск развития рака у ревматологических больных не выше, чем в популяции, но
чаще развиваются доброкачественные опухоли
E. +Американская ассоциация ревматологов рекомендует данной категории
пациентов вакцинацию от ВПЧ четырехвалентной вакциной для снижения риска рака
шейки матки

236) При обследовании у беременной пациентки были выявлены антитела к ВПГ-1, ЦМВ
и ВЭБ (в ИФА ОП образцов в 2 раза выше ОП критической). Герпетические высыпания
отрицает, жалоб нет, лимфаденопатии не выявлено. Какая тактика целесообразна в данной
ситуации?
A. +Наблюдение
B. Порекомендовать превентивный курс ацикловира
C. Провести курс индуктора интерферона
D. Провести курс ганцикловира
E. Назначить арбидол

237) У 34-летней женщины жалуется на зуд, выделения из влагалища с неприятным


«рыбным запахом». На зеркалах шейка матки - внешний вид не изменен. Какое из
следующего, скорее всего, соответствует этиологии данного состояния?
A. Neisseria gonorrhea
B. Chlamydia trachomatis
C. +Бактериальный вагиноз
D. Кандидозный вагинит
E. Staphylococcusaureus

238) 18-летняя девушка, ведущая половую жизнь, жалуется на острую боль в нижней
части живота, затрудняющую ходьбу. Она утверждает, что боли возникла через 2 дня
после окончания менструации, жалуется на выделения из влагалища. Выберите наиболее
подходящий возбудитель, вызывающий воспалительные заболевания органов малого таза:
A. Staphylococcus aureus
B. Bacteroides fragilis
C. Acinomyces Israeli
D. +Neisseria gonorrhoeae
E. Chlamydia trachomatis

239) Женщина 36 лет, страдающая артериальной гипертензией, длительно получала


антигипертензивный препарат. Но в связи с беременностью 3-4 недель, кардиолог
учитывая тератогенный эффект отменила его. Какой из перечисленных препаратов
НАИБОЛЕЕ вероятно обладает этим эффектом?
a) Клонидин
b) Метилдопа
c) Нифедипин
d) +Периндоприл
e) Метопролол

240) У ребенка 3-х лет в течение нескольких недель рвота, периодические судороги,
высокая температура, сопорозен. Из анамнеза: контакт с больной туберкулезом тетей, под
наблюдением фтизиатра не был. Не привит БЦЖ в связи с мед.отводом. Какое из
перечисленных обследований врач обязан провести в первую очередь для уточнения
диагноза?
a) Общий анализ крови и мочи
b) Рентгенографию легких
c) +Спинномозговую пункцию
d) Копрограмму и бакпосев кала
e) Внутрикожную пробу Манту и Диаскинтест

241) У студента 20 лет, при профилактическом флюорографическом обследовании в S1


правого легкого обнаружены очаги с нечеткими контурами малой и средней
интенсивности. Беспокоит периодически кашель со скудной мокротой. Из анамнеза: курит
в течение 5 лет. Предыдущее флюорографическое обследование в прошлом году без
патологических изменений. МБТ в мокроте не обнаружены. Какой наиболее
вероятный диагноз?
a) Очаговая пневмония
b) Хронический бронхит
c) +Очаговый туберкулез легких
d) Периферический рак легкого
e) Бронхолобулярный инфильтрат

242) Если к Вам обратился пациент с жалобами на лихорадку в течение 2 месяцев,


похудание на 6 кг за это же время, выраженную утомляемость, ночную потливость, при
осмотре были выявлены увеличенные подключичные и подмышечные лимфоузлы, то о
какое заболевание следует исключить в ПЕРВУЮ ОЧЕРЕДЬ?
a) Бруцеллез
b) Туберкулез
c) Сепсис
d) +ВИЧ-инфекция
e) Лимфогрануломатоз

243) Вы наблюдаете пациента, молодого мужчину с жалобами на лихорадку в течение


последнего месяца, похудание на 8 кг за это же время, диарею (стул 2-6 раз в сутки), при
осмотре были выявлены увеличенные подключичные, паховые и подмышечные
лимфоузлы, то о каком заболевании в первую очередь вы подумаете?
a) Хроническая дизентерия
b) Болезнь Крона
c) Сепсис
d) +ВИЧ-инфекция
e) Болезнь Уиппла

244) Мужчина 33 лет, житель Жамбылской области, обратился в поликлинику на 4-й день
болезни. Занимается торговлей баранов. Жалобы на сильную головную боль, озноб,
ломоту в теле, жидкий стул, носовое кровотечение. При осмотре: температура тела –
39,0°С, вялый. Лицо, шея гиперемированы, склеры глаз инъецированы. На туловище, на
задней поверхности бедер необильная петехиальная сыпь. Симптом жгута и щипка
положительные. Тоны сердца приглушены, ЧСС -86 в мин., АД – 80/50. В легких –
везикулярное дыхание. Живот мягкий, безболезненный. Печень увеличена на 1,5 см. Стул
кашицебразный, черного цвета.
a) ИФА на арбовирусы
b) +ПЦР крови на вирус ККГЛ
c) ИФА на маркеры вирусных гепатитов
d) бактериологическое исследование крови
e) бактериологическое исследование кала

245) Женщина 26 лет со сроком гестации 14-15 недель обратилась к врачу с жалобами на
озноб, повышение температуры, выраженную потливость, боли в крупных суставах
верхних и нижних конечностей, слабость. Считает себя больной в течение 3 недель.
Объективно: температура – 38,8С, кожные покровы - влажные на ощупь, пальпируются
шейные, подмышечные, паховые лимфоузлы. Печень увеличена на 1.0 см. Суставы
внешне не изменены, движения в них в полном объеме. Реакция Райта 1:400, Как
наиболее правильно классифицировать данный случай?
a) определенный случай
b) выявленный случай
c) +подтвержденный случай
d) случай у беременной
e) зарегистрированный случай

246) Мужчина, 36 лет, обратился в поликлинику с жалобами на сильно распухшую левую


кисть, повышение температуры, общую слабость в течение 3-х дней. Неделю назад
участвовал в разделке туши коровы, поранил руку. При осмотре: на тыльной поверхности
левой кисти безболезненная язва с плотным струпом черного цвета в центре, вокруг
слившиеся дочерние пузырьки с серозным содержимым, окруженные венчиком
гиперемии в виде валика, выраженный отек левой кисти. Какой диагноз из перечисленных
ниже НАИБОЛЕЕ вероятен?

a) Чума, кожная форма


b) Туляремия, кожная форма
c) Пастереллез, кожная форма
d) Лейшманиоз, кожная форма
e) +Сибирская язва, кожная форма

247) Мужчина 28 лет, житель Западно-Казахстанской области, обратился в


инфекционную больницу на 4-ый день болезни с жалобами на сильную головную боль,
озноб, ломоту в теле, боль в пояснице. При осмотре: температура тела – 39,9°С, вялый.
Лицо, шея гиперемированы, пастозны, склеры глаз инъецированы. На ягодицах, в местах
инъекций гематомы. Симптом жгута и щипка положительные. Симптом поколачивания по
пояснице положительный. Печень увеличена на 1,5 см. Снижение диуреза. Какой диагноз
НАИБОЛЕЕ вероятен?
a) лихорадка Денге
b) иктеро-геморрагическая лихорадка
c) Омская геморрагическая лихорадка
d) Конго-Крымская геморрагическая лихорадка
e) +Геморрагическая лихорадка с почечным синдромом

248) Девочка 8 месячного возраста осмотрена участковым врачом. Со слов матери


жалобы на повышение температуры тела до 39С, вялость, отказ от груди, рвоту. Сегодня,
на 4-й день болезни присоединилась желтуха, ребенок стал резко возбужденным,
отмечались судороги, рвота типа "кофейной гущи". В анамнезе: 3 месяца назад ребенку
проводилось переливание крови. Общий билирубин 165 мкмоль/л, прямая фракция - 144
мкмоль/л, АЛТ - 5,8 мкмоль/л, АСТ - 3,7 мкмоль/л, тимоловая проба -12 Ед. Наиболее
первоочередные действия участкового врача в данной ситуации.
a) передать экстренное извещение, срочно кровь на маркерную диагностику
b) срочно маркерная диагностика, затем госпитализация
c) срочно ОАК, затем госпитализация
d) передать экстренное извещение, до получения результатов маркерной диагностики
наблюдать на дому
e) +передать экстренное извещение, больную срочно госпитализировать

249) 38-летний мужчина, чабан, житель Аральского района, Кызылординской области.


Заболел остро, отмечает повышение температуры тела, слабость, головную боль,
болезненное образование в правой паховой области. Температура тела 40,00С. В паховой
области справа увеличенный лимфоузел, болезненный, неподвижный, с нечеткими
границами, кожа над ним гиперемированы. В области передней поверхности правой
голени пустула, наполнена темно-кровавым содержимым, болезненная. Вас вызвали к
пациенту на дом. Ваша тактика?
a) +в первую очередь сообщить о случае чумы по цепочке экстренного оповещения
b) в первую очередь надеть противочумный костюм
c) немедленно начать детоксикационную терапию
d) вызвать бригаду скорой помощи на себя
e) начать экстренную химиопрофилактику всех контактных лиц

250) Пациент 17 лет, при лечении в дневном стационаре ему сделали цефазолин в/м. Через
5 минут у больного началась выраженная одышка, отек слизистой рта и языка, бледность,
потливость. Пульс слабый нитевидный. Какой препарат НАИБОЛЕЕ целесообразно
ввести пациенту на первом этапе оказания неотложной помощи?
a) Супрастин
b) Димедрол
c) Триамциналон
d) +Адреналин
e) Допамин

251) У пациента с аллергией на препараты пенициллинового ряда был купирован


анафилактический шок. Какая дальнейшая тактика ведения пациента НАИБОЛЕЕ
целесообразна?
a) Ничего не назначать, но оставить под наблюдением на 24 часа
b) Назначить супрастин и отпустить домой
c) Назначить преднизолон 5 мг и отпустить домой
d) +Назначить тавегил в\м и оставить под наблюдением на 48-72 часа
e) Назначить адсорбикс и наблюдать не менее 7 дней

252) Эпинефрин протокол асматический, а потом уже метил преднезолон статус .


Больной 22 лет, страдающий бронхиальной астмой, вызвал врача на дом. Объективно:
возбужден, температура тела 36,7°С, ЧСС - 120 ударов в мин., число дыханий - 24 в мин.
При аускультации: дыхание резко ослаблено, единичные сухие хрипы. Из анамнеза: в
течение суток получил более 10 ингаляций беротека без эффекта. С какого препарата
следует начать оказание неотложной помощи?
a) Сальбутамол
b) Тиосульфат натрия
c) Эуфиллин
d) +Метилпреднизолон
e) Будесонид

253) Девушка 16 лет, студентка колледжа. Потеряла сознание во время собрания в


душном помещении на 1-1,5 минуты. Спутанности сознания после обморока не было.
Перед обмороком почувствовала тошноту. Отрицает заболевания сердца, прием
лекарственных препаратов, беременность. ЧСС 86 уд/мин, АД 110/70 мм рт.ст. Какая
дальнейшая тактика наиболее целесообразна?
a) Девушка здорова, отправить домой
b) Оставить под наблюдением на 24 часа
c) +Снять электрокардиограмму
d) Необходима консультация невропатолога
e) Снять электроэнцефалограмму
254) Пациентка 35 лет. Муж нашел ее без сознания около стола в не естественной позе
(рука была «как будто вывернута»). Когда пришла в себя – несколько минут была
дезориентирована. Язык оказался прикушен. Вызвал скорую помощь. Травмы, операции,
заболевания сердца отрицает. Что предшествовало обмороку – не помнит. Какая
дальнейшая тактика наиболее целесообразна?
a) Женщина здорова, отправить домой
b) Оставить под наблюдением на 24 часа
c) Снять электрокардиограмму
d) +Снять электроэнцефалограмму
e) Выполнить эхокардиограмму

255) Женщина 39 лет. Жалобы на боли в низу живота, нарастающие. Боль появилась
утром, при попытке встать с постели потеряла сознание. Болезненность при пальпации в
правом нижнем квадранте. ЧСС 95 уд/мин, АД 90/60 мм рт. Температура – 36,4°С. ЭКГ –
без патологии. Гематокрит – 36%. При попытке подняться – потеряла сознание. Какая
дальнейшая тактика наиболее целесообразна?
a) Сделать КТ брюшной полости
b) Оставить под наблюдением на 24 часа
c) +Выполнить УЗИ органов малого таза
d) Снять электроэнцефалограмму
e) Выполнить эхокардиограмму

256) У пациента с тяжелой пневмонией нарастает одышка и тахикардия, а АД упало до


80/40 мм рт.ст. Диагностирован инфекционно-токсический шок. Начата инфузионная
терапия. Какой препарат НАИБОЛЕЕ целесообразно добавить при интенсивной терапии
шока?
a) Атропин
b) Адреналин
c) +Допамин
d) Корглюкон
e) Гепарин

257) Мужчина 32 лет, после контакта с кошкой через 2 часа появилось удушье, свистящее
дыхание с удлиненным выдохом. Ранее при контакте с кошкой появлялся зуд глаз, носа,
чихание, слезотечение, ринорея и кашель. Какой препарат позволит купировать данное
состояние?
a) кетотифен
b) дезлоратадин
c) ксимтазолин
d) +сальбутамол
e) монтелукаст

258) Мужчина 67 лет. Жалобы на боли за грудиной, иррадиируют в правую руку, в


течение 30 мин, чувство нехватки воздуха. Самостоятельно разжевал таблетку аспирина.
На ЭКГ (фото). С какого препарата следует начать оказание неотложной помощи?
a) Пропранолол
b) +Морфин
c) альтеплаза
d) клопидогрель
e) гепарин

259) Мужчина 65 лет страдает артериальной гипертензией, изредка принимает энам в


таблетках. После стресса, час назад появилась резкая головная боль, ноющие боли в
области сердца, рвота, головокружение, «туман в глазах», сужение полей зрения, слабость
в правых конечностях, сухожильные рефлексы справа не вызываются. АД 220/110
мм.рт.ст. На ЭКГ, синусовый ритм, ЧСС 60 в мин. В RV5 >RV4 косо-нисходящее
смещение ST с от изолинии, зубец T отрицательный. Какой диагноз?
a) Гипертонический криз осложненный отеком сетчатки
b) Гипертонический криз, церебральный вариант
c) +Гипертонический криз, ОНМК
d) Гипертонический криз, отек мозга
e) Гипертонический криз, острая энцефалопатия

260) 45 лет после празднования с обильным застольем ночью беспокоила изжога и утром
появилась обильная кровавая рвота, сильная слабость. Доставлен бригадой скорой
помощи, которая начала инфузионную терапию 5% раствором глюкозы. Объективно:
состояние тяжелое, кожа сухая, бледная, телеангиоэктазии, расширены вены на передней
брюшной стенке, живот увеличен за счет умеренного асцита, печень выступает из под
реберной дуги на 2 см. Рвотные массы в виде темных кровяных сгустков объемом до 0,5 л.
АД – 100/70 мм рт.ст. Гемоглобин – 88 г/л. Какие неотложные мероприятия необходимо
провести?
a) +Установка зонда Блэкмора
b) Ввести аминокапроновую кислоту внутривенно
c) Начать инфузию свежезамороженной плазмы
d) ФЭГДС и эндоскопическое склерозирование, окреотид
e) Ввести нитроглицерин и вазопрессин и соматостатин

261) Мужчина 47 лет после двигательного и психического возбуждения внезапно потерял


сознание. При осмотре: кожные покровы влажные, на коже следы инъекций. Отмечается
подергивание мышц лица, зрачки расширены. ЧДД - 32 в минуту. Назначение какого
препарата НАИБОЛЕЕ целесообразно?
a) +40% раствор глюкозы
b) Натрия хлорида 0,9% раствора
c) Натрия хлорида 0,45% раствора
d) Натрия бикарбоната 2% раствора
e) Инсулина короткого действия 30 ЕД

262) Женщина 56 лет. Жалобы дискомфорт в эпигастрии, чувство переполнения после


еды, отрыжку воздухом, иногда тухлым. В анамнезе - аутоиммунный тиреоидит. При
ФЭГДС: (фото). Ее также беспокоит слабость, жжение языка, парестезии. При осмотре
кожа бледная с желтушным оттенком. Какая картина крови будет выявлена наиболее
вероятно?
a) Лейкоцитоз
b) Лейкопения
c) гипохромная анемия
d) +гиперхромная анемия
e) нормохромная анемия

263) Мужчина, 46 лет, жалуется на тупые боли в правом подреберье, горечь во рту,
тошноту, горечь во рту, возникающие после погрешностей в диете в течение последних 3
лет. Периодически – запоры. При обследовании: ОАК: гемоглобин – 138 г/л, лейкоциты –
6,5х109/л, СОЭ - 18 мм/ч. БАК: общий билирубин - 14 ммоль/л, АСТ - 32 ЕД/л, АЛТ - 28
ЕД/л, амилаза - 87 ЕД/л, ЩФ - 56 ЕД/л. УЗИ ОБП: Общий желчный проток не
визуализируется. Желчный пузырь: тонус его понижен, размеры несколько увеличены,
стенка желчного пузыря утолщена (6 мм), конкрементов нет. Какая лечебная тактика?
a) ЛИВ-52
b) креон
c) +дротаверин
d) лансопразол
e) урсодезоксихолевая кислота (только в ремиссию)

264) Женщина 52 лет. Жалобы на слабость, утомляемость, боли в суставах, иногда


высыпания, которые она принимает на аллергию, запоры. Данные жалобы появились 2
года назад, когда стала быстро набирать вес на фоне менопаузы (ИМТ - 32). В тот период
при обследовании было выявлено: билирубин – 54 ммоль/л, АЛТ – 67 МЕ/мл, АСТ – 44
МЕ/мл. В анамнезе – 3 беременности, 2 родов, 1 медаборт. Сделан скриннинг на гепатиты:
HBs-Ag отрицательно, anti HCV total - отрицательно. Билирубин – 27 ммоль/л, АЛТ – 124
МЕ, АСТ – 88 МЕ. Какой следующий шаг в диагностике?
a) +Полная маркерная диагностика на гепатит В
b) ПЦР HBV ДНК
c) Эластография печени
d) УЗИ органов брюшной полости
e) АНА, АМА

265) У мужчины 48 лет, ИМТ – 35. При обследовании выявлены увеличение печени,
признаки жирового гепатоза (УЗИ), дискинезия желчевыводящих путей. HBsAgотр., анти-
HCV суммарные - отр. Биохимия: глюкоза – 8.1 ммоль/л, АЛТ – 1,74 ммоль/л. АСТ – 1,25
ммоль/л. Билирубин – 35,25 мкмоль/л, прямой – 5,95 мкмоль/л, непрямой – 29,3
мкмоль/л, ГГТП – 112 ед/л, ЩФ – 185,5 ед/л, амилаза – 120 ед/л. Общий белок – 58 г/л.
HbA1c – 7,5%. В чем наиболее вероятная причина цитолического синдрома?
a) +Стеатогепатит
b) вирусный гепатит
c) токсический гепатит
d) лекарственный гепатит
e) жировая дистрофия печени

267) У больного 42 лет при обследовании выявлена следующая картина УЗИ: Передне-
задний размер правой доли 13,8 см, левой доли – 7 см, паренхима однородная,
эхоплотность повышена. Воротная вена печени 1,2 см. Желчный пузырь S-образной
формы 7,3 х 2,1 см, толщина стенки – 0,5 см, просвет гомогенный. Селезенка 65 см2. РФ –
450 МЕ/мл. АНА – положительные. О какой патологии следует думать в первую очередь?
a) Хронический вирусный гепатит
b) +Системная красная волчанка (печень в норме)
c) Хронический холецистит
d) Цирроз печени
e) Аутоиммунный гепатит

268) Мужчина, ветеринарный врач 50 лет, обратился с жалобами на отсутствие аппетита,


слабость, зуд кожных покровов, одышку, увеличение живота в течение последнего
месяца, боли в ногах. В анамнезе: употреблял еженедельно 160 мл этанола. Последние три
года периодически отмечалась умеренная желтуха, сопровождающаяся кожным зудом и
темным окрашиванием мочи, проходящая самостоятельно. При осмотре пониженного
питания, иктеричность кожных покровов со следами расчесов, тургор снижен, на груди и
спине сосудистые звездочки, тремор рук, контрактура Дюпюитрена. Живот увеличен в
объеме за счет асцита, пупочная грыжа из-за чего печень и селезенку пальпировать не
удается. Какой диагноз наиболее вероятен?
a) +Цирроз печени алкогольной этиологии.
b) Криптогенный цирроз печени.
c) Первичный билиарный цирроз печени.
d) Цирроз печени вирусной этиологии.
e) Гепатоцеллюлярная карцинома

269) Женщина 46 лет, учитель в школе, обратилась к врачу с жалобами на боль и


припухлость в проекции височно-нижнечелюстных суставов, трудно открывать рот,
одновременно появилась огрубление, а затем и осиплость голоса; заболела остро через 2
недели после ОРЗ. Назначенная участковым врачом физиотерапия (УВЧ) привела к
ухудшению состояния: боль и припухлость усилились, поднялась температура до 37,5°.
Объективно: локальная температура, покраснение и болезненность височно-
нижнечелюстных суставов. ОАК: Эр – 3,6, Нб – 118 г/л, Л- 9,9 *109/л, Нф – 67%, Лф –
29%, СОЭ 28 мм/ч. РФ – 52 МЕ/мл. РА отр., РХ – слабоположительная. Какой диагноз
НАИБОЛЕЕ вероятен?
a) бруцеллезный артрит
b) септический артрит
c) синдром Cтилла
d) +ревматоидный артрит
e) реактивный артрит

270) У женщины жалобы на скованность и боли в кистях рук, утомляемость, повышение


температуры до 37,8°, снижение массы тела на 2 кг, боли в правой половине грудной
клетки. На лице макулезная сыпь, выявлены алопеция и афты в полости рта. Р-графия
кистей: околосуставной остеопороз, отек мягких тканей. Р-графия ОГК: небольшой выпот
справа. ОАК: нормохромная анемия, лимфопения, СОЭ – 28 мм/ч. ОАМ: белок – 0,88 г/л,
эритроциты – 4-5 в п/з. СРБ +, РФ –отр, АНА – положительные. Какие антитела наиболее
специфичны для данного заболевания?
a) Перинуклеарныеантинейтрофильные
b) Цитоплазматические антинейтрофильные
c) +Антитела к двунитевой ДНК СКВ
d) Антимитохондриальные антитела
e) Антитета к рибонуклеопротеину

271) Пациенту выставлен диагноз неспецифического аортоартериита с поражением


брюшного отедела аорты и почечной артерии. Необходимо назначение кортикостероидов.
Какой побочный эффект кортикостероидов потребует наибольшего внимания со стороны
врача в данной ситуации?
a) +Артериальная гипертензия
b) Стероидный диабет
c) Остеопороз
d) Остеонекроз
e) Кровотечение из ЖКТ

272) Женщина 48 лет, обратилась с жалобами на приступы сердцебиения,


сопровождающееся ощущением нехватки воздуха, общей слабостью; потливость,
снижение веса, тремор рук, раздражительность. При обследовании в момент приступа на
ЭКГ (фото). Какое обследование наиболее целесообразно?
a) +ТТГ, Т3 и Т4
b) Эхокардиография
c) Холтеровское мониторирование ЭКГ
d) ВЭМ-тест
e) уровень антител к ТПО и ТГ

273) Женщина 33 года, обратилась с жалобами на массивные отеки. Со слов больна около
недели, после перенесенного ОРЗ, появились отеки с нарастанием, урежением диуреза.
Также беспокоят высыпания по типу крапивницы, боли в суставах. Anti-HCV IgM, IgG –
отрицательные, ПЦР HCV РНК – положительный. РФ – положительный, АНА -
отрицательные. В моче: суточная протеинурия 5 г в сутки, в крови: общий белок 52 г/л,
альбумин – 23 г/л, холестерин 7,5ммоль/л. Какой НАИБОЛЕЕ вероятный
предварительный диагноз?
a) +Хронический гломерулонефрит, ассоциированный с HCV
b) Острый гломерулонефрит
c) Геморрагический васкулит с поражением почек
d) Системная красная волчанка, волчаночный нефрит
e) IgA-нефропатия

274) Участковый врач осматривает ребенка 1 года по вызову на дому. Жалобы матери на
повышение температуры тела до 38,80С, насморк, кашель в течение 4 дней. При осмотре,
одышка, периоральный цианоз, адинамия, сонливость, грудь сосет неохотно. В легких
выслушиваются мелкопузырчатые хрипы, больше справа. ЧД 62 в мин. Сердечные тоны
приглушены, ЧСС 138 в мин. . SO2 92 %. Что из перечисленного требует госпитализации
в стационар:
a) тахипноэ, тахикардия
b) повышение температуры, нарушение сна
c) +вялость, сонливость, снижение аппетита
d) влажные мелкопузырчатые хрипы в легких
e) показатель сатурации кислорода

275) Ребенок 1 год. Жалобы на одышку, кашель, нарушение аппетита, вялость,


сонливость. Температура - 39,1оС. На рентгенограмме – очагово-сливная инфильтрация в
правой нижнемедиальной зоне. После назначенного лечения цефтриаксоном на 3-е сутки
температура в вечерние часы 37,6оС. Сон восстановился, аппетит лучше, стал активнее
днем. Ваша дальнейшая тактика?
a) перевести ребенка на альтернирующий режим введения антибиотика
b) заменить антибиотик на цефалоспорины 4 поколения
c) продолжить тот же антибиотик до нормализации температуры
d) сделать бак.посев мокроты и определить чувствительность возбудителя
e) +продолжить антибиотик до полного разрешения процесса на рентгенограмме

276) Ребенок 3 лет. Начало заболевания с катаральных явлений со стороны верхних


дыхательных путей, конъюнктивита, кашля. Температура 37,80С, интоксикация не
выражена. Получал ибуфен, ингаляции физ.раствора через небулайзер, амоксициллин.
Эффекта не было. Продолжает беспокоит кашель, спастического характера, иногда
заканчивается рвотой. Мокрота скудная, серовато-желтого цвета. Мать ребенка отметил,
усиление и учащение кашля, несмотря на лечение. При осмотре состояние средней
тяжести, интоксикация и ДН не выражены. В легких влажные мелкопузырчатые и сухие
хрипы, преимущественно справа. Перкуторный звук не изменен. ОАК- умеренный
лейкоцитоз, с/ядерные 62%, СОЭ 28 мм/час. Какая дальнейшая тактика целесообразна?
a) Амоксиклав
b) Цефтриаксон
c) Флуконазол
d) +ровамицин
e) котримасазол

277) Новорожденный ребенок 10 дней. Мама жалоб не предъявляет. При осмотре в легких
ослабленное дыхание, хрипов нет. Сердечные тоны ритмичные, грубый систолический
шум над всей сердечной областью, punt.max. в 3-4 межреберье. Признаков нарушения
гемодинамики нет. Наиболее вероятный диагноз:
a) Фиброэластоз
b) ВПС - дефект межпредсердной перегородки
c) +ВПС - дефект межжелудочковой перегородки
d) ВПС - открытый артериальный проток
e) ВПС - транспозиция магистральных сосудов
278) Мальчик 12 лет. Жалобы на изжогу, отрыжку кислым, воздухом, боли за грудиной.
Изжога усиливается после приема пищи, наклоне, в положении лежа, после занятий
физкультуры. Объективно язык обложен беловатым налетом, кариес 2/2 нижних коренных
зубов. При пальпации умеренная болезненность в эпигастральной области. На ЭФГДС -
выраженная гиперемия и рыхлость слизистой оболочки нижней трети пищевода,
фибринозный налет, единичные эрозии, контактная кровоточивость слизистой оболочки.
Уреазный тест отрицательный. Препараты первого выбора:
a) Алмагель
b) +Омепразол
c) Домперидон
d) Фамотидин
e) Сукральфат

279) Мальчик 14 лет. Жалобы на слабость, головокружение, снижение аппетита. Стал


плохо переносить обычные спортивные нагрузки. Слабость и утомляемость объясняет
нежеланием есть мясо. При осмотре бледность кожи и слизистых оболочек, ладоней.
Язык бледный, сглажены сосочки. Дыхание везикулярное. Сердечные тоны приглушены,
систолический шум на верхушке, ЧСС 92 в мин. АД 100/60 мм.рт.ст. Живот мягкий,
болезненный в эпигастральной области, печень пальпируется на 1 см из-под реберной
дуги. ОАК: эритроциты 3,58х1012/л, HB 89 г/л, Ht 30 %, СОЭ 28 мм/час. Какое
обследование с наибольшей достоверностью позволит выявить причину данного
состояния:
a) дыхательный тест
b) +ЭФГДС с биопсией
c) УЗИ органов брюшной полости
d) Общий белок, АЛТ, АСТ, СРБ
e) стернальная пункция

280) Девочка 14 лет жалуется на боли в эпигастрии, ближе к левой половине,


иррадиирующей в спину слева. Боли возникают после еды, особенно обильной, жирной и
острой. Также боль провоцируется употреблением гамбургеров. Симптомы наблюдаются
в течение 6 мес. Из-за избыточного веса девочка периодически устраивает разгрузочные
дни. При полном голодании, приеме только воды боли исчезают. Стул неустойчивый,
чаще разжижженый. Наиболее информативный показатель для подтверждения диагноза:
a) +++УЗИ органов брюшной полости
b) МРТ органов брюшной полости
c) содержание амилазы, холестерина, триглицеридов
d) уровень эластазы-1
e) уровень гликированного гемоглобина
281) Девочка 9 мес. Жалобы на повышение температуры тела до 38,7ОС, отказ от груди,
рвота после кормления. Антенатальный и постнатальный анамнез без особенностей.
Прививки в срок, без реакций. При осмотре катаральных явлений со стороны верхних
дыхательных путей нет. Ригидность затылочных мышц. Кожные покровы бледные с
сероватым колоритом, губы суховаты. В легких жестковатое дыхание, хрипов нет, ЧД 52
в мин. Сердечные тоны звучные, ритмичные, ЧСС 142 в мин. Живот мягкий, болезненный
при пальпации. Мочится мало. Дизурия. Стул жидкий. Наиболее вероятный диагноз:
a) внебольничная пневмония
b) острая кишечная инфекция
c) острый гнойный отит
d) +вирусный менингит
e) инфекция мочевых путей
282) Мальчик 6 лет на приеме у участкового педиатра для оценки подготовленности к
обучению в школе. Жалоб нет. Болеет редко. За последний год перенес 3 раза ОРВИ без
осложнений. Какой из показателей требует консультации специалиста:
a) количество постоянных зубов – 4
b) положительный «филиппинский» тест
c) +тест на срисовывание написанного текста 3 балла
d) рост соответствует возрасту
e) масса тела соответствует длине тела

283) Ребенок 3 мес находится на естественном вскармливании. В весе хорошо прибавляет.


Сон не нарушен. Сосет активно, каждые 3-3,5 часа. Стул нерегулярный, 1 раз в 2-3 дня.
При дефекации натуживается. Периодически используют газоотводную трубку. Стул
чаще сформированный. Ваша тактика:
A. запрещение грудного вскармливания, перевести на лечебную смесь
B. грудное вскармливание чередовать с кисломолочной смесью
C. продолжить кормление грудью, ввести овощной прикорм
D. +продолжить кормление грудью, назначить лактулозу
E. продолжить кормлении грудью, назначить матери прием лактулозы

284) Девочка 4 месяцев. Родилась недоношенной, с весом 2100 грамм, ростом - 44см.
Растет и развивается соответственно возрасту. На диспансерном учете у невропатолога с
диагнозом: «Перинатальная энцефалопатия». Прививки получает в срок. Какая наиболее
вероятная вакцинация в этом возрасте?
a) АбКДС3 + ИПВ3+ Hib3
b) ККП + ИПВ4+пневмо
c) RV АбКДС + RV Hib
d) RV БЦЖ + RV АДС+ RV ККП
e) +АбКДС3 + ИПВ3+ Hib3+ВГВ

285) У пациента, 45 лет, после физической нагрузки появились схваткообразные боли по


всему животу. С течением времени появились тошнота, рвота и вздутие живота, перестали
отходить газы. Данный симптомокомплекс характерен для?
a) +острая кишечная непроходимость
b) прободная язва желудка
c) острый панкреатит
d) острый аппендицит
e) острый холецистит

286) Мужчина 45 лет, обратился с жалобами на боли в левой стопе и голени, возникающие
при ходьбе. Болен 4 года. Вначале мог пройти без остановки 500 - 600 м, в последнее
время не более 50 - 60 м. Курит до 30 сигарет в день. Кожные покровы левой стопы и
голени бледные, на ощупь холоднее симметричных участков правой нижней конечности.
Артериальная пульсация на левой стопе отсутствует, справа – ослаблена на тыле стопы.
На бедренных и подколенных артериях с обеих сторон артериальная пульсация
определяется отчетливо. Ваш диагноз?
a) облитерирующий атеросклероз
b) +облитирирующий эндоартериит
c) синдром Лериша
d) болезнь Рейно
e) тромбоз подколенной артерии
Атеросклероз нижних конечностей и облитерирующий эндоартериит отличаются друг от
друга то что в одном случае будут сохраняться пульсация на бедренных и подколенных
артериях это эндоартериит а при атеросклерозе вен нижних конечностей будет снижена
либо во все отсутсвовать пульсация на бедренной и подколенной артерий.

287) 8-ми месячный ребенок с жалобами на боли в животе, рвоту, стул. За 12 часов до
поступления ребенка накормили мантами. При пальпации живота в правой подвздошной
области пальпируется плотное образование. Какова НАИБОЛЕЕ вероятная причина
заболевания.
a) тонко-тонкокишечная инвагинация
b) +тонко-толстокишечная инвагинация
c) энтеровирусная инфекция
d) опухоль Вильмса
e) пищевое отравление

288) Мужчина 50 лет, обратился с жалобами на сильную одышку, усиливающуюся при


минимальной физической нагрузке. В анамнезе перенесенный туберкулез легких. Все
началось 1,5 часа назад на работе. При подъеме тяжести появилась внезапная боль в левой
половине грудной клетки и кашель. При перкуссии- тимпанит над всей поверхностью
правой половины грудной клетки. Дыхание справа не прослушивается. На
рентгенограмме грудной клетки - смещение сердца влево. Пульс 122 уд/мин. Какой
диагноз?
a) +Спонтанный пневмоторакс
b) Тромбэмболия легочной артерии
c) Ателектаз правого легкого
d) Ателектаз левого легкого
e) Колапс левого легкого

289) На приеме в поликлинике при первичном осмотре ребенка 3 мес возраста,


определяется ассиметрия кожных складок на бедрах, симптом ограничения отведения
левого бедра. Других отклонений не выявлено. Какая наиболее правильная тактика
ведения данного ребенка?
a) наблюдение с повторным осмотром через 3 мес
b) назначить консультацию невропатолога
c) +назначить рентгенографию тазобедренных суставов
d) назначить КТ тазобедренных суставов
e) назначить ЛФК и общий массаж

290) У вас на приеме ребенок 3 дней жизни. Мама жалуется на плач ребенка при
пеленании. При пальпации правой ключицы ребенок начинает беспокоится, при движения
в правом плечевом суставе. Какая НАИБОЛЕЕ подходящая тактика лечения?
a) гипсовая иммобилизация в физиологическом положении
b) гипсовая повязка Дезо
c) +мягкая повязка Дезо
d) иммобилизации не требуется
e) восьмиобразная повязка.

291) После родов у женщины развилась тревога, снизилось настроение, она не ест, очень
взволнована и выражает опасения, что ее ребенок страдает неизлечимой болезнью, просит
помощи. Какой из нижеперечисленных синдромов наиболее вероятен?
a) Обсессивный
b) Маниакальный
c) Астенический
d) +Депрессивный
e) Бредовый
292) Женщина 42 лет. На фоне злоупотребления у нее наблюдаются нарушения памяти в
форме фиксационной и ретроградной амнезии. Провалы памяти больной заполняет
вымыслами, «вспоминает» то, чего на самом деле не было. Также, во время беседы,
рассказывает о событиях, действительно происходивших, но искаженных, неверно
соотнесенных во времени или месте. Данному расстройству сопутствует полинейропатия.
Какой из нижеперечисленных синдромов наиболее вероятен?
a) Кандинского-Клерамбо
b) Депрессивный
c) Психоорганический
d) +Корсакова
e) Делириозный

293) У мужчины 30 лет, заболел остро с высокой температурой – 41°С. В вечернее время
усиливается тревога, беспокойство, говорит что видит как по комнате «летают желтые
тумбочки», он слышит музыку. Не спит, беспокоен. В дневное время поведение
упорядочивается, однако к вечеру вновь становится беспокойным. Какой из
нижеперечисленных синдромов наиболее вероятен?
a) Кандинского-Клерамбо
b) Депрессивный
c) Психоорганический
d) Синдром Корсакова
e) +Делириозный

294)
У девушки 17 лет грипп с высокой температурой. При осмотре беспокойна, тревожна,
суетлива, отвлекаема. Не знает где находится, в окружающей обстановке ориентировка
нарушена. Часто вскакивает с постели, что-то ищет под подушкой, заглядывает под
кровать, копается в постельном белье, шарит рукой по стене, пытаясь поймать
воображаемых насекомых. Временами испуганно озирается, разговаривает сама с собой.
Какая тактика наиболее целесообразна?
a) начать лечение осельтамавиром
b) начать детоксикационную терапию
c) начать транквилизаторы
d) госпитализировать в психдиспансер
e) +госпитализировать в инфекционную больницу

295) Мужчина 40 лет, социально благополучный, обращается с затяжными приступами


болей, напоминающих почечную колику. Тянущие боли в области поясницы сохранялись
в течении 4-х недель. При обследовании, включающем УЗИ и рентгенографию, патологии
со стороны почек не выявлено. Назначение спазмолитиков эффекта не дало. Больной
тревожится о своем будущем, плохо спит, хуже себя чувствует утром, вечером боли
практически не беспокоят. Считает что врачи не хотят обращать должного внимания его
заболеванию. Какой из нижеперечисленных диагнозов наиболее вероятный
a) Фобический синдром
b) Шизофрения
c) +Маскированная депрессия
d) Неврастения
e) Нераспознанная органная патология

296) Подросток 14 лет. Вернулся домой после встречи с друзьями в необычном состоянии.
Зрачки широкие, глаза блестят, склеры гиперемированы. Потребовал у матери дать ему
поесть, торопил ее когда она готовила еду, нашел себе холодного супа и с жадностью поел
его. Съел первое, руками стал доставать из кастрюли жаркое и есть. После этого ушел в
свою комнату и уснул. Мать отметила, что от одежды сына исходил сладковатый запах. В
течение 4-х последующих дней был раздражительным, жаловался на слабость. Какой из
нижеуказанных наркотиков наиболее вероятно употребил подросток?
a) +Марихуана
b) Героин
c) Эфедрон
d) «Экстази»
e) ЛСД

297) Мужчина 39 лет заболел остро, сегодня утром, когда возникла сильная головная боль
и многократная рвота, повысилась температура тела до 40°, сердцебиение, одышка,
судороги, днем была вызвана скорая помощь и больной доставлен в больницу. Со слов
родственников- 2 недели получал лечение по поводу пневмонии, страдает алкоголизмом.
В неврологическом статусе: сознание расстроено по типу сопора, зрачки D=S,
положительный симптом Манн-Гуревича. Парезов конечностей нет. Сухожильные
рефлексы равномерно оживлены. Ригидность мышц затылка 4 см, симптом Кернига под
углом 90°. Положительный нижний симптом Брудзинского. Ликвор мутный,
зеленоватый вытекает под давлением 300мм вод.ст. Цитоз – 2330 клеток в 1 мкл за счет
нейтрофилов, белок – 2,5 г/л, глюкоза – 0,20 г/л. Какой из нижеперечисленных диагнозов
наиболее вероятный?
a) серозный менигит
b) туберкулезный менингит
c) менингококковый менингит
d) +пневмококковый менингит
e) спиная сухотка (нейросифилис)

298)
Женщина 25 лет жалуется на чувство онемения и слабости в ногах, шаткость при ходьбе,
учащенное мочеиспускание, которое держится в течение 3-х дней. Болеет 3 года, когда
после стресса возникло онемение ног, которое держалось в течение 3-х недель и
самостоятельно прошло. Через год появилась слабость в ногах и шаткость при ходьбе,
которые прошли после гормональной терапии. Через 2 года слабость в ногах
возобновилась, и появились тазовые расстройства в виде императивных позывов. При
осмотре: горизонтальный нистагм, нижний спастический парапарез, мозжечковая атаксия,
императивные позывы при мочеиспускании. На МРТ головного мозга: множественные
гиперинтенсивные очаги демиелинизации в белом веществе полушарий головного мозга.
Какой из нижеперечисленных клинических диагнозов наиболее вероятный?
a) боковой амиотрофический склероз
b) острый миелит
c) сирингомиелия
d) +рассеянный склероз
e) синдром Гийена-Барре

299) Юноша 17 лет жалуется на двоение при взгляде вправо, слабость в ногах,
пошатывание при ходьбе, периодическую задержку мочеиспускания. В 15 лет - в течение
3-5 дней отмечал двоение при взгляде вправо. Месяц назад после стресса почувствовал
слабость в ногах и шаткость при ходьбе. В неврологическом статусе: легкое сходящееся
косоглазие справа, диплопия при взгляде вправо, скандированная речь, нижний
спастический парапарез с высокими сухожильными, отсутствуют брюшные рефлексы,
неустойчивость в позе Ромберга, походка с широко расставленными ногами. На глазном
дне – побледнение височных половин сосков зрительных нервов. Какой из
нижеперечисленных клинических диагнозов наиболее вероятный?
a) боковой амиотрофический склероз
b) +рассеянный склероз
c) острый рассеяный энцефаломиелит
d) сирингомиелия
e) полиомиелит

300) Юноша 18 лет, внезапно потерял сознание на короткий отрезок времени. При
осмотре: сознания- оглушение, головная боль, рвота, ригидность затылочных мышц,
симптом Кернига и Брудзинского. Брадикардия, повышение температуры до 38°С. В
общем анализе крови лейкоцитоз. Больной пропунктирован. Спинномозговая жидкость
окрашена в красный или желтоватый цвет. Какой дополнительный метод диагностики
необходимо назначить для уточнения диагноза?
a) эхо-энцефалография
b) +КТ головного мозга
c) Электроэнцефалография
d) Реоэнцефалография
e) Коагулограмма

301) У женщины 35 лет, после стресса, появились приступы сердцебиения, головной боли,
повышения АД до 170 мм рт.ст., повышенной потливости, ознобоподобного дрожания,
повышения температуры тела, с выраженным чувством страха и тревоги. В конце
приступа наблюдается обильное мочеиспускание. Сознание больной не нарушается.
Продолжительность приступа от 40 минут до часа. В неврологическом статусе:
органическая симптоматика отсутствует. Выражены вегетативные нарушения: зрачки
широкие D=S, белый возвышенный дермографизм, выражен общий гипергидроз, тремор
век и пальцев рук. Какой из нижеперечисленных диагнозов наиболее вероятный?
a) вегетативныей криз
b) +паническая атака
c) гипертензионный синдром
d) симпато-адреналовый криз
e) гипертонический синдром

302) Женщина 54 лет жалуется на боль в пояснице, которая иррадиирует в правую ногу.
Со слов боль появилась после подъема тяжести, стала усиливаться при движении, при
длительном нахождении в одной позе. Положение на левом боку, с согнутой правой ногой
в коленном и тазобедренном суставах - приносило облегчение. Объективно: спина
фиксирована в слегка согнутом положении, резко ограничен наклон в больную сторону,
отмечается выраженное напряжение паравертебральных мышц. Боль локализована по
задней поверхности правой ноги до 5пальца и пятки. Чувствительность снижена по
наружной поверхности стопы, подошвы справа. Симптом Ласега положителен справа,
Ахиллов рефлекс справа отсутствует. На каком уровне позвоночного столба наиболее
вероятно патология?
a) Th 12- L1
b) L1-L2
c) +L5-S1
d) L3-L4
e) L2-L3

303) Вызов врача общей практики на дом. После употребления вяленой рыбы, купленной
на стихийном рынке, у 25-летнего мужчины через 16 часов появились тошнота, рвота,
слабость, вялость, двоение в глазах. Объективно: снижение мышечного тонуса,
анизокория, вялый глоточный и сухожильный рефлексы. Ваш предварительный диагноз?
a) Пищевая токсикоинфекция.
b) Острый энцефалит.
c) +Ботулизм.
d) Сальмонеллёз.
e) Острый менингит.

304) Участковый врач пришел по вызову к 2-х летнему ребенку. Ребенок посещает
детский сад, болен 2-й день. Жалобы на повышение температуры тела 38 -39º, головную
боль, вялость, однократную рвоту, сыпь на теле. Высыпания на коже появляются к вечеру
или на следующий день с момента острого проявления заболевания. Сыпь мелкая, имеет
точечную форму и яркий красный цвет. Началось высыпание с области шеи и лица
образованием мелких пятнышек. Далее сыпь возникла на внутренней поверхности бедер,
на туловище: груди, по бокам живота. Зона носогубного треугольника свободна от сыпи.
При фарингоскопии – «пылающий зев»». Ваш предварительный диагноз?
a) Корь
b) Псевдотуберкулез
c) +Скарлатина
d) Краснуха
e) Ветряная оспа

305) Ребенок 6-ти лет. Со слов матери ребенок болен 10 дней, начало с подъема
температуры до 37,8⁰С, слабости, головной боли, снижения аппетита. В динамике
температура поднималась до 38,7⁰С, появился озноб, чередующийся с потоотделением,
боли в нижних конечностях и мышцах. Из эпиданамнеза: мама поила ребенка свежим
козьим молоком. При осмотре: повышенная потливость. Шейные и паховые
лимфатические узлы увеличены. Боли в области коленных и голеностопных суставов.
Гепатоспленомегалия. В ОАК: гипохромная анемия, лейкопения, лимфоцитоз,
тромбоцитопения, ускоренная СОЭ. Какие из ниже перечисленных возбудителей может
быть наиболее вероятной причиной данного заболевания?
a) Listeria monocytogenes
b) Legionella pneumophila
c) Iersinia enterocolitica
d) Iersinia pestis
e) +Brucellamelitensis

306)
Новорожденный родился с желтухой, множественными геморрагическими проявлениями,
ВПС в виде дефекта межжелудочковой перегородки, катарактой. Из анамнеза: мать
ребенка во время беременности перенесла ОРВИ, аллергическая сыпь. Какие из ниже
перечисленных возбудителей может быть наиболее вероятной причиной заболевания?
a) Риккетсии
b) Хламидии
c) +Вирус
d) Грибы
e) Бактерии

307) У ребенка 5-ти лет отмечалось повышение температуры тела до 40оС, многократная
рвота, головная боль, через 10 часов от начала заболевания появилась звездчатая,
геморрагическая сыпь по телу, преимущественно в области ягодиц, голени, бедер.
Наиболее вероятный диагноз:
a) +менингококковая инфекция
b) стрептококковая инфекция
c) геморрагический васкулит
d) тромбоцитопеническая пурпура
e) иерсиниозная инфекция

308) Ребенок 3-х лет. Болен в течении 3-х дней. Начало с подъема Т тела, вялости,
покашливания, насморка, конъюнктивита. На 4-й день все симптомы усилились, t 39оС, на
лице появилась розовая, пятнисто-папулезная сыпь, которая на 5-й день распространилась
на туловище, на следующий день и на конечности. За 10 дней до начала заболевания был
контакт с родственником, у которого был кашель и «аллергическая сыпь». Наиболее
вероятный предварительный диагноз
a) +Корь
b) Скарлатина
c) герпетическая ангина
d) листериоз
e) ветряная оспа

309) Мужчина 50 лет. В прошлом году прошел полный курс терапии по 1 категории по
поводу впервые выявленного инфильтративного туберкулеза верхней доли левого
легкого, в фазе распада, МБТ+. Достигнут терапевтический эффект, прекратилось
бактериовыделение, однако на месте распада сформировалась каверна. На рентгенограмме
слева в верхней доле видна тонкостенная кольцевидная тень с четкими контурами.
Какой из перечисленных ниже диагнозов наиболее вероятен?
a) инфильтративный туберкулез верхней доли левого легкого в фазе распада
b) фиброзно-кавернозный туберкулез верхней доли левого легкого
c) туберкулема верхней доли левого легкого в фазе распада
d) +кавернозный туберкулез верхней доли левого легкого
e) абсцедирующая пневмония слева

310) Отец новорожденного ребенка болен фиброзно-кавернозным туберкулезом легких.


Контакт больного отца с ребенком не желателен, но допустим:
a) сразу после выписки из роддома, если ребенок привит БЦЖ
b) контакт не опасен при грудном вскармливании
c) +через 6-8 недель после прививки БЦЖ
d) через 9-10 недель после прививки БЦЖ
e) после излечения больного отца

311) Женщина, 28 лет, после родов (ребенку 3 месяца) обратилась к врачу с жалобами на
головную боль, недомогание, повышенную утомляемость, раздражительность, нарушение
сна, светобоязнь, субфебрильную температуру 37,3оС. Больной себя считает в течение
недели. Объективно: общее состояние ближе к удовлетворительному, периферические
лимфоузлы не пальпируются, в легких везикулярное дыхание. Наблюдается ригидность
затылочных мышц на 1,5 п/п. На обзорной рентгенограмме ОГК в нижней доле справа
очаг Гона и кальцинаты в корнях справа. Какой из перечисленных методов обследования
является важным для своевременной диагностики развившегося осложнения?
a) биохимический анализ крови
b) продольная томография
c) +люмбальная пункция
d) общий анализ крови
e) КТ головного мозга

312) Мужчина 19 лет, обратился к терапевту с жалобами на увеличение шейных


лимфоузлов, их болезненность, повышение температуры до 38оС, слабость. При осмотре
кожа над поверхностью лимфлоузлов гиперемирована. При пальпации лимфоузлы
эластичны, подвижны, не спаяны между собой, болезненны. На рентгенограмме ОГК
петрификаты в корнях. Какой из перечисленных методов обследования является наиболее
информативным?
a) рентгенография лимфоузлов
b) туберкулинодиагностика
c) трепанобиопсия костного мозга
d) +Биопсия лимфоузлов
e) КТ органов грудной клетки

313) Женщина 38 лет, обратилась к врачу с жалобами на усталость и ночную боль в спине.
Боль усиливается при нагрузке. При осмотре наблюдается вынужденная осанка с
ригидностью мышц спины. При пальпации грудного отдела боль иррадирует в грудную
клетку и живот. Какой из перечисленных методов обследования является наиболее
важным для уточнения диагноза?
a) обзорная рентгенограмма в двух проекциях
b) компьютерная томография
c) ИФА на противбруцеллезные антитела
d) Артроскопия
e) +МРТ

314) Ребенку 6 лет, проведена туберкулиновая проба Манту с 2 ТЕ, результат - папула
размером 15 мм, Диаскинтест положительный. После дообследования ребенка выставлен
диагноз: «Гиперергическая реакция». Ребенок часто болеет простудными заболеваниями и
бронхитом. Какая Ваша тактика наиболее целесообразна?
a) провести химиопрофилактикуэтамбутолом и рифампицином 2 месяца
b) провести химиопрофилактику изониазидом и этамбутолом 2 месяца
c) +провести химиопрофилактику изониазидом 6 месяцев
d) провести химиопрофилактику изониазидом 3 месяца
e) провести лечение по 1 категории

315) Женщина 29 лет, через 6 месяцев после рождения второго ребенка, кормит грудью,
хочет надежную долговременную контрацепцию. Из анамнеза: хронический гастрит.
Каков наиболее оптимальный метод контрацепции в данном случае?
A. лактационная аменорея
B. +внутриматочная контрацепция
C. чистые прогестиновые контрацептивы
D. комбинированные оральные контрацептивы
E. барьерный метод (+каз)

316) Женщина 34 лет. Стоит на учете с диагнозом "Системная красная волчанка". Ей


планируется поведение биологической терапии. Ей необходим надежный метод
контрацепции. Какой оптимальный метод контрацепции в данном случае?
A. +внутриматочная спираль с медью
B. внутриматочная спираль с гормонами
C. чистые прогестиновые контрацептивы
D. комбинированные оральные контрацептивы
E. спермициды

317) Женщина 45 лет. Не замужем, в анамнезе 3 беременности, все - искусственное


прерывание беременности. Курит в течение 28 лет. Положенный скрининг не проходила.
При пальпации молочных желез заметила небольшое уплотнение в наружном верхнем
квадранте. Какая дальнейшая тактика?
a) направить на консультацию к гинекологу
b) направить на консультацию к маммологу
c) +выполнить маммографию
d) КТ органов грудной клетки
e) биопсия уплотненного участка

318) Женщина 65 лет. Обратилась с жалобами на увеличение живота, недомогание


слабость. УЗИ выявило свободную жидкость в брюшной полости в объеме более 1,5 л.
Размеры печени и селезнки в пределах нормы, диаметр v.portae - 1,0 см. Онкомаркеры -
ОММА (СА-125) резко повышен, альфа-фетопротеин - повышен в 3 раза. Какой
предварительный диагноз наиболее вероятен?
a) гепатоцеллюлярная карцинома
b) рак поджелудочной железы
c) +рак яичников
d) эндометриоидная киста
e) эндометриоз

319) Мужчина 36 лет. Стоит на учете с диагнозом хронический вирусный гепатит В.


Эластография печени - стадия F2 Metavir. Вирусная нагрузка менее 10 000 МЕ/мл. В
последние 3 месяца появился субфебрилитет, незудящие высыпания на голенях, после
которых остаются участки гиперпигментации. Альфа-фетопротеин повышен в 34 раза.
Какое обследование наиболее целесообразно на следующем этапе диагностики?
a) УЗИ печени и селезенки
b) +МРТ печени и селезенки
c) пункционная биопсия печени
d) АНА и АМА
e) СЕА и СА19-9

320) Больной 48 лет с диагнозом цирроз печени в исходе вирусного гепатита С,


компенсированный. Какое исследование следует проводить больному каждые 3 месяца с
целью раннего выявления возможного онкологического осложнения?
a) +АФП и УЗИ печени и селезенки
b) МРТ печени и селезенки
c) пункционная биопсия печени
d) ФЭГДС
e) СЕА и СА19-9

321) Мужчина 78 лет. Обратил внимание на то что в последнее время он часто мочится в
течение дня, просыпается по ночам 2-3 раза, чтобы помочиться. Струя мочи слабая и
прерывистая. Ощущение неполного опорожнения мочевого пузыря. При пальцевом
исследовании простата увеличена в размерах, плотноватой консистенции. Какое
обследование позволит уточнить диагноз?
a) Альфа-фетопротеин
b) раково-эмбриональный антиген
c) СА 125
d) +Простато-специфический антиген
e) СА19-9

322) Мужчина 36 лет. Строитель, работа связана с пребыванием на открытом воздухе, под
солнцем. При профилактическом осмотре врач обратил внимание на невус (фото). Со слов
пациента родинка стала увеличиваться в размере в последние месяцы. Какая дальнейшая
тактика наиболее целесообразна?
a) наблюдение, это пограничный невус
b) консультация дерматолога
c) +консультация онколога
d) определение уровня белка S100 до консультации онколога
e) биопсия невуса до консультации онколога

323) Ребенок 2,5 мес от 1 беременности и 1 родов. Жалобы матери на подъем


температуры до 37,8 ºС, беспокойство, кашель сухой, учащенное дыхание, срыгивание.
При осмотре семейный врач обратила внимание на одышку, тахипноэ, ЧД 72 в мин.,
периоральный цианоз, нечастый малопродуктивный кашель. Обструктивный синдром
отсутствует. Грудь сосет вяло, неохотно, устает. При аускультации легких – обилие
мелкопузырчатых хрипов, ослабление дыхания справа в подлопаточной области.
Сердечные тоны учащены 156 в 1 мин. Печень и селезенка не увеличены. Заболел
впервые. Жилищные условия неблагоприятные. От госпитализации отказалась. Учитывая
высокий риск развития осложнений, врач назначила антибактериальную терапию. Какой
из антимикробных препаратов наиболее предпочтителен:
a) амоксициллин внутрь
b) гентамицин в/м
c) азитромицин внутрь
d) +цефтриаксон в/м
e) цефазолин в/м

324) Женщина 53 лет, жалуется на ночные стреляющие боли в ногах, неустойчивость при
ходьбе особенно в темноте. В неврологическом статусе — зрачки узкие, D >S; реакция их
на свет отсутствует, на конвергенцию сохранена. Объем движений и мышечная сила в
руках и ногах достаточная. Гипотония в мышцах ног. Сухожильные и периостальные
рефлексы на руках живые. Коленные и ахилловы рефлексы не вызываются. Грубо
изменено мышечно-суставное чувство в пальцах ног. В позе Ромберга с закрытыми
глазами — резкое пошатывание. На глазном дне — диски зрительных нервов сероватого
цвета.
Какой предварительный диагноз наиболее вероятен?
*+Нейросифилис, спинная сухотка
* Нейробруцеллез
* Нейроревматизм
* ЛейкоэнцефалитШильдера
* Гнойный менингит

325) Какие медицинские учреждения имеют право выдавать лист временной


нетрудоспособности?
+А) государственные учреждения по оказанию первичной медико-санитарной помощи
В) организации по переливанию крови
С) травм пункты и приемный покой мед учреждения
D) санаторно-курортные учреждения
Е) Судебные учреждения

326) У женщины 30 лет жалобы на скованность и боли в кистях рук, утомляемость,


повышение температуры до 37,8°, снижение массы тела на 2 кг, боли в правой половине
грудной клетки. На лице макулезная сыпь, выявлены алопеция и афты в полости рта. Р-
графия кистей: околосуставной остеопороз, отек мягких тканей. Р-графия ОГК:
небольшой выпот справа. ОАК: нормохромная анемия, лимфопения, СОЭ – 28 мм/ч.
ОАМ: белок – 0,88 г/л, эритроциты – 4-5 в п/з. СРБ +, РФ –отр, АНА – положительные.
Какие антитела наиболее специфичны для данного заболевания?
a) Перинуклеарныеантинейтрофильные
b) Цитоплазматические антинейтрофильные
c) +Антитела к двунитевой ДНК СКВ
d) Антимитохондриальные антитела
e) Антитета к рибонуклеопротеину

327) У беременной 24 лет со сроком беременности 30 недель на приеме в поликлинике


впервые повысилось АД до 150/90 мм.рт.ст. Жалоб не предъявляет. Состояние
удовлетворительное. Отеков нет. Наиболее обоснованная тактика.
a) Дать внутрь таблетку допегита
b) Дать внутрь таблетку нифедипина
c) Ввести стартовую дозу сернокислой магнезии
d) +Повторно измерить артериальное давление
e) Госпитализировать в родильный дом.

328) Девушка-подросток, 16 лет, страдает себореей (увеличение сальности кожи), акне,


повышенное оволосение на руках, ногах, передней брюшной стенке. АД -111/74 мм рт ст.,
вес 45 кг, рост 152 см, ИМТ-19,5. Какой гестагенный компонент в составе гормональных
контрацептивов наиболее приемлем в данном случае?
А)левоногестрел
Б)дезогетрел
В)норгестимат
Г)дроспиренон+
Д)гестоден

329) Женщина 42 года. Жалобы схваткообразные боли в околопупочной области,


кашицеобразный обильный стул с примесью слизи до 7-8 раз в сутки, тяжесть в правом
подреберье, периодические высыпания на нижних конечностях, кожный зуд (особенно
ночью), слабость, снижение памяти, плохой сон, ночные кошмары, снижение аппетита.
Узловая эритема. Живот мягкий, при пальпации резко болезненный в околопупочной
области, умеренно болезненный в области правого подреберья. Печень по Курлову 12-10-
9см. Поверхность слегка бугристая, края неровные. Какое обследование позволит
верифицировать диагноз?
a) Обзорная рентгенография брюшной полости
b) КТ органов брюшной полости
c) Колоноскопия с биопсией +
d) Ректоромоноскопия
e) Ирригоскопия

330) Девочке 1 месяц. Ребенок от 1 беременности, родилась на 41-й неделе с весом-4100г,


Рост-54 см, роды физиологические. С рождения на грудном вскармливании. В последнее
время стала вяло сосать, быстро засыпает у груди, хорошо и много спит. Объективно
масса – 4800 г, рост -57 см. При осмотре беспокойная, с низким тембром голоса. Кожные
покровы чистые, сухие с икретичным оттенком. Отмечается пастозность лица, широкая
седловидная переносица, гипертелоризм глаз, низкое расположение ушных раковин. Тоны
сердца приглушены. ЧСС-100 уд/мин. Живот увеличен в объеме , пупочная грыжа,
мышечная гипотония. Стул 1 раз в сутки, кашицеобразный. Наружные половые органы по
женскому типу. Щитовидная железа не увеличена. Поставьте предварительный диагноз:
a) +врожденный гипотиреоз
b) паратрофия
c) болезнь Дауна
d) аутоиммунный тиреоидит
e) приобретенный гипотиреоз

331) Женщина 25 лет, учитель в школе, обратилась к врачу с жалобами на симметричные


боли и припухлость в проекции межфаланговых проксимальных и лучезапястных
суставах. Назначенная участковым врачом физиотерапия (УВЧ) привела к ухудшению
состояния: боль и припухлость усилились, поднялась температура до 37,5°. Объективно:
локальная температура, покраснение и болезненность межфаланговых проксимальных и
лучезапястных суставов. ОАК: Эр – 3,6, Нб – 118 г/л, Л- 9,9 *109/л, Нф – 67%, Лф – 29%,
СОЭ 28 мм/ч. РФ – 52 МЕ/мл. РА отр., РХ – слабоположительная. Какой диагноз
НАИБОЛЕЕ вероятен?
a) бруцеллезный артрит
b) септический артрит
c) синдром Cтилла
d) +ревматоидный артрит
e) реактивный артрит

332) Мальчик 14 лет, занимается в школе олимпийского резерва. Жалобы на слабость,


головокружение, снижение аппетита. Стал плохо переносить обычные спортивные
нагрузки. Слабость и утомляемость объясняет нежеланием есть мясо. При осмотре
бледность кожи и слизистых оболочек, ладоней. Язык бледный, сглажены сосочки.
Дыхание везикулярное. Сердечные тоны приглушены, систолический шум на верхушке,
ЧСС 92 в мин. АД 100/60 мм.рт.ст. Живот мягкий, болезненный в эпигастральной области,
печень пальпируется на 1 см из-под реберной дуги. ОАК: эритроциты 3,58х1012/л, HB 89
г/л, Ht 30 %, СОЭ 28 мм/час. Какое обследование с наибольшей достоверностью позволит
выявить причину данного состояния:
a) дыхательный тест
b) +ЭФГДС с биопсией
c) УЗИ органов брюшной полости
d) Общий белок, АЛТ, АСТ, СРБ
e) стернальная пункция

333) У больного К. 65 лет выявляется следующая симптоматика: высокая устойчивая


артериальная гиепртензия, утренние головные боли , локализующиеся в зтылочной
области, сердцебиение, ухудшение зрения – на глазном дне ангиоретинопатия IV степени,
систолический шум, выслушиваемый в эпигастрии. Какой диагноз НАИБОЛЕЕ вероятен?
a) Коарктация аорты
b) Расслаивающаяся аневризма аорты
c) +Атеросклероз брюшной аорты
d) Хронический гломерулонефрит
e) Узелковый периартериит

334) Мужчина 50 лет жалуется на избыточную прибавку в весе , общую слабость. При
осмотре: повышенного питания, ИМТ 32 кг/м2. При обследовании: гликемия 10 ммоль/л,
гликозилированный гемоглобин 8,4%. Целью патогенетической терапии НАИБОЛЕЕ
вероятно является?
a) Уменьшение всасывания глюкозы в кишечнике
b) Потенцирование действия экзогенного инсулина
c) Снижение чувствительности инсулиновых рецепторов
d) Стимулирование функции бета-клеток островков Лангерганса
e) Повышение чувствительности периферических тканей к инсулину

335) Мужчина, 46 лет, жалуется на тупые боли в правом подреберье, горечь во рту,
тошноту, горечь во рту, возникающие после погрешностей в диете в течение последних 3
лет. Периодически – запоры. При обследовании: ОАК: гемоглобин – 138 г/л, лейкоциты –
6,5х109/л, СОЭ - 18 мм/ч. БАК: общий билирубин - 14 ммоль/л, АСТ - 32 ЕД/л, АЛТ - 28
ЕД/л, амилаза - 87 ЕД/л, ЩФ - 56 ЕД/л. УЗИ ОБП: Общий желчный проток не
визуализируется. Желчный пузырь: тонус его понижен, размеры несколько увеличены,
стенка желчного пузыря утолщена (6 мм), конкрементов нет. Какая лечебная тактика?
a) ЛИВ-52
b) креон
c) +дротаверин
d) лансопразол
e) урсодезоксихолевая кислота (только в ремиссию)

Вам также может понравиться